Re: Quantum immortality

2019-09-22 Thread Bruno Marchal

> On 19 Sep 2019, at 12:22, Jason Resch  wrote:
> 
> 
> 
> On Thu, Sep 19, 2019, 5:02 AM John Clark  > wrote:
> On Thu, Sep 19, 2019 at 5:54 AM Jason Resch  > wrote:
> 
> > This is exactly the break that occurred going from mind-brain identity 
> > theory to multiple-realizability theories 
> > (functionalism/computationalism/mechanism). It's the conventional meaning 
> > implied by mechanism that there's no identity (one to one relationship) 
> > between a mind and any particular one if it's instantiations.
> 
> I've read that 3 times and I still can't figure out what it means.
> 
> Sorry I should have included references. This article explains it further:
> 
> https://en.m.wikipedia.org/wiki/Multiple_realizability 
> 
> 


OK. But note that some people (like Jones on this list, sometimes ago) who 
accept the multiple-realisability, but still insist that they have each to be 
implement physically. But that does not make sense once we say yes to the 
doctor, as a machine cannot feel any difference between being emulated by a 
physical reality or by the arithmetical reality. 

Yet, importantly, the machine can measure the palsuiblity of mechanism by doing 
experiments in the physical reality, and thanks to QM, Mechanism, and thus its 
immaterialism, is vindicated empirically. 

Bruno



> Jason
> 
> 
> 
> 
>  John K Clark
>  
> 
> 
> -- 
> You received this message because you are subscribed to the Google Groups 
> "Everything List" group.
> To unsubscribe from this group and stop receiving emails from it, send an 
> email to everything-list+unsubscr...@googlegroups.com 
> .
> To view this discussion on the web visit 
> https://groups.google.com/d/msgid/everything-list/CAJPayv3oTnjg%3DF-6GzfPpBcByj2uMikVmCqkOXE4Qd0BOgK0mw%40mail.gmail.com
>  
> .
> 
> -- 
> You received this message because you are subscribed to the Google Groups 
> "Everything List" group.
> To unsubscribe from this group and stop receiving emails from it, send an 
> email to everything-list+unsubscr...@googlegroups.com 
> .
> To view this discussion on the web visit 
> https://groups.google.com/d/msgid/everything-list/CA%2BBCJUgy1R-%2BuhCwCkeBFRxGqhScwg2LhxVBWqi5OzfwH5FbVw%40mail.gmail.com
>  
> .

-- 
You received this message because you are subscribed to the Google Groups 
"Everything List" group.
To unsubscribe from this group and stop receiving emails from it, send an email 
to everything-list+unsubscr...@googlegroups.com.
To view this discussion on the web visit 
https://groups.google.com/d/msgid/everything-list/6A09DDB9-19DC-419A-9D92-98A2FFD6CE7E%40ulb.ac.be.


Re: Quantum immortality

2019-09-19 Thread Bruno Marchal

> On 17 Sep 2019, at 00:56, Bruce Kellett  wrote:
> 
> On Tue, Sep 17, 2019 at 3:53 AM Bruno Marchal  <mailto:marc...@ulb.ac.be>> wrote:
> On 16 Sep 2019, at 05:51, 'Brent Meeker' via Everything List 
> mailto:everything-list@googlegroups.com>> 
> wrote:
>> On 9/15/2019 6:13 AM, Bruno Marchal wrote:
>>> And memory is fallible, and memory of age has no more meaning when your age 
>>> is bigger that the nameable or describable number, which happens very soon, 
>>> relatively, for the immortal being trying to keep track of their birthday. 
>>> 
>>> Immortality is when you are to old to be able to even name your age. After 
>>> that, you have always the same age.
>> 
>> Nice aphorisms.  But irrelevant.  The question is why don't we see almost 
>> everyone else as younger?  
> 
> That happens when we are not old enough, but also, we might always backtrack 
> to younger people when close to death or when dying, …
> 
> What utter nonsense. You cannot jump between Everett branches,

You don’t need to jump. You are in all branches simulating the relevant digital 
processes.



> so you cannot jump to a branch in which you were young. Physics rules out 
> backward causation and branch jumping.
>  
> … may be up to something like this video below, which is an oversimplifying 
> view (mixing G and G* all the time) of Neoplatonism, or theology close to 
> Mechanism:
> 
> https://www.youtube.com/watch?v=h6fcK_fRYaI 
> <https://www.youtube.com/watch?v=h6fcK_fRYaI>
> 
> You can get very old, and always believed that you are young, even a baby, 
> and be right on that, just by not memorising everything.
> 
> Losing you mind when you are very old is quite common, but that does not mean 
> that you become young again.

Indeed.

Bruno



> 
> Bruce
> 
> 
> Technological immortality is a sort of egotic complacency in the Samsara, and 
> a sort of Procrastination of Nirvana. But why not? There is something to 
> contemplate here, but here is only an aspect of a bigger and simpler reality. 
> And there is something to contemplate there too. With Mechanism, mathematics 
> can give a glimpse, and evacuate some fake certainties. Nature also used some 
> authoritative argument sometimes...
> 
> Bruno
> 
> -- 
> You received this message because you are subscribed to the Google Groups 
> "Everything List" group.
> To unsubscribe from this group and stop receiving emails from it, send an 
> email to everything-list+unsubscr...@googlegroups.com 
> <mailto:everything-list+unsubscr...@googlegroups.com>.
> To view this discussion on the web visit 
> https://groups.google.com/d/msgid/everything-list/CAFxXSLSy%3DtB8XgPieA0bEfyRX%3Dz1aXvYyWUp1e_DdhMWmQ0p-Q%40mail.gmail.com
>  
> <https://groups.google.com/d/msgid/everything-list/CAFxXSLSy%3DtB8XgPieA0bEfyRX%3Dz1aXvYyWUp1e_DdhMWmQ0p-Q%40mail.gmail.com?utm_medium=email&utm_source=footer>.

-- 
You received this message because you are subscribed to the Google Groups 
"Everything List" group.
To unsubscribe from this group and stop receiving emails from it, send an email 
to everything-list+unsubscr...@googlegroups.com.
To view this discussion on the web visit 
https://groups.google.com/d/msgid/everything-list/FEDCC32D-F1F5-4CFA-A9E6-B5758A6D4525%40ulb.ac.be.


Re: Quantum immortality

2019-09-19 Thread Bruno Marchal

> On 16 Sep 2019, at 14:17, John Clark  wrote:
> 
> On Sun, Sep 15, 2019 at 8:34 AM Bruno Marchal  > wrote:
> 
> > With mechanism [...]
> 
> Bruno, I really wish you wouldn't start long paragraphs with those two words 
> because when you do I don't know if I agree with you or not.


You agree with the THEORY (hypothesis/postulate), which is simply the "yes 
doctor”, the idea that the brain is digitally emulable, etc. That is explained 
in all my paper and in all my post.

Where you disagree is in the THEOREM. Your argument is that step 3 is flawed. 
You disagree also with the step 7, as it used the well know fact that the 
notion of computation is purely arithmetical, which is known (at least by 
logicians and theoretical computer scientists) since the 1930s.




> Please be more specific and spell out exactly what assumptions you're 
> starting from,

The assumption is that we can survive with an artficial physical brain.

The theorem is that in that case physics has to be reduced to a statistic on 
all computations, which exists and are well structured (differently for each 
points of view) in the arithmetical reality.



> your all purpose word "mechanism" just isn't good enough. I thought I knew 
> what "mechanism" mente today but quickly realized I was entirely wrong 
> because immediately after those two words you added  "you are not identical 
> with your atoms configurations”.

Indeed, once you accept the digital brain transplant, you understand that you 
can in principle change of body everyday, which illustrate that your identity 
is not in the atoms, but in the program implemented by your atom today, but by 
other atoms tomorrow. You can even survive with Babbage machine, which ahas no 
carbon atom.

All this sis well known. What is less well known is that eventually the atoms 
are no more made of matter, but are mathematical invariant in some number 
theoretical relations, which is my contribution.

Bruno




> 
> John K Clark
> 
> 
> -- 
> You received this message because you are subscribed to the Google Groups 
> "Everything List" group.
> To unsubscribe from this group and stop receiving emails from it, send an 
> email to everything-list+unsubscr...@googlegroups.com 
> .
> To view this discussion on the web visit 
> https://groups.google.com/d/msgid/everything-list/CAJPayv1xdGTxjAnooW9KR-dLzVJ02OC%3DCg3-y___L3pRSPd8xA%40mail.gmail.com
>  
> .

-- 
You received this message because you are subscribed to the Google Groups 
"Everything List" group.
To unsubscribe from this group and stop receiving emails from it, send an email 
to everything-list+unsubscr...@googlegroups.com.
To view this discussion on the web visit 
https://groups.google.com/d/msgid/everything-list/2494383E-5A32-42AA-A605-045028B898DB%40ulb.ac.be.


Re: Quantum immortality

2019-09-19 Thread Jason Resch
On Thu, Sep 19, 2019, 5:02 AM John Clark  wrote:

> On Thu, Sep 19, 2019 at 5:54 AM Jason Resch  wrote:
>
> *> This is exactly the break that occurred going from mind-brain identity
>> theory to multiple-realizability theories
>> (functionalism/computationalism/mechanism). It's the conventional meaning
>> implied by mechanism that there's no identity (one to one relationship)
>> between a mind and any particular one if it's instantiations.*
>>
>
> I've read that 3 times and I still can't figure out what it means.
>

Sorry I should have included references. This article explains it further:

https://en.m.wikipedia.org/wiki/Multiple_realizability

Jason




>  John K Clark
>
>
>>
>> --
> You received this message because you are subscribed to the Google Groups
> "Everything List" group.
> To unsubscribe from this group and stop receiving emails from it, send an
> email to everything-list+unsubscr...@googlegroups.com.
> To view this discussion on the web visit
> https://groups.google.com/d/msgid/everything-list/CAJPayv3oTnjg%3DF-6GzfPpBcByj2uMikVmCqkOXE4Qd0BOgK0mw%40mail.gmail.com
> 
> .
>

-- 
You received this message because you are subscribed to the Google Groups 
"Everything List" group.
To unsubscribe from this group and stop receiving emails from it, send an email 
to everything-list+unsubscr...@googlegroups.com.
To view this discussion on the web visit 
https://groups.google.com/d/msgid/everything-list/CA%2BBCJUgy1R-%2BuhCwCkeBFRxGqhScwg2LhxVBWqi5OzfwH5FbVw%40mail.gmail.com.


Re: Quantum immortality

2019-09-19 Thread John Clark
On Thu, Sep 19, 2019 at 5:54 AM Jason Resch  wrote:

*> This is exactly the break that occurred going from mind-brain identity
> theory to multiple-realizability theories
> (functionalism/computationalism/mechanism). It's the conventional meaning
> implied by mechanism that there's no identity (one to one relationship)
> between a mind and any particular one if it's instantiations.*
>

I've read that 3 times and I still can't figure out what it means.

 John K Clark


>
>

-- 
You received this message because you are subscribed to the Google Groups 
"Everything List" group.
To unsubscribe from this group and stop receiving emails from it, send an email 
to everything-list+unsubscr...@googlegroups.com.
To view this discussion on the web visit 
https://groups.google.com/d/msgid/everything-list/CAJPayv3oTnjg%3DF-6GzfPpBcByj2uMikVmCqkOXE4Qd0BOgK0mw%40mail.gmail.com.


Re: Quantum immortality

2019-09-19 Thread Jason Resch
On Mon, Sep 16, 2019, 7:18 AM John Clark  wrote:

> On Sun, Sep 15, 2019 at 8:34 AM Bruno Marchal  wrote:
>
> > *With mechanism* [...]
>
>  I thought I knew what "mechanism" mente today but quickly realized I was
> entirely wrong because immediately after those two words you added  "*you
> are not identical with your atoms configurations*".
>
> John K Clark
>

This is exactly the break that occurred going from mind-brain identity
theory to multiple-realizability theories
(functionalism/computationalism/mechanism). It's the conventional meaning
implied by mechanism that there's no identity (one to one relationship)
between a mind and any particular one if it's instantiations.

Jason

-- 
You received this message because you are subscribed to the Google Groups 
"Everything List" group.
To unsubscribe from this group and stop receiving emails from it, send an email 
to everything-list+unsubscr...@googlegroups.com.
To view this discussion on the web visit 
https://groups.google.com/d/msgid/everything-list/CA%2BBCJUgxnNEGMQEv6hf9S41kVn8KQudr_7Xfk%3D54H9M5_6KiMA%40mail.gmail.com.


Re: Quantum immortality

2019-09-18 Thread 'Brent Meeker' via Everything List




On 9/18/2019 1:07 AM, Bruce Kellett wrote:
That means that you certainly cannot become an earlier version of 
yourself, because that would not be a continuation of your current 
mental state. If you die, you might continue as a copy that was made 
at the instant of death, but nothing else could count as a 
continuation of your mental state.


Or looked at another way, once you're dead your continuation can be as a 
rock.  ;-)


Brent

--
You received this message because you are subscribed to the Google Groups 
"Everything List" group.
To unsubscribe from this group and stop receiving emails from it, send an email 
to everything-list+unsubscr...@googlegroups.com.
To view this discussion on the web visit 
https://groups.google.com/d/msgid/everything-list/0341018b-04d5-3128-f3ee-28da27bfe419%40verizon.net.


Re: Quantum immortality

2019-09-18 Thread Stathis Papaioannou
On Wed, 18 Sep 2019 at 18:07, Bruce Kellett  wrote:

> On Wed, Sep 18, 2019 at 4:06 PM Stathis Papaioannou 
> wrote:
>
>> On Wed, 18 Sep 2019 at 08:37, Bruce Kellett 
>> wrote:
>>
>>> On Tue, Sep 17, 2019 at 10:43 PM Telmo Menezes 
>>> wrote:
>>>
>>>> On Mon, Sep 16, 2019, at 22:56, Bruce Kellett wrote:
>>>>
>>>> On Tue, Sep 17, 2019 at 3:53 AM Bruno Marchal 
>>>> wrote:
>>>>
>>>> On 16 Sep 2019, at 05:51, 'Brent Meeker' via Everything List <
>>>> everything-list@googlegroups.com> wrote:
>>>>
>>>> On 9/15/2019 6:13 AM, Bruno Marchal wrote:
>>>>
>>>> And memory is fallible, and memory of age has no more meaning when your
>>>> age is bigger that the nameable or describable number, which happens very
>>>> soon, relatively, for the immortal being trying to keep track of their
>>>> birthday.
>>>>
>>>> Immortality is when you are to old to be able to even name your age.
>>>> After that, you have always the same age.
>>>>
>>>>
>>>> Nice aphorisms.  But irrelevant.  The question is why don't we see
>>>> almost everyone else as younger?
>>>>
>>>>
>>>> That happens when we are not old enough, but also, we might always
>>>> backtrack to younger people when close to death or when dying, …
>>>>
>>>>
>>>> What utter nonsense. You cannot jump between Everett branches, so you
>>>> cannot jump to a branch in which you were young.
>>>>
>>>> Depends on what you mean by "you".
>>>>
>>>
>>> I mean the person who has lived on this branch since birth. In every
>>> other branch in which copies exist, they occupy that space. So you can't
>>> simply "become" one of those copies on another branch -- what would happen
>>> to the one that was there? So you do not transfer memories or anything
>>> like. And you certainly can't become a younger copy of yourself.
>>>
>>
>> You can "jump" to any entity that is a continuation of your current
>> mental state, even if there is no causal connection. Having a persistent
>> human body that is in good shape is means of ensuring that such entities
>> exist. There is no actual "jumping", of course, even in ordinary life; it
>> is just an illusion.
>>
>
> That means that you certainly cannot become an earlier version of
> yourself, because that would not be a continuation of your current mental
> state. If you die, you might continue as a copy that was made at the
> instant of death, but nothing else could count as a continuation of your
> mental state.
>

Right, you can’t become an earlier version of yourself because if you could
you would live forever going backwards. The moments of your life are
ordered implicitly by their content, not by how, where or when they are
generated, except insofar as this determines content.

> --
Stathis Papaioannou

-- 
You received this message because you are subscribed to the Google Groups 
"Everything List" group.
To unsubscribe from this group and stop receiving emails from it, send an email 
to everything-list+unsubscr...@googlegroups.com.
To view this discussion on the web visit 
https://groups.google.com/d/msgid/everything-list/CAH%3D2ypW%2BdNghMUv70jpzCt98GLNFJsoa0DBM8SFUvaG-au9qJg%40mail.gmail.com.


Re: Quantum immortality

2019-09-18 Thread Bruce Kellett
On Wed, Sep 18, 2019 at 4:06 PM Stathis Papaioannou 
wrote:

> On Wed, 18 Sep 2019 at 08:37, Bruce Kellett  wrote:
>
>> On Tue, Sep 17, 2019 at 10:43 PM Telmo Menezes 
>> wrote:
>>
>>> On Mon, Sep 16, 2019, at 22:56, Bruce Kellett wrote:
>>>
>>> On Tue, Sep 17, 2019 at 3:53 AM Bruno Marchal  wrote:
>>>
>>> On 16 Sep 2019, at 05:51, 'Brent Meeker' via Everything List <
>>> everything-list@googlegroups.com> wrote:
>>>
>>> On 9/15/2019 6:13 AM, Bruno Marchal wrote:
>>>
>>> And memory is fallible, and memory of age has no more meaning when your
>>> age is bigger that the nameable or describable number, which happens very
>>> soon, relatively, for the immortal being trying to keep track of their
>>> birthday.
>>>
>>> Immortality is when you are to old to be able to even name your age.
>>> After that, you have always the same age.
>>>
>>>
>>> Nice aphorisms.  But irrelevant.  The question is why don't we see
>>> almost everyone else as younger?
>>>
>>>
>>> That happens when we are not old enough, but also, we might always
>>> backtrack to younger people when close to death or when dying, …
>>>
>>>
>>> What utter nonsense. You cannot jump between Everett branches, so you
>>> cannot jump to a branch in which you were young.
>>>
>>> Depends on what you mean by "you".
>>>
>>
>> I mean the person who has lived on this branch since birth. In every
>> other branch in which copies exist, they occupy that space. So you can't
>> simply "become" one of those copies on another branch -- what would happen
>> to the one that was there? So you do not transfer memories or anything
>> like. And you certainly can't become a younger copy of yourself.
>>
>
> You can "jump" to any entity that is a continuation of your current mental
> state, even if there is no causal connection. Having a persistent human
> body that is in good shape is means of ensuring that such entities exist.
> There is no actual "jumping", of course, even in ordinary life; it is just
> an illusion.
>

That means that you certainly cannot become an earlier version of yourself,
because that would not be a continuation of your current mental state. If
you die, you might continue as a copy that was made at the instant of
death, but nothing else could count as a continuation of your mental state.

Bruce

-- 
You received this message because you are subscribed to the Google Groups 
"Everything List" group.
To unsubscribe from this group and stop receiving emails from it, send an email 
to everything-list+unsubscr...@googlegroups.com.
To view this discussion on the web visit 
https://groups.google.com/d/msgid/everything-list/CAFxXSLRxoP4RNOMdCwJDomrke3%2BSX6hg%3DtThgt0Sm3wGL7%2BKzw%40mail.gmail.com.


Re: Quantum immortality

2019-09-17 Thread Stathis Papaioannou
On Wed, 18 Sep 2019 at 08:37, Bruce Kellett  wrote:

> On Tue, Sep 17, 2019 at 10:43 PM Telmo Menezes 
> wrote:
>
>> On Mon, Sep 16, 2019, at 22:56, Bruce Kellett wrote:
>>
>> On Tue, Sep 17, 2019 at 3:53 AM Bruno Marchal  wrote:
>>
>> On 16 Sep 2019, at 05:51, 'Brent Meeker' via Everything List <
>> everything-list@googlegroups.com> wrote:
>>
>> On 9/15/2019 6:13 AM, Bruno Marchal wrote:
>>
>> And memory is fallible, and memory of age has no more meaning when your
>> age is bigger that the nameable or describable number, which happens very
>> soon, relatively, for the immortal being trying to keep track of their
>> birthday.
>>
>> Immortality is when you are to old to be able to even name your age.
>> After that, you have always the same age.
>>
>>
>> Nice aphorisms.  But irrelevant.  The question is why don't we see almost
>> everyone else as younger?
>>
>>
>> That happens when we are not old enough, but also, we might always
>> backtrack to younger people when close to death or when dying, …
>>
>>
>> What utter nonsense. You cannot jump between Everett branches, so you
>> cannot jump to a branch in which you were young.
>>
>> Depends on what you mean by "you".
>>
>
> I mean the person who has lived on this branch since birth. In every other
> branch in which copies exist, they occupy that space. So you can't simply
> "become" one of those copies on another branch -- what would happen to the
> one that was there? So you do not transfer memories or anything like. And
> you certainly can't become a younger copy of yourself.
>

You can "jump" to any entity that is a continuation of your current mental
state, even if there is no causal connection. Having a persistent human
body that is in good shape is means of ensuring that such entities exist.
There is no actual "jumping", of course, even in ordinary life; it is just
an illusion.

-- 
Stathis Papaioannou

<https://www.avast.com/sig-email?utm_medium=email&utm_source=link&utm_campaign=sig-email&utm_content=webmail>
Virus-free.
www.avast.com
<https://www.avast.com/sig-email?utm_medium=email&utm_source=link&utm_campaign=sig-email&utm_content=webmail>
<#DAB4FAD8-2DD7-40BB-A1B8-4E2AA1F9FDF2>

-- 
You received this message because you are subscribed to the Google Groups 
"Everything List" group.
To unsubscribe from this group and stop receiving emails from it, send an email 
to everything-list+unsubscr...@googlegroups.com.
To view this discussion on the web visit 
https://groups.google.com/d/msgid/everything-list/CAH%3D2ypWG67rb9qvMgt7XV%3Dvv6Gy%3D-Nwb%2B9T5Jjnu_gcrP0zpFw%40mail.gmail.com.


Re: Quantum immortality

2019-09-17 Thread Russell Standish
On Wed, Sep 11, 2019 at 10:01:42PM +1000, Bruce Kellett wrote:
> On Wed, Sep 11, 2019 at 4:57 PM Quentin Anciaux  wrote:
> 
> The argument of the measure is based on ASSA and that's why it is flawed,
> moments are not random sampled from all possible moments, with this
> argument and without QI, you should have never find yourself young... But
> somewhere just before your death.
> 
> 
> ASSA is not a law of physics. I am not assuming random sampling from anything.
> It is just that you spend more time old than young given quantum immortality.
> That is not to say that you are never young -- of course you have to pass
> through all the years since your birth, one year at a time. It is just that
> there are more years after any given age than before that age.

Yes, and I'm still passing through those younger moments. That is the
RSSA position. If you want to say you are randomly sampled from all
your life's observer moments (ASSA), you have to take account the
nonuniform measure as a function of age over observer moments.


-- 


Dr Russell StandishPhone 0425 253119 (mobile)
Principal, High Performance Coders
Visiting Senior Research Fellowhpco...@hpcoders.com.au
Economics, Kingston University http://www.hpcoders.com.au


-- 
You received this message because you are subscribed to the Google Groups 
"Everything List" group.
To unsubscribe from this group and stop receiving emails from it, send an email 
to everything-list+unsubscr...@googlegroups.com.
To view this discussion on the web visit 
https://groups.google.com/d/msgid/everything-list/20190918050814.GE2417%40zen.


Re: Quantum immortality

2019-09-17 Thread 'Brent Meeker' via Everything List




On 9/17/2019 10:01 PM, Russell Standish wrote:

On Tue, Sep 10, 2019 at 05:18:51PM -0700, 'Brent Meeker' via Everything List 
wrote:


On 9/10/2019 4:30 PM, Bruce Kellett wrote:

Another argument that has been given here before is that if quantum
immortality is true, then we should expect to see a number of people who
are considerably older than the normal life expectancy -- and we do not
see people who are two or three hundred years old. Even if the
probabilities are very low, there have been an awful lot of people born
within the last 500 or so years -- some must have survived on our branch
if this scenario is true.

My argument was that each of us should find ourselves to be much older than
even the oldest people we know.

Only if the ASSA is true, not the RSSA. But even if the ASSA is true,
the total measure of very old observer moments may well be
insignificant compared with those of moderate age, so no, your
argument fails. The latter was my mistake in an argument I had with
Jacques Mallah once.


I don't understand what you mean by ASSA vs RSSA. Can you expand?

Whether your measure at ages below 100 outweighs the measure above 120 
would depend on what the branching is like.  That's why I was trying to 
pin down what the theory of branching is?  If every instant of your life 
has a successor, then every branch has a path to infinity. But if almost 
all branches reach death then only one or a few reach 200.  Bruno has in 
mind that your measure is made of "conscious moments" and "you" are just 
some set of them or all of them.


Brent







--
You received this message because you are subscribed to the Google Groups 
"Everything List" group.
To unsubscribe from this group and stop receiving emails from it, send an email 
to everything-list+unsubscr...@googlegroups.com.
To view this discussion on the web visit 
https://groups.google.com/d/msgid/everything-list/00e0671d-7133-adfc-d503-69c33551c1b2%40verizon.net.


Re: Quantum immortality

2019-09-17 Thread Russell Standish
On Wed, Sep 11, 2019 at 12:00:04PM +1000, Bruce Kellett wrote:
> On Wed, Sep 11, 2019 at 10:18 AM 'Brent Meeker' via Everything List <
> everything-list@googlegroups.com> wrote:
> 
> On 9/10/2019 4:30 PM, Bruce Kellett wrote:
> > Another argument that has been given here before is that if quantum
> > immortality is true, then we should expect to see a number of people
> > who are considerably older than the normal life expectancy -- and we
> > do not see people who are two or three hundred years old. Even if the
> > probabilities are very low, there have been an awful lot of people
> > born within the last 500 or so years -- some must have survived on our
> > branch if this scenario is true.
> 
> My argument was that each of us should find ourselves to be much older
> than even the oldest people we know.
> 
>  
> That is probably the best single argument against quantum immortality: if QI 
> is
> true, then the measure of our lifetime after one reaches a normal lifetime is
> infinitely greater than the measure before age , say, 120 yr. So if one finds
> oneself younger than 120 years, QI is false, and if MWI is still considered to
> be true, there must be another argument why MWI does not imply QI.
>

Once I tried to use this argument against the ASSA in a debate with
Jacques Mallah. I lost. This line of argument fails.


-- 


Dr Russell StandishPhone 0425 253119 (mobile)
Principal, High Performance Coders
Visiting Senior Research Fellowhpco...@hpcoders.com.au
Economics, Kingston University http://www.hpcoders.com.au


-- 
You received this message because you are subscribed to the Google Groups 
"Everything List" group.
To unsubscribe from this group and stop receiving emails from it, send an email 
to everything-list+unsubscr...@googlegroups.com.
To view this discussion on the web visit 
https://groups.google.com/d/msgid/everything-list/20190918050411.GD2417%40zen.


Re: Quantum immortality

2019-09-17 Thread Russell Standish
On Tue, Sep 10, 2019 at 05:18:51PM -0700, 'Brent Meeker' via Everything List 
wrote:
> 
> 
> On 9/10/2019 4:30 PM, Bruce Kellett wrote:
> > Another argument that has been given here before is that if quantum
> > immortality is true, then we should expect to see a number of people who
> > are considerably older than the normal life expectancy -- and we do not
> > see people who are two or three hundred years old. Even if the
> > probabilities are very low, there have been an awful lot of people born
> > within the last 500 or so years -- some must have survived on our branch
> > if this scenario is true.
> 
> My argument was that each of us should find ourselves to be much older than
> even the oldest people we know.

Only if the ASSA is true, not the RSSA. But even if the ASSA is true,
the total measure of very old observer moments may well be
insignificant compared with those of moderate age, so no, your
argument fails. The latter was my mistake in an argument I had with
Jacques Mallah once.


-- 


Dr Russell StandishPhone 0425 253119 (mobile)
Principal, High Performance Coders
Visiting Senior Research Fellowhpco...@hpcoders.com.au
Economics, Kingston University http://www.hpcoders.com.au


-- 
You received this message because you are subscribed to the Google Groups 
"Everything List" group.
To unsubscribe from this group and stop receiving emails from it, send an email 
to everything-list+unsubscr...@googlegroups.com.
To view this discussion on the web visit 
https://groups.google.com/d/msgid/everything-list/20190918050120.GC2417%40zen.


Re: Quantum immortality

2019-09-17 Thread Bruce Kellett
On Wed, Sep 18, 2019 at 10:25 AM 'Brent Meeker' via Everything List <
everything-list@googlegroups.com> wrote:

> On 9/17/2019 3:36 PM, Bruce Kellett wrote:
>
> On Tue, Sep 17, 2019 at 10:43 PM Telmo Menezes 
> wrote:
>
>> On Mon, Sep 16, 2019, at 22:56, Bruce Kellett wrote:
>>
>> On Tue, Sep 17, 2019 at 3:53 AM Bruno Marchal  wrote:
>>
>> On 16 Sep 2019, at 05:51, 'Brent Meeker' via Everything List <
>> everything-list@googlegroups.com> wrote:
>>
>> On 9/15/2019 6:13 AM, Bruno Marchal wrote:
>>
>> And memory is fallible, and memory of age has no more meaning when your
>> age is bigger that the nameable or describable number, which happens very
>> soon, relatively, for the immortal being trying to keep track of their
>> birthday.
>>
>> Immortality is when you are to old to be able to even name your age.
>> After that, you have always the same age.
>>
>>
>> Nice aphorisms.  But irrelevant.  The question is why don't we see almost
>> everyone else as younger?
>>
>>
>> That happens when we are not old enough, but also, we might always
>> backtrack to younger people when close to death or when dying, …
>>
>>
>> What utter nonsense. You cannot jump between Everett branches, so you
>> cannot jump to a branch in which you were young.
>>
>> Depends on what you mean by "you".
>>
>
> I mean the person who has lived on this branch since birth. In every other
> branch in which copies exist, they occupy that space. So you can't simply
> "become" one of those copies on another branch -- what would happen to the
> one that was there? So you do not transfer memories or anything like. And
> you certainly can't become a younger copy of yourself.
>
>
> I think it's what Bruno refers to as "indexial". He just stops pointing to
> one branch saying "me" and starts pointing to a surviving branch and says
> "me".
>

And that is supposed to mean something?

Bruce

-- 
You received this message because you are subscribed to the Google Groups 
"Everything List" group.
To unsubscribe from this group and stop receiving emails from it, send an email 
to everything-list+unsubscr...@googlegroups.com.
To view this discussion on the web visit 
https://groups.google.com/d/msgid/everything-list/CAFxXSLQeg972p6WJhzW9YWJwmTshxgHci6z8pJxSiOLZi9v76g%40mail.gmail.com.


Re: Quantum immortality

2019-09-17 Thread 'Brent Meeker' via Everything List



On 9/17/2019 3:36 PM, Bruce Kellett wrote:
On Tue, Sep 17, 2019 at 10:43 PM Telmo Menezes <mailto:te...@telmomenezes.net>> wrote:


On Mon, Sep 16, 2019, at 22:56, Bruce Kellett wrote:

On Tue, Sep 17, 2019 at 3:53 AM Bruno Marchal mailto:marc...@ulb.ac.be>> wrote:

On 16 Sep 2019, at 05:51, 'Brent Meeker' via Everything List
mailto:everything-list@googlegroups.com>> wrote:

On 9/15/2019 6:13 AM, Bruno Marchal wrote:

And memory is fallible, and memory of age has no more
meaning when your age is bigger that the nameable or
describable number, which happens very soon, relatively,
for the immortal being trying to keep track of their birthday.

Immortality is when you are to old to be able to even name
your age. After that, you have always the same age.


Nice aphorisms.  But irrelevant. The question is why don't
we see almost everyone else as younger?


That happens when we are not old enough, but also, we might
always backtrack to younger people when close to death or
when dying, …


What utter nonsense. You cannot jump between Everett branches, so
you cannot jump to a branch in which you were young.


Depends on what you mean by "you".


I mean the person who has lived on this branch since birth. In every 
other branch in which copies exist, they occupy that space. So you 
can't simply "become" one of those copies on another branch -- what 
would happen to the one that was there? So you do not transfer 
memories or anything like. And you certainly can't become a younger 
copy of yourself.


I think it's what Bruno refers to as "indexial". He just stops pointing 
to one branch saying "me" and starts pointing to a surviving branch and 
says "me".


Brent


Physics rules out backward causation and branch jumping.


Physics is inconsistent with itself and fails to explain
significant aspects of observable nature, so it doesn't really
matter what it "rules out". We don't know enough for such bold claims.


Physics is not inconsistent with itself or else existence would be 
impossible! Some physical theories may be marginally inconsistent with 
others, but that is another matter. It does not mean that there is 
open slather for you to believe whatever you want, especially if that 
is inconsistent with well-established theories.


Bruce
--
You received this message because you are subscribed to the Google 
Groups "Everything List" group.
To unsubscribe from this group and stop receiving emails from it, send 
an email to everything-list+unsubscr...@googlegroups.com 
<mailto:everything-list+unsubscr...@googlegroups.com>.
To view this discussion on the web visit 
https://groups.google.com/d/msgid/everything-list/CAFxXSLQ12tqbBW6NXiaF9x4HkgTCT92MJ6K70McKMp03VQ6cRw%40mail.gmail.com 
<https://groups.google.com/d/msgid/everything-list/CAFxXSLQ12tqbBW6NXiaF9x4HkgTCT92MJ6K70McKMp03VQ6cRw%40mail.gmail.com?utm_medium=email&utm_source=footer>.


--
You received this message because you are subscribed to the Google Groups 
"Everything List" group.
To unsubscribe from this group and stop receiving emails from it, send an email 
to everything-list+unsubscr...@googlegroups.com.
To view this discussion on the web visit 
https://groups.google.com/d/msgid/everything-list/794615e9-4f41-9a84-8c3a-60734c66b8e6%40verizon.net.


Re: Quantum immortality

2019-09-17 Thread Bruce Kellett
On Tue, Sep 17, 2019 at 10:43 PM Telmo Menezes 
wrote:

> On Mon, Sep 16, 2019, at 22:56, Bruce Kellett wrote:
>
> On Tue, Sep 17, 2019 at 3:53 AM Bruno Marchal  wrote:
>
> On 16 Sep 2019, at 05:51, 'Brent Meeker' via Everything List <
> everything-list@googlegroups.com> wrote:
>
> On 9/15/2019 6:13 AM, Bruno Marchal wrote:
>
> And memory is fallible, and memory of age has no more meaning when your
> age is bigger that the nameable or describable number, which happens very
> soon, relatively, for the immortal being trying to keep track of their
> birthday.
>
> Immortality is when you are to old to be able to even name your age. After
> that, you have always the same age.
>
>
> Nice aphorisms.  But irrelevant.  The question is why don't we see almost
> everyone else as younger?
>
>
> That happens when we are not old enough, but also, we might always
> backtrack to younger people when close to death or when dying, …
>
>
> What utter nonsense. You cannot jump between Everett branches, so you
> cannot jump to a branch in which you were young.
>
> Depends on what you mean by "you".
>

I mean the person who has lived on this branch since birth. In every other
branch in which copies exist, they occupy that space. So you can't simply
"become" one of those copies on another branch -- what would happen to the
one that was there? So you do not transfer memories or anything like. And
you certainly can't become a younger copy of yourself.


> Physics rules out backward causation and branch jumping.
>
>
> Physics is inconsistent with itself and fails to explain significant
> aspects of observable nature, so it doesn't really matter what it "rules
> out". We don't know enough for such bold claims.
>

Physics is not inconsistent with itself or else existence would be
impossible! Some physical theories may be marginally inconsistent with
others, but that is another matter. It does not mean that there is open
slather for you to believe whatever you want, especially if that is
inconsistent with well-established theories.

Bruce

-- 
You received this message because you are subscribed to the Google Groups 
"Everything List" group.
To unsubscribe from this group and stop receiving emails from it, send an email 
to everything-list+unsubscr...@googlegroups.com.
To view this discussion on the web visit 
https://groups.google.com/d/msgid/everything-list/CAFxXSLQ12tqbBW6NXiaF9x4HkgTCT92MJ6K70McKMp03VQ6cRw%40mail.gmail.com.


Re: Quantum immortality

2019-09-17 Thread Telmo Menezes


On Mon, Sep 16, 2019, at 22:56, Bruce Kellett wrote:
> On Tue, Sep 17, 2019 at 3:53 AM Bruno Marchal  wrote:
>> On 16 Sep 2019, at 05:51, 'Brent Meeker' via Everything List 
>>  wrote:
>>> On 9/15/2019 6:13 AM, Bruno Marchal wrote:
>>>> And memory is fallible, and memory of age has no more meaning when your 
>>>> age is bigger that the nameable or describable number, which happens very 
>>>> soon, relatively, for the immortal being trying to keep track of their 
>>>> birthday. 
>>>> 
>>>> Immortality is when you are to old to be able to even name your age. After 
>>>> that, you have always the same age.
>>> 
>>> Nice aphorisms. But irrelevant. The question is why don't we see almost 
>>> everyone else as younger? 
>> 
>> That happens when we are not old enough, but also, we might always backtrack 
>> to younger people when close to death or when dying, …
> 
> What utter nonsense. You cannot jump between Everett branches, so you cannot 
> jump to a branch in which you were young.
> 

Depends on what you mean by "you".

> Physics rules out backward causation and branch jumping.

Physics is inconsistent with itself and fails to explain significant aspects of 
observable nature, so it doesn't really matter what it "rules out". We don't 
know enough for such bold claims.

Telmo.

> 
>> … may be up to something like this video below, which is an oversimplifying 
>> view (mixing G and G* all the time) of Neoplatonism, or theology close to 
>> Mechanism:
>> 
>> https://www.youtube.com/watch?v=h6fcK_fRYaI
>> 
>> You can get very old, and always believed that you are young, even a baby, 
>> and be right on that, just by not memorising everything.
> 
> Losing you mind when you are very old is quite common, but that does not mean 
> that you become young again.
> 
> Bruce
> 
> 
>> Technological immortality is a sort of egotic complacency in the Samsara, 
>> and a sort of Procrastination of Nirvana. But why not? There is something to 
>> contemplate here, but here is only an aspect of a bigger and simpler 
>> reality. And there is something to contemplate there too. With Mechanism, 
>> mathematics can give a glimpse, and evacuate some fake certainties. Nature 
>> also used some authoritative argument sometimes...
>> 
>> Bruno
> 

> --
>  You received this message because you are subscribed to the Google Groups 
> "Everything List" group.
>  To unsubscribe from this group and stop receiving emails from it, send an 
> email to everything-list+unsubscr...@googlegroups.com.
>  To view this discussion on the web visit 
> https://groups.google.com/d/msgid/everything-list/CAFxXSLSy%3DtB8XgPieA0bEfyRX%3Dz1aXvYyWUp1e_DdhMWmQ0p-Q%40mail.gmail.com
>  
> <https://groups.google.com/d/msgid/everything-list/CAFxXSLSy%3DtB8XgPieA0bEfyRX%3Dz1aXvYyWUp1e_DdhMWmQ0p-Q%40mail.gmail.com?utm_medium=email&utm_source=footer>.

-- 
You received this message because you are subscribed to the Google Groups 
"Everything List" group.
To unsubscribe from this group and stop receiving emails from it, send an email 
to everything-list+unsubscr...@googlegroups.com.
To view this discussion on the web visit 
https://groups.google.com/d/msgid/everything-list/f6d72d12-d07f-4a1a-9fc7-b3d41c993b22%40www.fastmail.com.


Re: Quantum immortality

2019-09-16 Thread Bruce Kellett
On Tue, Sep 17, 2019 at 3:53 AM Bruno Marchal  wrote:

> On 16 Sep 2019, at 05:51, 'Brent Meeker' via Everything List <
> everything-list@googlegroups.com> wrote:
>
> On 9/15/2019 6:13 AM, Bruno Marchal wrote:
>
> And memory is fallible, and memory of age has no more meaning when your
> age is bigger that the nameable or describable number, which happens very
> soon, relatively, for the immortal being trying to keep track of their
> birthday.
>
> Immortality is when you are to old to be able to even name your age. After
> that, you have always the same age.
>
>
> Nice aphorisms.  But irrelevant.  The question is why don't we see almost
> everyone else as younger?
>
>
> That happens when we are not old enough, but also, we might always
> backtrack to younger people when close to death or when dying, …
>

What utter nonsense. You cannot jump between Everett branches, so you
cannot jump to a branch in which you were young. Physics rules out backward
causation and branch jumping.


> … may be up to something like this video below, which is an
> oversimplifying view (mixing G and G* all the time) of Neoplatonism, or
> theology close to Mechanism:
>
> https://www.youtube.com/watch?v=h6fcK_fRYaI
>
> You can get very old, and always believed that you are young, even a baby,
> and be right on that, just by not memorising everything.
>

Losing you mind when you are very old is quite common, but that does not
mean that you become young again.

Bruce


Technological immortality is a sort of egotic complacency in the Samsara,
> and a sort of Procrastination of Nirvana. But why not? There is something
> to contemplate here, but here is only an aspect of a bigger and simpler
> reality. And there is something to contemplate there too. With Mechanism,
> mathematics can give a glimpse, and evacuate some fake certainties. Nature
> also used some authoritative argument sometimes...
>
> Bruno
>

-- 
You received this message because you are subscribed to the Google Groups 
"Everything List" group.
To unsubscribe from this group and stop receiving emails from it, send an email 
to everything-list+unsubscr...@googlegroups.com.
To view this discussion on the web visit 
https://groups.google.com/d/msgid/everything-list/CAFxXSLSy%3DtB8XgPieA0bEfyRX%3Dz1aXvYyWUp1e_DdhMWmQ0p-Q%40mail.gmail.com.


Re: Quantum immortality

2019-09-16 Thread Bruno Marchal

> On 16 Sep 2019, at 05:58, 'Brent Meeker' via Everything List 
>  wrote:
> 
> 
> 
> On 9/15/2019 6:41 AM, Bruno Marchal wrote:
 If in H you are multiplied in W and M, but directly killed in M, you 
 survive in W with probability one. That is why we add p or <>t to []p to 
 transform the logic of belief ([]p) into a probability logic ([]p & <>t).
>>> Suppose you live a few seconds in M.  Do you then survive in W with 
>>> probability 0.5?
>> Assuming you do die in M, even after some years, the probability in H to be 
>> feeling the one in W will be one, assuming you never dies in W. But this 
>> assumes mortality, and some transitivity of the probability rules, so the  
>> question is very complex. The probability in H to be W or M, for a short 
>> time,  is one half, but the probability to be in the place where you stay 
>> for a long time, will be close to one in a sort of retrospective way. 
>> 
>> All this comes from a simple fact: absolute-death is not a first person 
>> experience. There is no entry in the first person diary which mention “I 
>> died today”.
>> 
>> The difficulty is that the first person renormalise the probabilities all 
>> the time, and that is why making them transitive leads to paradoxes.
> 
> I think what makes them paradoxical is that you jump around between 
> subjective probabilities of different persons beliefs.

It works because I take them all, in the relative computationalist way (which 
is amenable to the mathematics of the diverse form of self-reference).

It is the materialist which invokes special selector making them unique. A bit 
like those who invoke the wave packet reduction in QM.




> 
>> 
>> Let me try to illustrate. You are in H, just before the WM-duplication. You 
>> are told in advance that in W you will get a cup of tea, and then be killed. 
>> In W you get a cup of coffee, and not killed. 
> 
> Is that last W supposed to be "M”?

Right.

Bruno



> 
>> What is the probability (in H) that you will get a cup of tea. It is 1/2. 
>> But what is the probability, in H, that you will have a long lasting memory 
>> of having drink a cup of coffee after that experiments: It is 1. In fact, in 
>> Moscow, you could (although it is psychologically very difficult) still bet 
>> that “you” will have a memory of having doing coffee, and just an amnesia of 
>> M and its cup of tea. This also gives some sense that we survive more in our 
>> kids and in the value we transmit to them, than in bodies and personal first 
>> person happening. 
>> 
>> Now, that renormalisation process is not easy, a bit like in QFT, we get 
>> infinities which are hard to subtract. 
> 
> Brent
> 
> -- 
> You received this message because you are subscribed to the Google Groups 
> "Everything List" group.
> To unsubscribe from this group and stop receiving emails from it, send an 
> email to everything-list+unsubscr...@googlegroups.com 
> .
> To view this discussion on the web visit 
> https://groups.google.com/d/msgid/everything-list/d673b090-1a42-fc42-4abc-68fc09633e9e%40verizon.net
>  
> .

-- 
You received this message because you are subscribed to the Google Groups 
"Everything List" group.
To unsubscribe from this group and stop receiving emails from it, send an email 
to everything-list+unsubscr...@googlegroups.com.
To view this discussion on the web visit 
https://groups.google.com/d/msgid/everything-list/76E6853D-7661-4100-9136-24BF1E92DA26%40ulb.ac.be.


Re: Quantum immortality

2019-09-16 Thread Bruno Marchal

> On 16 Sep 2019, at 05:51, 'Brent Meeker' via Everything List 
>  wrote:
> 
> 
> 
> On 9/15/2019 6:13 AM, Bruno Marchal wrote:
>> 
>>> On 13 Sep 2019, at 22:28, 'Brent Meeker' via Everything List 
>>> >> <mailto:everything-list@googlegroups.com>> wrote:
>>> 
>>> 
>>> 
>>> On 9/13/2019 10:59 AM, Jason Resch wrote:
>>>> 
>>>> 
>>>> On Wed, Sep 11, 2019 at 6:38 PM Bruce Kellett >>> <mailto:bhkellet...@gmail.com>> wrote:
>>>> On Thu, Sep 12, 2019 at 2:55 AM Jason Resch >>> <mailto:jasonre...@gmail.com>> wrote:
>>>> On Tuesday, September 10, 2019, Bruce Kellett >>> <mailto:bhkellet...@gmail.com>> wrote:
>>>> On Wed, Sep 11, 2019 at 10:18 AM 'Brent Meeker' via Everything List 
>>>> >>> <mailto:everything-list@googlegroups.com>> wrote:
>>>> On 9/10/2019 4:30 PM, Bruce Kellett wrote:
>>>> > Another argument that has been given here before is that if quantum 
>>>> > immortality is true, then we should expect to see a number of people 
>>>> > who are considerably older than the normal life expectancy -- and we 
>>>> > do not see people who are two or three hundred years old. Even if the 
>>>> > probabilities are very low, there have been an awful lot of people 
>>>> > born within the last 500 or so years -- some must have survived on our 
>>>> > branch if this scenario is true.
>>>> 
>>>> My argument was that each of us should find ourselves to be much older 
>>>> than even the oldest people we know.
>>>>  
>>>> That is probably the best single argument against quantum immortality: if 
>>>> QI is true, then the measure of our lifetime after one reaches a normal 
>>>> lifetime is infinitely greater than the measure before age , say, 120 yr. 
>>>> So if one finds oneself younger than 120 years, QI is false, and if MWI is 
>>>> still considered to be true, there must be another argument why MWI does 
>>>> not imply QI.
>>>> 
>>>> 
>>>> Why do you think that measure only increases with age? On an objective 
>>>> level it only decreases.
>>>> 
>>>> As Bruno would say, "you confuse the 1p with the 1pp." I am talking about 
>>>> my personal measure of the number of years I have lived. As I get older, 
>>>> the number of years I have lived increases. If I live to 1000, I have 
>>>> lived more years between 100 and 1000 than between 1 and 100. This is 
>>>> arithmetic, after all.
>>>> 
>>>> I see.  This reasoning works only under the assumption that finding 
>>>> yourself in any particular year across your infinite lifespan is 
>>>> equiprobable (i.e. you can ignore the effects of the number or measure of 
>>>> the various yous in other branches).  This is what I thought you mean by 
>>>> measure, in terms of how to calculate probabilities / weights of the 
>>>> various branches.
>>>>  
>>>> 
>>>> But this discussion has gone off the rails. It started as a discussion of 
>>>> quantum immortality, and the arguments against this notion, even in MWI. 
>>>> The arguments against QI that have been advanced are that life-threatening 
>>>> events tend not to be binary or quantum, but rather we enter a period of 
>>>> slow decline, due to illness or other factors. Consequently, there is no 
>>>> reason for us to expect to be immortal, even in MWI.
>>>> 
>>>> I don't see how that last sentence follows.  It is true MWI doesn't 
>>>> guarantee we should expect to always survive in the same condition, but it 
>>>> does guarantee we should survive in some form.
>>> 
>>> But what does "we" refer to. Are you saying Jason, with the memories he has 
>>> at this moment, will always have a successor in the future.   Or are you 
>>> saying there'll always be a Jason that shares my childhood memories or my 
>>> memories of last year when that lightning bolt just missed me.
>>> 
>>>>  
>>>> The other argument is that if QI is true, then you would expect to be very 
>>>> old.
>>>> 
>>>> We only know we are very old if our memories accumulate without limit, but 
>>>> MWI does not guarantee persistence of memory.  It also follows from thi

Re: Quantum immortality

2019-09-16 Thread Alan Grayson


On Monday, September 16, 2019 at 6:18:29 AM UTC-6, John Clark wrote:
>
> On Sun, Sep 15, 2019 at 8:34 AM Bruno Marchal  > wrote:
>
> > *With mechanism* [...]
>
>
> Bruno, I really wish you wouldn't start long paragraphs with those two 
> words because when you do I don't know if I agree with you or not. Please 
> be more specific and spell out exactly what assumptions you're starting 
> from, your all purpose word "mechanism" just isn't good enough. I thought I 
> knew what "mechanism" mente today but quickly realized I was entirely wrong 
> because immediately after those two words you added  "*you are not 
> identical with your atoms configurations*".
>
> John K Clark
>

By "mechanism" I think he means that a human brain and/or nervous system 
can be replaced by digital circuits, and can reproduce consciousness, if 
computability and the natural numbers are assumed. Agreeing to have your 
brain and/or nervous system replaced is the "Yes doctor" in this scenario. 
AG

-- 
You received this message because you are subscribed to the Google Groups 
"Everything List" group.
To unsubscribe from this group and stop receiving emails from it, send an email 
to everything-list+unsubscr...@googlegroups.com.
To view this discussion on the web visit 
https://groups.google.com/d/msgid/everything-list/aceb658d-3448-458b-bc2c-80cafd53ef79%40googlegroups.com.


Re: Quantum immortality

2019-09-16 Thread John Clark
On Sun, Sep 15, 2019 at 8:34 AM Bruno Marchal  wrote:

> *With mechanism* [...]


Bruno, I really wish you wouldn't start long paragraphs with those two
words because when you do I don't know if I agree with you or not. Please
be more specific and spell out exactly what assumptions you're starting
from, your all purpose word "mechanism" just isn't good enough. I thought I
knew what "mechanism" mente today but quickly realized I was entirely wrong
because immediately after those two words you added  "*you are not
identical with your atoms configurations*".

John K Clark

-- 
You received this message because you are subscribed to the Google Groups 
"Everything List" group.
To unsubscribe from this group and stop receiving emails from it, send an email 
to everything-list+unsubscr...@googlegroups.com.
To view this discussion on the web visit 
https://groups.google.com/d/msgid/everything-list/CAJPayv1xdGTxjAnooW9KR-dLzVJ02OC%3DCg3-y___L3pRSPd8xA%40mail.gmail.com.


Re: Quantum immortality

2019-09-15 Thread 'Brent Meeker' via Everything List



On 9/15/2019 6:41 AM, Bruno Marchal wrote:

If in H you are multiplied in W and M, but directly killed in M, you survive in W with 
probability one. That is why we add p or <>t to []p to transform the logic of belief 
([]p) into a probability logic ([]p & <>t).

Suppose you live a few seconds in M.  Do you then survive in W with probability 
0.5?

Assuming you do die in M, even after some years, the probability in H to be 
feeling the one in W will be one, assuming you never dies in W. But this 
assumes mortality, and some transitivity of the probability rules, so the  
question is very complex. The probability in H to be W or M, for a short time,  
is one half, but the probability to be in the place where you stay for a long 
time, will be close to one in a sort of retrospective way.

All this comes from a simple fact: absolute-death is not a first person 
experience. There is no entry in the first person diary which mention “I died 
today”.

The difficulty is that the first person renormalise the probabilities all the 
time, and that is why making them transitive leads to paradoxes.


I think what makes them paradoxical is that you jump around between 
subjective probabilities of different persons beliefs.




Let me try to illustrate. You are in H, just before the WM-duplication. You are 
told in advance that in W you will get a cup of tea, and then be killed. In W 
you get a cup of coffee, and not killed.


Is that last W supposed to be "M"?


What is the probability (in H) that you will get a cup of tea. It is 1/2. But 
what is the probability, in H, that you will have a long lasting memory of 
having drink a cup of coffee after that experiments: It is 1. In fact, in 
Moscow, you could (although it is psychologically very difficult) still bet 
that “you” will have a memory of having doing coffee, and just an amnesia of M 
and its cup of tea. This also gives some sense that we survive more in our kids 
and in the value we transmit to them, than in bodies and personal first person 
happening.

Now, that renormalisation process is not easy, a bit like in QFT, we get 
infinities which are hard to subtract.


Brent

--
You received this message because you are subscribed to the Google Groups 
"Everything List" group.
To unsubscribe from this group and stop receiving emails from it, send an email 
to everything-list+unsubscr...@googlegroups.com.
To view this discussion on the web visit 
https://groups.google.com/d/msgid/everything-list/d673b090-1a42-fc42-4abc-68fc09633e9e%40verizon.net.


Re: Quantum immortality

2019-09-15 Thread 'Brent Meeker' via Everything List



On 9/15/2019 6:13 AM, Bruno Marchal wrote:


On 13 Sep 2019, at 22:28, 'Brent Meeker' via Everything List 
<mailto:everything-list@googlegroups.com>> wrote:




On 9/13/2019 10:59 AM, Jason Resch wrote:



On Wed, Sep 11, 2019 at 6:38 PM Bruce Kellett <mailto:bhkellet...@gmail.com>> wrote:


On Thu, Sep 12, 2019 at 2:55 AM Jason Resch
mailto:jasonre...@gmail.com>> wrote:

On Tuesday, September 10, 2019, Bruce Kellett
mailto:bhkellet...@gmail.com>> wrote:

On Wed, Sep 11, 2019 at 10:18 AM 'Brent Meeker' via
Everything List mailto:everything-list@googlegroups.com>> wrote:

On 9/10/2019 4:30 PM, Bruce Kellett wrote:
> Another argument that has been given here before
    is that if quantum
> immortality is true, then we should expect to see
a number of people
> who are considerably older than the normal life
expectancy -- and we
> do not see people who are two or three hundred
years old. Even if the
> probabilities are very low, there have been an
awful lot of people
> born within the last 500 or so years -- some must
have survived on our
> branch if this scenario is true.

My argument was that each of us should find
ourselves to be much older
than even the oldest people we know.

    That is probably the best single argument against
quantum immortality: if QI is true, then the measure of
our lifetime after one reaches a normal lifetime is
infinitely greater than the measure before age , say,
120 yr. So if one finds oneself younger than 120 years,
QI is false, and if MWI is still considered to be true,
there must be another argument why MWI does not imply QI.



Why do you think that measure only increases with age? On an
objective level it only decreases.


As Bruno would say, "you confuse the 1p with the 1pp." I am
talking about my personal measure of the number of years I have
lived. As I get older, the number of years I have lived
increases. If I live to 1000, I have lived more years between
100 and 1000 than between 1 and 100. This is arithmetic, after all.


I see.  This reasoning works only under the assumption that finding 
yourself in any particular year across your infinite lifespan is 
equiprobable (i.e. you can ignore the effects of the number or 
measure of the various yous in other branches).  This is what I 
thought you mean by measure, in terms of how to calculate 
probabilities / weights of the various branches.



    But this discussion has gone off the rails. It started as a
discussion of quantum immortality, and the arguments against
this notion, even in MWI. The arguments against QI that have
been advanced are that life-threatening events tend not to be
binary or quantum, but rather we enter a period of slow decline,
due to illness or other factors. Consequently, there is no
reason for us to expect to be immortal, even in MWI.


I don't see how that last sentence follows.  It is true MWI doesn't 
guarantee we should expect to always survive in the same condition, 
but it does guarantee we should survive in some form.


But what does "we" refer to. Are you saying Jason, with the memories 
he has at this moment, will always have a successor in the future.   
Or are you saying there'll always be a Jason that shares my childhood 
memories or my memories of last year when that lightning bolt just 
missed me.



The other argument is that if QI is true, then you would expect
to be very old.


We only know we are very old if our memories accumulate without 
limit, but MWI does not guarantee persistence of memory.  It also 
follows from this that to know one is immortal (has lived an 
infinite number of years) requires an infinitely large brain and 
memory capacity.


I don't have to remember everything that happened over 80yrs to know 
I'm 80yrs old.  In fact I only need to remember my birthday.



And memory is fallible, and memory of age has no more meaning when 
your age is bigger that the nameable or describable number, which 
happens very soon, relatively, for the immortal being trying to keep 
track of their birthday.


Immortality is when you are to old to be able to even name your age. 
After that, you have always the same age.


Nice aphorisms.  But irrelevant.  The question is why don't we see 
almost everyone else as younger?




Mortality is an illusion enjoyed by the gods when tired of eternity. 
It is very long 


Brent
"Eternity is very long.  Especially near the end."
        ---

Re: Quantum immortality

2019-09-15 Thread 'Brent Meeker' via Everything List



On 9/15/2019 5:54 AM, Bruno Marchal wrote:


On 13 Sep 2019, at 22:17, 'Brent Meeker' via Everything List 
<mailto:everything-list@googlegroups.com>> wrote:




On 9/13/2019 4:18 AM, Quentin Anciaux wrote:



Le ven. 13 sept. 2019 à 13:16, Bruce Kellett <mailto:bhkellet...@gmail.com>> a écrit :


On Fri, Sep 13, 2019 at 8:49 PM Bruno Marchal mailto:marc...@ulb.ac.be>> wrote:

On 12 Sep 2019, at 01:50, Bruce Kellett
mailto:bhkellet...@gmail.com>> wrote:

On Thu, Sep 12, 2019 at 1:55 AM Bruno Marchal
mailto:marc...@ulb.ac.be>> wrote:

On 11 Sep 2019, at 01:30, Bruce Kellett
mailto:bhkell...@optusnet.com.au>> wrote:

From: *Bruno Marchal* mailto:marc...@ulb.ac.be>>

On 8 Sep 2019, at 13:59, Bruce Kellett
mailto:bhkellet...@gmail.com>> wrote:

If the only relevance you can find for many worlds
is quantum immortality, then many worlds is indeed
dead. Quantum immortality has been shown many times
to be a complete nonsense.


Really. I did not known that. Could you give the
references.


Follow the Wikipedia entry on quantum suicide.


That is not what I mean by a  reference.


I later gave a reference to the paper by Mallah -- whom you
know of, apparently. The paper is available at

https://arxiv.org/abs/0902.0187





Yes, the oldest participant in this list have know Jacques
Mallah, who participated a lot in this list.

Mallah is wrong here:

<<
Max Tegmark publicized the QS idea, but in some ways he is
more of a moderate on the issue than most of its believers
are. If he were to follow in the footsteps of Don Page
and alter his views, recanting belief in QS, it would be a
great help in exposing the belief as a fallacy, and I hold
out hope that it is possible that he will do so.

In his paper [Tegmark 1] QS is explained as follows:

“Since there is exactly one observer having perceptions both
before and after the trigger event, and since it occurred
too fast to notice, the MWI prediction is that” (the
experimenter) “will hear “click” with 100% certainty.”

That is a rather odd statement because he is certainly aware
that in the MWI there is no sense in which it can be
rightfully said that “there is exactly one observer” either
before


or after the experiment. The ket notation may be unhelpful
here; indeed, if the tensor product of kets on the left hand
side were expanded instead of factoring out the
observer, there would appear to have been “two observers”
initially.
>>


I don't get Mallah's point here, either. I will have to look
more clearly at his argument against QS. I don't think that case
is a clear-cut as for QI. The fact that I am not the oldest
person around is clear evidence against QI.


It's wrong, that imply you can nerver have been young.


And I am young, therefore quantum immortality is wrong. But exactly 
where is it wrong. There seem to be two different ideas of quantum 
immortality.  In one verison, the everything-happens version, is that 
whatever your state there is a physically possible way for you to 
survive...like invoking Bruno's magic cosmic rays that just happen to 
trigger the right nerves for the brain damaged student to ace her test.



Just to be clear, nobody believes in such magic cosmic rays. They have 
a probability zero in all histories, and I used them only to 
illustrate a point (indeed, they are replaced by the movie projection 
at the following step).


The computationalist immortality does not rely on such magic. To be sure.




In this version, no matter your age or circumstance, there will be a 
'you' that remembers your age and circumstance indefinitely far into 
the future.


The other version says that almost all Everettian 'copies' of your 
future will die but there's a non-zero probability of one still 
existing at any future time.  So then the relative measure of your 
future self depends on the ratio of copies that haven't died to those 
that have.  This implicitly assumes that whatever event that causes 
you to die does not also cause a surviving copy to be created (i.e. 
it's not a Tegmark machine gun).


I don’t see the difference. In all cases, when old and sick, surviving 
is like a white rabbit. The immortality comes only from the fact that 
no matter how the probability of surviving is small, from “your" first 
person view, death is not an experience, and “you” are always there.


But that's the probability of you surviving conditional on you knowing 
it.  That doesn&#

Re: Quantum immortality

2019-09-15 Thread Bruno Marchal


> On 13 Sep 2019, at 22:57, 'Brent Meeker' via Everything List 
>  wrote:
> 
> 
> 
> On 9/13/2019 3:38 AM, Bruno Marchal wrote:
>>> On 11 Sep 2019, at 16:51, smitra  wrote:
>>> 
>>> Back to basics. There exists a universal wavefunction that evolves 
>>> according to the Schrodinger equation. Observers are internal structures in 
>>> this description. Whether or not one believes that the Born rule can be 
>>> derived or not, what matters in practice is that you'll end up having to 
>>> use it, so you have to assign a measure for observations that is given by 
>>> the summation of the squared modulus of the states that correspond to those 
>>> observations. The information about personal identity must then also be 
>>> extracted from the wavefunction, so one cannot insert this in an ad hoc way.
>>> 
>>> Quantum immortality is therefore wrong because the measure of the states 
>>> that correspond to extremely old observers is small.
>> 
>> The same reasoning would apply to “quantum suicide”, where it is clear that 
>> we survive all the time; given that we cannot take into account the world 
>> where we do not.
>> 
>> If in H you are multiplied in W and M, but directly killed in M, you survive 
>> in W with probability one. That is why we add p or <>t to []p to transform 
>> the logic of belief ([]p) into a probability logic ([]p & <>t).
> 
> Suppose you live a few seconds in M.  Do you then survive in W with 
> probability 0.5?

Assuming you do die in M, even after some years, the probability in H to be 
feeling the one in W will be one, assuming you never dies in W. But this 
assumes mortality, and some transitivity of the probability rules, so the  
question is very complex. The probability in H to be W or M, for a short time,  
is one half, but the probability to be in the place where you stay for a long 
time, will be close to one in a sort of retrospective way. 

All this comes from a simple fact: absolute-death is not a first person 
experience. There is no entry in the first person diary which mention “I died 
today”.

The difficulty is that the first person renormalise the probabilities all the 
time, and that is why making them transitive leads to paradoxes.

Let me try to illustrate. You are in H, just before the WM-duplication. You are 
told in advance that in W you will get a cup of tea, and then be killed. In W 
you get a cup of coffee, and not killed. What is the probability (in H) that 
you will get a cup of tea. It is 1/2. But what is the probability, in H, that 
you will have a long lasting memory of having drink a cup of coffee after that 
experiments: It is 1. In fact, in Moscow, you could (although it is 
psychologically very difficult) still bet that “you” will have a memory of 
having doing coffee, and just an amnesia of M and its cup of tea. This also 
gives some sense that we survive more in our kids and in the value we transmit 
to them, than in bodies and personal first person happening. 

Now, that renormalisation process is not easy, a bit like in QFT, we get 
infinities which are hard to subtract. 


Bruno





> 
> Brent
> 
> -- 
> You received this message because you are subscribed to the Google Groups 
> "Everything List" group.
> To unsubscribe from this group and stop receiving emails from it, send an 
> email to everything-list+unsubscr...@googlegroups.com.
> To view this discussion on the web visit 
> https://groups.google.com/d/msgid/everything-list/76a574e1-dc37-c7eb-33f1-1e12d18ec5a6%40verizon.net.

-- 
You received this message because you are subscribed to the Google Groups 
"Everything List" group.
To unsubscribe from this group and stop receiving emails from it, send an email 
to everything-list+unsubscr...@googlegroups.com.
To view this discussion on the web visit 
https://groups.google.com/d/msgid/everything-list/5FB6A2DC-B54A-45BE-ADFA-653C21C8FB35%40ulb.ac.be.


Re: Quantum immortality

2019-09-15 Thread Bruno Marchal

> On 13 Sep 2019, at 22:28, 'Brent Meeker' via Everything List 
>  wrote:
> 
> 
> 
> On 9/13/2019 10:59 AM, Jason Resch wrote:
>> 
>> 
>> On Wed, Sep 11, 2019 at 6:38 PM Bruce Kellett > <mailto:bhkellet...@gmail.com>> wrote:
>> On Thu, Sep 12, 2019 at 2:55 AM Jason Resch > <mailto:jasonre...@gmail.com>> wrote:
>> On Tuesday, September 10, 2019, Bruce Kellett > <mailto:bhkellet...@gmail.com>> wrote:
>> On Wed, Sep 11, 2019 at 10:18 AM 'Brent Meeker' via Everything List 
>> mailto:everything-list@googlegroups.com>> 
>> wrote:
>> On 9/10/2019 4:30 PM, Bruce Kellett wrote:
>> > Another argument that has been given here before is that if quantum 
>> > immortality is true, then we should expect to see a number of people 
>> > who are considerably older than the normal life expectancy -- and we 
>> > do not see people who are two or three hundred years old. Even if the 
>> > probabilities are very low, there have been an awful lot of people 
>> > born within the last 500 or so years -- some must have survived on our 
>> > branch if this scenario is true.
>> 
>> My argument was that each of us should find ourselves to be much older 
>> than even the oldest people we know.
>>  
>> That is probably the best single argument against quantum immortality: if QI 
>> is true, then the measure of our lifetime after one reaches a normal 
>> lifetime is infinitely greater than the measure before age , say, 120 yr. So 
>> if one finds oneself younger than 120 years, QI is false, and if MWI is 
>> still considered to be true, there must be another argument why MWI does not 
>> imply QI.
>> 
>> 
>> Why do you think that measure only increases with age? On an objective level 
>> it only decreases.
>> 
>> As Bruno would say, "you confuse the 1p with the 1pp." I am talking about my 
>> personal measure of the number of years I have lived. As I get older, the 
>> number of years I have lived increases. If I live to 1000, I have lived more 
>> years between 100 and 1000 than between 1 and 100. This is arithmetic, after 
>> all.
>> 
>> I see.  This reasoning works only under the assumption that finding yourself 
>> in any particular year across your infinite lifespan is equiprobable (i.e. 
>> you can ignore the effects of the number or measure of the various yous in 
>> other branches).  This is what I thought you mean by measure, in terms of 
>> how to calculate probabilities / weights of the various branches.
>>  
>> 
>> But this discussion has gone off the rails. It started as a discussion of 
>> quantum immortality, and the arguments against this notion, even in MWI. The 
>> arguments against QI that have been advanced are that life-threatening 
>> events tend not to be binary or quantum, but rather we enter a period of 
>> slow decline, due to illness or other factors. Consequently, there is no 
>> reason for us to expect to be immortal, even in MWI.
>> 
>> I don't see how that last sentence follows.  It is true MWI doesn't 
>> guarantee we should expect to always survive in the same condition, but it 
>> does guarantee we should survive in some form.
> 
> But what does "we" refer to. Are you saying Jason, with the memories he has 
> at this moment, will always have a successor in the future.   Or are you 
> saying there'll always be a Jason that shares my childhood memories or my 
> memories of last year when that lightning bolt just missed me.
> 
>>  
>> The other argument is that if QI is true, then you would expect to be very 
>> old.
>> 
>> We only know we are very old if our memories accumulate without limit, but 
>> MWI does not guarantee persistence of memory.  It also follows from this 
>> that to know one is immortal (has lived an infinite number of years) 
>> requires an infinitely large brain and memory capacity.
> 
> I don't have to remember everything that happened over 80yrs to know I'm 
> 80yrs old.  In fact I only need to remember my birthday.


And memory is fallible, and memory of age has no more meaning when your age is 
bigger that the nameable or describable number, which happens very soon, 
relatively, for the immortal being trying to keep track of their birthday. 

Immortality is when you are to old to be able to even name your age. After 
that, you have always the same age.

Mortality is an illusion enjoyed by the gods when tired of eternity. It is very 
long 

Bruno


What, you ask, was the beginning of it all?

And it is this ...
Existence that multiplied i

Re: Quantum immortality

2019-09-15 Thread Bruno Marchal

> On 13 Sep 2019, at 22:17, 'Brent Meeker' via Everything List 
>  wrote:
> 
> 
> 
> On 9/13/2019 4:18 AM, Quentin Anciaux wrote:
>> 
>> 
>> Le ven. 13 sept. 2019 à 13:16, Bruce Kellett > <mailto:bhkellet...@gmail.com>> a écrit :
>> On Fri, Sep 13, 2019 at 8:49 PM Bruno Marchal > <mailto:marc...@ulb.ac.be>> wrote:
>> On 12 Sep 2019, at 01:50, Bruce Kellett > <mailto:bhkellet...@gmail.com>> wrote:
>>> On Thu, Sep 12, 2019 at 1:55 AM Bruno Marchal >> <mailto:marc...@ulb.ac.be>> wrote:
>>> On 11 Sep 2019, at 01:30, Bruce Kellett >> <mailto:bhkell...@optusnet.com.au>> wrote:
>>>> From: Bruno Marchal mailto:marc...@ulb.ac.be>>
>>>>>> On 8 Sep 2019, at 13:59, Bruce Kellett >>>>> <mailto:bhkellet...@gmail.com>> wrote:
>>>>>> 
>>>>>> If the only relevance you can find for many worlds is quantum 
>>>>>> immortality, then many worlds is indeed dead. Quantum immortality has 
>>>>>> been shown many times to be a complete nonsense.
>>>>> 
>>>>> Really. I did not known that. Could you give the references.
>>>> Follow the Wikipedia entry on quantum suicide.
>>>> 
>>> That is not what I mean by a  reference.
>>> 
>>> I later gave a reference to the paper by Mallah -- whom you know of, 
>>> apparently. The paper is available at
>>> 
>>> https://arxiv.org/abs/0902.0187 <https://arxiv.org/abs/0902.0187>
>>> 
>>> 
>> 
>> 
>> Yes, the oldest participant in this list have know Jacques Mallah, who 
>> participated a lot in this list.
>> 
>> Mallah is wrong here:
>> 
>> <<
>> Max Tegmark publicized the QS idea, but in some ways he is more of a 
>> moderate on the issue than most of its believers are. If he were to follow 
>> in the footsteps of Don Page and alter his views, recanting belief in QS, it 
>> would be a great help in exposing the belief as a fallacy, and I hold out 
>> hope that it is possible that he will do so.
>> 
>> In his paper [Tegmark 1] QS is explained as follows:
>> 
>> “Since there is exactly one observer having perceptions both before and 
>> after the trigger event, and since it occurred too fast to notice, the MWI 
>> prediction is that” (the experimenter) “will hear “click” with 100% 
>> certainty.”
>> 
>> That is a rather odd statement because he is certainly aware that in the MWI 
>> there is no sense in which it can be rightfully said that “there is exactly 
>> one observer” either before
>> 
>> 
>> or after the experiment. The ket notation may be unhelpful here; indeed, if 
>> the tensor product of kets on the left hand side were expanded instead of 
>> factoring out the observer, there would appear to have been “two observers” 
>> initially.
>> >>
>> 
>> I don't get Mallah's point here, either. I will have to look more clearly at 
>> his argument against QS. I don't think that case is a clear-cut as for QI. 
>> The fact that I am not the oldest person around is clear evidence against QI.
>> 
>> It's wrong, that imply you can nerver have been young.
> 
> And I am young, therefore quantum immortality is wrong.  But exactly where is 
> it wrong. There seem to be two different ideas of quantum immortality.  In 
> one verison, the everything-happens version, is that whatever your state 
> there is a physically possible way for you to survive...like invoking Bruno's 
> magic cosmic rays that just happen to trigger the right nerves for the brain 
> damaged student to ace her test. 


Just to be clear, nobody believes in such magic cosmic rays. They have a 
probability zero in all histories, and I used them only to illustrate a point 
(indeed, they are replaced by the movie projection at the following step).

The computationalist immortality does not rely on such magic. To be sure.




> In this version, no matter your age or circumstance, there will be a 'you' 
> that remembers your age and circumstance indefinitely far into the future.  
> 
> The other version says that almost all Everettian 'copies' of your future 
> will die but there's a non-zero probability of one still existing at any 
> future time.  So then the relative measure of your future self depends on the 
> ratio of copies that haven't died to those that have.  This implicitly 
> assumes that whatever event that causes you to die does not also cause a 
> surviving copy to be created (i.e. it's

Re: Quantum immortality

2019-09-15 Thread Bruno Marchal

> On 13 Sep 2019, at 13:53, John Clark  wrote:
> 
> On Fri, Sep 13, 2019 at 2:14 AM Quentin Anciaux  > wrote:
> 
> > Wel if by "dualist soul" you mean something immaterial about our 
> > consciousness (like I don't know information) can be duplicated then yes it 
> > is dualist and any computational theory of mind is dualist in this sense 
> > then.
> 
> Yes I agree completely. I'm a dualist but there is nothing mystical about 
> that, I just believe that nouns and adjectives refer to different things, 
> nouns refer to physical objects and adjectives refer to nouns. 

A noun refers to any object (that a mind can conceive).

With mechanism, you are not identical with your atoms configurations, which 
change all the time already, and even more when each morning you change of body 
(which makes sense in the Mechanism frame).

But you keep your noun in the process, and indeed, with Mechanism you are the 
owner of your body, but you are not identical with your body.

But you are right: if you believe in mind and matter you are a dualist. The 
problem is how you relate the mind and the body.

If ten very different computers (physical or not) emulate your current brain 
process, you will have ten bodies, and your mind will not be related to “only 
one” of them. Your consciousness is not localised, and eventually the idea of 
locality of consciousness is a mind construct (again, assuming Mechanism all 
along).



> 
> > I suppose you would say that if someone believe "he" can be copied and 
> > uploaded in a virtual environment then he is a dualist 
> 
> I agree again. A standard dictionary will say Information is a noun but I 
> think that demonstrates an inconsistency in human language because adjectives 
> describe nouns and so does information. In the case of uploads information 
> describes the way generic atoms are arranged, and this time I agree with the 
> dictionary, atoms are nouns. Arrange atoms one way and they're me, arrange 
> those exact same atoms another way and they're you.

Atoms are not really nouns, but atoms can have nouns, but then numbers have 
noun too. To say that noun applied only to physical material token necessity 
the belief in some primary substance, which does not work with digital 
mechanism.

Bruno



> 
>  John K Clark
> 
> -- 
> You received this message because you are subscribed to the Google Groups 
> "Everything List" group.
> To unsubscribe from this group and stop receiving emails from it, send an 
> email to everything-list+unsubscr...@googlegroups.com 
> .
> To view this discussion on the web visit 
> https://groups.google.com/d/msgid/everything-list/CAJPayv2DGXZzQfdH4wxwO0YXMaB7SAOP12XoK2C7oimWPskxdQ%40mail.gmail.com
>  
> .

-- 
You received this message because you are subscribed to the Google Groups 
"Everything List" group.
To unsubscribe from this group and stop receiving emails from it, send an email 
to everything-list+unsubscr...@googlegroups.com.
To view this discussion on the web visit 
https://groups.google.com/d/msgid/everything-list/015EF49C-275D-4D2A-9719-A4780112D457%40ulb.ac.be.


Re: Quantum immortality

2019-09-13 Thread Stathis Papaioannou
On Sat, 14 Sep 2019 at 14:30, 'Brent Meeker' via Everything List <
everything-list@googlegroups.com> wrote:

>
>
> On 9/13/2019 3:12 PM, Stathis Papaioannou wrote:
>
>
>
> On Sat, 14 Sep 2019 at 06:38, 'Brent Meeker' via Everything List <
> everything-list@googlegroups.com> wrote:
>
>>
>>
>> On 9/12/2019 11:59 PM, Stathis Papaioannou wrote:
>>
>>
>>
>> On Fri, 13 Sep 2019 at 14:49, Bruce Kellett 
>> wrote:
>>
>>> ...
>>>
>>> Your RSSA assumption is effectively a dualist model -- there is only one
>>> soul that makes you really you, and that soul goes at random into one and
>>> only one copy at any time. Then the chances that this soul-containing copy
>>> is the one that survives, does indeed decrease rapidly with age. But that
>>> is the wrong way to look at it -- there is no 'soul' that makes a copy you.
>>> On the MWI assumptions, every copy is 'you', so since at least one copy
>>> always survives, 'you' will always survive. The number of years you survive
>>> past age 100 is indefinitely large, so you spend more time in those years,
>>> and you have probability one of getting there.
>>>
>>
>> I would not call it dualism. There are many copies, but I am one and only
>> one copy. I do not assume there is a “soul”, just a process that can
>> reflect and say “hey, it’s me”. I don’t know which copy I am and it doesn’t
>> matter. What matters, because it defines survival, is that there be an
>> entity in the future that identifies as being me and remembers being me.
>> Effectively, since I am a process rather than a persisting physical object,
>> I die with every passing moment, and it is only the existence of such
>> entities that identify as being me and remember being me that creates the
>> illusion of survival. I die if no such entities exist anywhere or any time.
>>
>>
>> If the Stahis-here-and-now dies, then no entities remember being
>> Stahis-here-and-now, but there may be ones that remember being
>> Stahis-yesterday.
>>
>
> If I have an episode of amnesia, then effectively that version of me dies,
> while the version of me prior to the period of amnesia survives.
>
>
> According to MWI there are different versions of you who have survived
> consciousness changing events yesterday, last week, last year,
>

But they are not the continuation of my present self. If I have a drug like
midazolam, I will have amnesia for an hour or two, and if I am aware that
this will happen it will be equivalent to anticipating my death.
-- 
Stathis Papaioannou

-- 
You received this message because you are subscribed to the Google Groups 
"Everything List" group.
To unsubscribe from this group and stop receiving emails from it, send an email 
to everything-list+unsubscr...@googlegroups.com.
To view this discussion on the web visit 
https://groups.google.com/d/msgid/everything-list/CAH%3D2ypXFCBhXUGZome1sQzc0%2B%2BF15xjCeDzm1eEQjm8k72%3DQhg%40mail.gmail.com.


Re: Quantum immortality

2019-09-13 Thread 'Brent Meeker' via Everything List



On 9/13/2019 3:25 PM, Jason Resch wrote:



On Fri, Sep 13, 2019 at 3:28 PM 'Brent Meeker' via Everything List 
<mailto:everything-list@googlegroups.com>> wrote:


...
I don't have to remember everything that happened over 80yrs to
know I'm 80yrs old.  In fact I only need to remember my birthday.


To know your birthday requires log(n) bits, which goes to infinity as 
n goes to infinity.


Fortunately, 30 July 1939 doesn't got to infinity.  It stays the same.

Without an ever expanding memory, you are limited to experiencing at 
most M^2 states, where M is your memory capacity in bits.  If M is 
finite, then infinite years don't matter, you will begin to revisit 
previous states.


That 2^M.

By that kind of reasoning the universe will revisit previous states and 
there's no meaning to "immortality".  But there's still obvious meaning 
to living a 10,000yrs vs 100yrs.


Brent

--
You received this message because you are subscribed to the Google Groups 
"Everything List" group.
To unsubscribe from this group and stop receiving emails from it, send an email 
to everything-list+unsubscr...@googlegroups.com.
To view this discussion on the web visit 
https://groups.google.com/d/msgid/everything-list/c4181791-0f80-b4bc-712e-9d12669cd6dc%40verizon.net.


Re: Quantum immortality

2019-09-13 Thread 'Brent Meeker' via Everything List



On 9/13/2019 3:12 PM, Stathis Papaioannou wrote:



On Sat, 14 Sep 2019 at 06:38, 'Brent Meeker' via Everything List 
> wrote:




On 9/12/2019 11:59 PM, Stathis Papaioannou wrote:



On Fri, 13 Sep 2019 at 14:49, Bruce Kellett
mailto:bhkellet...@gmail.com>> wrote:

...

Your RSSA assumption is effectively a dualist model -- there
is only one soul that makes you really you, and that soul
goes at random into one and only one copy at any time. Then
the chances that this soul-containing copy is the one that
survives, does indeed decrease rapidly with age. But that is
the wrong way to look at it -- there is no 'soul' that makes
a copy you. On the MWI assumptions, every copy is 'you', so
since at least one copy always survives, 'you' will always
survive. The number of years you survive past age 100 is
indefinitely large, so you spend more time in those years,
and you have probability one of getting there.


I would not call it dualism. There are many copies, but I am one
and only one copy. I do not assume there is a “soul”, just a
process that can reflect and say “hey, it’s me”. I don’t know
which copy I am and it doesn’t matter. What matters, because it
defines survival, is that there be an entity in the future that
identifies as being me and remembers being me. Effectively, since
I am a process rather than a persisting physical object, I die
with every passing moment, and it is only the existence of such
entities that identify as being me and remember being me that
creates the illusion of survival. I die if no such entities exist
anywhere or any time.


If the Stahis-here-and-now dies, then no entities remember being
Stahis-here-and-now, but there may be ones that remember being
Stahis-yesterday.


If I have an episode of amnesia, then effectively that version of me 
dies, while the version of me prior to the period of amnesia survives.


According to MWI there are different versions of you who have survived 
consciousness changing events yesterday, last week, last year, ...


Brent

--
You received this message because you are subscribed to the Google Groups 
"Everything List" group.
To unsubscribe from this group and stop receiving emails from it, send an email 
to everything-list+unsubscr...@googlegroups.com.
To view this discussion on the web visit 
https://groups.google.com/d/msgid/everything-list/7626e66a-791c-48da-169e-d03f3e8ec726%40verizon.net.


Re: Quantum immortality

2019-09-13 Thread Jason Resch
On Friday, September 13, 2019, Bruce Kellett  wrote:

> On Sat, Sep 14, 2019 at 8:25 AM Jason Resch  wrote:
>
>> On Fri, Sep 13, 2019 at 3:28 PM 'Brent Meeker' via Everything List <
>> everything-list@googlegroups.com> wrote:
>>
>>> I don't have to remember everything that happened over 80yrs to know I'm
>>> 80yrs old.  In fact I only need to remember my birthday.
>>>
>>>
>> To know your birthday requires log(n) bits, which goes to infinity as n
>> goes to infinity.
>>
>
> Rubbish. I just have to look at my birth certificate!
>

But you would be unable to calculate (or contemplate) your age.


>
>
>>  Without an ever expanding memory, you are limited to experiencing at
>> most M^2 states, where M is your memory capacity in bits.  If M is finite,
>> then infinite years don't matter, you will begin to revisit previous states.
>>
>
> That assume that in order to survive you have to remember everything that
> ever happened to you. And that is such obvious nonsense that it doesn't
> merit a rebuttal.
>
>
You misunderstand. This has nothing to do with remembering everything, it
speaks to the finite number of possible experiences a finite mind can have.

Jason

-- 
You received this message because you are subscribed to the Google Groups 
"Everything List" group.
To unsubscribe from this group and stop receiving emails from it, send an email 
to everything-list+unsubscr...@googlegroups.com.
To view this discussion on the web visit 
https://groups.google.com/d/msgid/everything-list/CA%2BBCJUhbvGYTZr46B-oe_bTK9SzGGY9d6kwJmH05jaodwOsEiQ%40mail.gmail.com.


Re: Quantum immortality

2019-09-13 Thread smitra

On 13-09-2019 22:23, 'Brent Meeker' via Everything List wrote:

On 9/13/2019 10:12 AM, smitra wrote:

On 11-09-2019 19:41, Stathis Papaioannou wrote:

On Thu, 12 Sep 2019 at 00:51, smitra  wrote:


Back to basics. There exists a universal wavefunction that evolves
according to the Schrodinger equation. Observers are internal
structures
in this description. Whether or not one believes that the Born rule
can
be derived or not, what matters in practice is that you'll end up
having
to use it, so you have to assign a measure for observations that is
given by the summation of the squared modulus of the states that
correspond to those observations. The information about personal
identity must then also be extracted from the wavefunction, so one
cannot insert this in an ad hoc way.

Quantum immortality is therefore wrong because the measure of the
states
that correspond to extremely old observers is small.


This means that if you don’t know if you are young or very old and
have to guess, you are more likely to be right if you guess that you
are young. But it does not mean that you won’t inevitably become
very old.--



Indeed, there is always going to be a very small probability of 
finding yourself in a very atypical situation. In the MWI we would 
need to take into account all possible relevant processes here, and 
forgetting about some fact and learning about that again will tend to 
replace extremely atypical information from he memory and replace that 
by more typical information.


So if I forget about that accident in 1987 my collar bone won't have
been broken and it will be straight again?


There exists a copy of you who didn't had that accident. But he may have 
suffered some other accident.




In the MWI the so-called stability of records of measurements is not 
valid in an absolute sense. It's only valid in each branch, but you 
can hop from one to another branch such that all your memories are 
swapped with those of your copy. So, if I find myself to be a trillion 
years old and I go top sleep then I'll most likely wake up as a copy 
that has that information about me being a trillion years old replaced 
by a more reasonable lower number.


If that happened how would you know?  You'd be finding yourself where
you were...which is just a tautology.

That's right, so this doesn't provide for a test for the MWI. But when 
we assume the validity of the MWI we must take into account this effect 
which effectively erases freak events.


This also means that even if you win the lottery and become a multi 
millionaire, you'll very likely end up forgetting this and finding 
yourself to not having won the lottery.


Which is to say you'd not win.



Indeed, you won't find yourself in the sector where you've won.


Saibal

--
You received this message because you are subscribed to the Google Groups 
"Everything List" group.
To unsubscribe from this group and stop receiving emails from it, send an email 
to everything-list+unsubscr...@googlegroups.com.
To view this discussion on the web visit 
https://groups.google.com/d/msgid/everything-list/721a25f25dd77e57fe4a4d56d697ca49%40zonnet.nl.


Re: Quantum immortality

2019-09-13 Thread Bruce Kellett
On Sat, Sep 14, 2019 at 8:25 AM Jason Resch  wrote:

> On Fri, Sep 13, 2019 at 3:28 PM 'Brent Meeker' via Everything List <
> everything-list@googlegroups.com> wrote:
>
>> I don't have to remember everything that happened over 80yrs to know I'm
>> 80yrs old.  In fact I only need to remember my birthday.
>>
>>
> To know your birthday requires log(n) bits, which goes to infinity as n
> goes to infinity.
>

Rubbish. I just have to look at my birth certificate!


>  Without an ever expanding memory, you are limited to experiencing at most
> M^2 states, where M is your memory capacity in bits.  If M is finite, then
> infinite years don't matter, you will begin to revisit previous states.
>

That assume that in order to survive you have to remember everything that
ever happened to you. And that is such obvious nonsense that it doesn't
merit a rebuttal.

Bruce

-- 
You received this message because you are subscribed to the Google Groups 
"Everything List" group.
To unsubscribe from this group and stop receiving emails from it, send an email 
to everything-list+unsubscr...@googlegroups.com.
To view this discussion on the web visit 
https://groups.google.com/d/msgid/everything-list/CAFxXSLRuQ7y-MEX9WB-YYr-cYaY18P6xpN-fS%2B8L9ng%3DK-nJ6A%40mail.gmail.com.


Re: Quantum immortality

2019-09-13 Thread Jason Resch
On Fri, Sep 13, 2019 at 5:25 PM Jason Resch  wrote:

>
> Without an ever expanding memory, you are limited to experiencing at most
> M^2 states, where M is your memory capacity in bits.  If M is finite, then
> infinite years don't matter, you will begin to revisit previous states.
>
>
Correction: the above should be 2^M states.

-- 
You received this message because you are subscribed to the Google Groups 
"Everything List" group.
To unsubscribe from this group and stop receiving emails from it, send an email 
to everything-list+unsubscr...@googlegroups.com.
To view this discussion on the web visit 
https://groups.google.com/d/msgid/everything-list/CA%2BBCJUgedio7sjCbN%3DVRHczs2p_nxRyjJFsYYLnaHV8H2CDh1Q%40mail.gmail.com.


Re: Quantum immortality

2019-09-13 Thread Jason Resch
On Fri, Sep 13, 2019 at 3:28 PM 'Brent Meeker' via Everything List <
everything-list@googlegroups.com> wrote:

>
>
> On 9/13/2019 10:59 AM, Jason Resch wrote:
>
>
>
> On Wed, Sep 11, 2019 at 6:38 PM Bruce Kellett 
> wrote:
>
>> On Thu, Sep 12, 2019 at 2:55 AM Jason Resch  wrote:
>>
>>> On Tuesday, September 10, 2019, Bruce Kellett 
>>> wrote:
>>>
>>>> On Wed, Sep 11, 2019 at 10:18 AM 'Brent Meeker' via Everything List <
>>>> everything-list@googlegroups.com> wrote:
>>>>
>>>>> On 9/10/2019 4:30 PM, Bruce Kellett wrote:
>>>>> > Another argument that has been given here before is that if quantum
>>>>> > immortality is true, then we should expect to see a number of people
>>>>> > who are considerably older than the normal life expectancy -- and we
>>>>> > do not see people who are two or three hundred years old. Even if
>>>>> the
>>>>> > probabilities are very low, there have been an awful lot of people
>>>>> > born within the last 500 or so years -- some must have survived on
>>>>> our
>>>>> > branch if this scenario is true.
>>>>>
>>>>> My argument was that each of us should find ourselves to be much older
>>>>> than even the oldest people we know.
>>>>
>>>>
>>>> That is probably the best single argument against quantum immortality:
>>>> if QI is true, then the measure of our lifetime after one reaches a normal
>>>> lifetime is infinitely greater than the measure before age , say, 120 yr.
>>>> So if one finds oneself younger than 120 years, QI is false, and if MWI is
>>>> still considered to be true, there must be another argument why MWI does
>>>> not imply QI.
>>>>
>>>
>>>
>>> Why do you think that measure only increases with age? On an objective
>>> level it only decreases.
>>>
>>
>> As Bruno would say, "you confuse the 1p with the 1pp." I am talking about
>> my personal measure of the number of years I have lived. As I get older,
>> the number of years I have lived increases. If I live to 1000, I have lived
>> more years between 100 and 1000 than between 1 and 100. This is arithmetic,
>> after all.
>>
>
> I see.  This reasoning works only under the assumption that finding
> yourself in any particular year across your infinite lifespan is
> equiprobable (i.e. you can ignore the effects of the number or measure of
> the various yous in other branches).  This is what I thought you mean by
> measure, in terms of how to calculate probabilities / weights of the
> various branches.
>
>
>>
>> But this discussion has gone off the rails. It started as a discussion of
>> quantum immortality, and the arguments against this notion, even in MWI.
>> The arguments against QI that have been advanced are that life-threatening
>> events tend not to be binary or quantum, but rather we enter a period of
>> slow decline, due to illness or other factors. Consequently, there is no
>> reason for us to expect to be immortal, even in MWI.
>>
>
> I don't see how that last sentence follows.  It is true MWI doesn't
> guarantee we should expect to always survive in the same condition, but it
> does guarantee we should survive in some form.
>
>
> But what does "we" refer to. Are you saying Jason, with the memories he
> has at this moment, will always have a successor in the future.   Or are
> you saying there'll always be a Jason that shares my childhood memories or
> my memories of last year when that lightning bolt just missed me.
>

It depends on your definition of personal identity. I think in this case,
all of them might work.  There is both bodily/physical and
mental/psychological continuity under MWI.
There will always exist someone remembering my current state as their
previous state, and there is physical/bodily continuity, or viewed from the
other direction, with the mind as a program, there is always some branch
that realizes a consistent extension of that computation.


>
>
>
>> The other argument is that if QI is true, then you would expect to be
>> very old.
>>
>
> We only know we are very old if our memories accumulate without limit, but
> MWI does not guarantee persistence of memory.  It also follows from this
> that to know one is immortal (has lived an infinite number of years)
> requires an infinitely large brain and memory capacity.
>
>
> I don't have to remember everything tha

Re: Quantum immortality

2019-09-13 Thread Stathis Papaioannou
On Sat, 14 Sep 2019 at 06:38, 'Brent Meeker' via Everything List <
everything-list@googlegroups.com> wrote:

>
>
> On 9/12/2019 11:59 PM, Stathis Papaioannou wrote:
>
>
>
> On Fri, 13 Sep 2019 at 14:49, Bruce Kellett  wrote:
>
>> ...
>>
>> Your RSSA assumption is effectively a dualist model -- there is only one
>> soul that makes you really you, and that soul goes at random into one and
>> only one copy at any time. Then the chances that this soul-containing copy
>> is the one that survives, does indeed decrease rapidly with age. But that
>> is the wrong way to look at it -- there is no 'soul' that makes a copy you.
>> On the MWI assumptions, every copy is 'you', so since at least one copy
>> always survives, 'you' will always survive. The number of years you survive
>> past age 100 is indefinitely large, so you spend more time in those years,
>> and you have probability one of getting there.
>>
>
> I would not call it dualism. There are many copies, but I am one and only
> one copy. I do not assume there is a “soul”, just a process that can
> reflect and say “hey, it’s me”. I don’t know which copy I am and it doesn’t
> matter. What matters, because it defines survival, is that there be an
> entity in the future that identifies as being me and remembers being me.
> Effectively, since I am a process rather than a persisting physical object,
> I die with every passing moment, and it is only the existence of such
> entities that identify as being me and remember being me that creates the
> illusion of survival. I die if no such entities exist anywhere or any time.
>
>
> If the Stahis-here-and-now dies, then no entities remember being
> Stahis-here-and-now, but there may be ones that remember being
> Stahis-yesterday.
>

If I have an episode of amnesia, then effectively that version of me dies,
while the version of me prior to the period of amnesia survives.

> --
Stathis Papaioannou

-- 
You received this message because you are subscribed to the Google Groups 
"Everything List" group.
To unsubscribe from this group and stop receiving emails from it, send an email 
to everything-list+unsubscr...@googlegroups.com.
To view this discussion on the web visit 
https://groups.google.com/d/msgid/everything-list/CAH%3D2ypWFX7ZHV9OzfOTu0inePEHQ4n5QRHckL5jE6aKAGSY0LA%40mail.gmail.com.


Re: Quantum immortality

2019-09-13 Thread Stathis Papaioannou
On Sat, 14 Sep 2019 at 03:12, smitra  wrote:

> On 11-09-2019 19:41, Stathis Papaioannou wrote:
> > On Thu, 12 Sep 2019 at 00:51, smitra  wrote:
> >
> >> Back to basics. There exists a universal wavefunction that evolves
> >> according to the Schrodinger equation. Observers are internal
> >> structures
> >> in this description. Whether or not one believes that the Born rule
> >> can
> >> be derived or not, what matters in practice is that you'll end up
> >> having
> >> to use it, so you have to assign a measure for observations that is
> >> given by the summation of the squared modulus of the states that
> >> correspond to those observations. The information about personal
> >> identity must then also be extracted from the wavefunction, so one
> >> cannot insert this in an ad hoc way.
> >>
> >> Quantum immortality is therefore wrong because the measure of the
> >> states
> >> that correspond to extremely old observers is small.
> >
> > This means that if you don’t know if you are young or very old and
> > have to guess, you are more likely to be right if you guess that you
> > are young. But it does not mean that you won’t inevitably become
> > very old.--
>
>
> Indeed, there is always going to be a very small probability of finding
> yourself in a very atypical situation. In the MWI we would need to take
> into account all possible relevant processes here, and forgetting about
> some fact and learning about that again will tend to replace extremely
> atypical information from he memory and replace that by more typical
> information.
>
> In the MWI the so-called stability of records of measurements is not
> valid in an absolute sense. It's only valid in each branch, but you can
> hop from one to another branch such that all your memories are swapped
> with those of your copy. So, if I find myself to be a trillion years old
> and I go top sleep then I'll most likely wake up as a copy that has that
> information about me being a trillion years old replaced by a more
> reasonable lower number.
>
> This also means that even if you win the lottery and become a multi
> millionaire, you'll very likely end up forgetting this and finding
> yourself to not having won the lottery.


I only survive because I remember the person I was. If a branch I find
myself in is a dead end then that version of me dies. So if I find myself
being a trillion years old and a trillion year old plus a day copy does not
exist, or exists but forgets he is me, then the trillion year old copy
dies. But if such a copy does exist with an intact memory, then the
trillion year copy survives in this copy. It is not an option that I switch
from a trillion year old copy to a younger copy with greater measure, any
more than it is an option that I will tomorrow wake up Chinese, because
there are more Chinese than Australians.

> --
Stathis Papaioannou

-- 
You received this message because you are subscribed to the Google Groups 
"Everything List" group.
To unsubscribe from this group and stop receiving emails from it, send an email 
to everything-list+unsubscr...@googlegroups.com.
To view this discussion on the web visit 
https://groups.google.com/d/msgid/everything-list/CAH%3D2ypUAq_Uv%2Bo3i_cfAsLAwd3M593x5u%2BmRkRA6LdJVTnHUjA%40mail.gmail.com.


Re: Quantum immortality

2019-09-13 Thread 'Brent Meeker' via Everything List




On 9/13/2019 3:38 AM, Bruno Marchal wrote:

On 11 Sep 2019, at 16:51, smitra  wrote:

Back to basics. There exists a universal wavefunction that evolves according to 
the Schrodinger equation. Observers are internal structures in this 
description. Whether or not one believes that the Born rule can be derived or 
not, what matters in practice is that you'll end up having to use it, so you 
have to assign a measure for observations that is given by the summation of the 
squared modulus of the states that correspond to those observations. The 
information about personal identity must then also be extracted from the 
wavefunction, so one cannot insert this in an ad hoc way.

Quantum immortality is therefore wrong because the measure of the states that 
correspond to extremely old observers is small.


The same reasoning would apply to “quantum suicide”, where it is clear that we 
survive all the time; given that we cannot take into account the world where we 
do not.

If in H you are multiplied in W and M, but directly killed in M, you survive in W with 
probability one. That is why we add p or <>t to []p to transform the logic of belief 
([]p) into a probability logic ([]p & <>t).


Suppose you live a few seconds in M.  Do you then survive in W with 
probability 0.5?


Brent

--
You received this message because you are subscribed to the Google Groups 
"Everything List" group.
To unsubscribe from this group and stop receiving emails from it, send an email 
to everything-list+unsubscr...@googlegroups.com.
To view this discussion on the web visit 
https://groups.google.com/d/msgid/everything-list/76a574e1-dc37-c7eb-33f1-1e12d18ec5a6%40verizon.net.


Re: Quantum immortality

2019-09-13 Thread 'Brent Meeker' via Everything List



On 9/12/2019 11:59 PM, Stathis Papaioannou wrote:



On Fri, 13 Sep 2019 at 14:49, Bruce Kellett > wrote:


...

Your RSSA assumption is effectively a dualist model -- there is
only one soul that makes you really you, and that soul goes at
random into one and only one copy at any time. Then the chances
that this soul-containing copy is the one that survives, does
indeed decrease rapidly with age. But that is the wrong way to
look at it -- there is no 'soul' that makes a copy you. On the MWI
assumptions, every copy is 'you', so since at least one copy
always survives, 'you' will always survive. The number of years
you survive past age 100 is indefinitely large, so you spend more
time in those years, and you have probability one of getting there.


I would not call it dualism. There are many copies, but I am one and 
only one copy. I do not assume there is a “soul”, just a process that 
can reflect and say “hey, it’s me”. I don’t know which copy I am and 
it doesn’t matter. What matters, because it defines survival, is that 
there be an entity in the future that identifies as being me and 
remembers being me. Effectively, since I am a process rather than a 
persisting physical object, I die with every passing moment, and it is 
only the existence of such entities that identify as being me and 
remember being me that creates the illusion of survival. I die if no 
such entities exist anywhere or any time.


If the Stahis-here-and-now dies, then no entities remember being 
Stahis-here-and-now, but there may be ones that remember being 
Stahis-yesterday.


Brent

--
You received this message because you are subscribed to the Google Groups 
"Everything List" group.
To unsubscribe from this group and stop receiving emails from it, send an email 
to everything-list+unsubscr...@googlegroups.com.
To view this discussion on the web visit 
https://groups.google.com/d/msgid/everything-list/f7d2adc1-0029-b08a-8dd0-59003ef729c3%40verizon.net.


Re: Quantum immortality

2019-09-13 Thread 'Brent Meeker' via Everything List



On 9/13/2019 10:59 AM, Jason Resch wrote:



On Wed, Sep 11, 2019 at 6:38 PM Bruce Kellett <mailto:bhkellet...@gmail.com>> wrote:


On Thu, Sep 12, 2019 at 2:55 AM Jason Resch mailto:jasonre...@gmail.com>> wrote:

On Tuesday, September 10, 2019, Bruce Kellett
mailto:bhkellet...@gmail.com>> wrote:

On Wed, Sep 11, 2019 at 10:18 AM 'Brent Meeker' via
Everything List mailto:everything-list@googlegroups.com>> wrote:

On 9/10/2019 4:30 PM, Bruce Kellett wrote:
> Another argument that has been given here before is
    that if quantum
> immortality is true, then we should expect to see a
number of people
> who are considerably older than the normal life
expectancy -- and we
> do not see people who are two or three hundred years
old. Even if the
> probabilities are very low, there have been an awful
lot of people
> born within the last 500 or so years -- some must
have survived on our
> branch if this scenario is true.

My argument was that each of us should find ourselves
to be much older
than even the oldest people we know.

That is probably the best single argument against quantum
immortality: if QI is true, then the measure of our
lifetime after one reaches a normal lifetime is infinitely
greater than the measure before age , say, 120 yr. So if
one finds oneself younger than 120 years, QI is false, and
if MWI is still considered to be true, there must be
another argument why MWI does not imply QI.



Why do you think that measure only increases with age? On an
objective level it only decreases.


As Bruno would say, "you confuse the 1p with the 1pp." I am
talking about my personal measure of the number of years I have
lived. As I get older, the number of years I have lived increases.
If I live to 1000, I have lived more years between 100 and 1000
than between 1 and 100. This is arithmetic, after all.


I see.  This reasoning works only under the assumption that finding 
yourself in any particular year across your infinite lifespan is 
equiprobable (i.e. you can ignore the effects of the number or measure 
of the various yous in other branches).  This is what I thought you 
mean by measure, in terms of how to calculate probabilities / weights 
of the various branches.



But this discussion has gone off the rails. It started as a
discussion of quantum immortality, and the arguments against this
notion, even in MWI. The arguments against QI that have been
advanced are that life-threatening events tend not to be binary or
quantum, but rather we enter a period of slow decline, due to
illness or other factors. Consequently, there is no reason for us
to expect to be immortal, even in MWI.


I don't see how that last sentence follows.  It is true MWI doesn't 
guarantee we should expect to always survive in the same condition, 
but it does guarantee we should survive in some form.


But what does "we" refer to. Are you saying Jason, with the memories he 
has at this moment, will always have a successor in the future. Or are 
you saying there'll always be a Jason that shares my childhood memories 
or my memories of last year when that lightning bolt just missed me.



The other argument is that if QI is true, then you would expect to
be very old.


We only know we are very old if our memories accumulate without limit, 
but MWI does not guarantee persistence of memory.  It also follows 
from this that to know one is immortal (has lived an infinite number 
of years) requires an infinitely large brain and memory capacity.


I don't have to remember everything that happened over 80yrs to know I'm 
80yrs old.  In fact I only need to remember my birthday.


Brent



This argument was advanced by Mallah (arXiv: 0905.0187) and has
not been satisfactorily rebutted.


Mallah used to contribute to this list.  You can review some of his 
past discussions in the archives which debate this very point: 
https://groups.google.com/forum/#!searchin/everything-list/Mallah%7Csort:date (some 
now 20 years old)


Jason
--
You received this message because you are subscribed to the Google 
Groups "Everything List" group.
To unsubscribe from this group and stop receiving emails from it, send 
an email to everything-list+unsubscr...@googlegroups.com 
<mailto:everything-list+unsubscr...@googlegroups.com>.
To view this discussion on the web visit 
https://groups.google.com/d/msgid/everything-list/CA%2BBCJUiZ0pLP7%3D2_Oyy3jYHxTVxYfXX4aZwXgri_THNoMCVSYA%40mai

Re: Quantum immortality

2019-09-13 Thread 'Brent Meeker' via Everything List




On 9/13/2019 10:12 AM, smitra wrote:

On 11-09-2019 19:41, Stathis Papaioannou wrote:

On Thu, 12 Sep 2019 at 00:51, smitra  wrote:


Back to basics. There exists a universal wavefunction that evolves
according to the Schrodinger equation. Observers are internal
structures
in this description. Whether or not one believes that the Born rule
can
be derived or not, what matters in practice is that you'll end up
having
to use it, so you have to assign a measure for observations that is
given by the summation of the squared modulus of the states that
correspond to those observations. The information about personal
identity must then also be extracted from the wavefunction, so one
cannot insert this in an ad hoc way.

Quantum immortality is therefore wrong because the measure of the
states
that correspond to extremely old observers is small.


This means that if you don’t know if you are young or very old and
have to guess, you are more likely to be right if you guess that you
are young. But it does not mean that you won’t inevitably become
very old.--



Indeed, there is always going to be a very small probability of 
finding yourself in a very atypical situation. In the MWI we would 
need to take into account all possible relevant processes here, and 
forgetting about some fact and learning about that again will tend to 
replace extremely atypical information from he memory and replace that 
by more typical information.


So if I forget about that accident in 1987 my collar bone won't have 
been broken and it will be straight again?


In the MWI the so-called stability of records of measurements is not 
valid in an absolute sense. It's only valid in each branch, but you 
can hop from one to another branch such that all your memories are 
swapped with those of your copy. So, if I find myself to be a trillion 
years old and I go top sleep then I'll most likely wake up as a copy 
that has that information about me being a trillion years old replaced 
by a more reasonable lower number.


If that happened how would you know?  You'd be finding yourself where 
you were...which is just a tautology.




This also means that even if you win the lottery and become a multi 
millionaire, you'll very likely end up forgetting this and finding 
yourself to not having won the lottery.


Which is to say you'd not win.

Brent

--
You received this message because you are subscribed to the Google Groups 
"Everything List" group.
To unsubscribe from this group and stop receiving emails from it, send an email 
to everything-list+unsubscr...@googlegroups.com.
To view this discussion on the web visit 
https://groups.google.com/d/msgid/everything-list/36226106-cf16-6b09-9a58-d610af2080ee%40verizon.net.


Re: Quantum immortality

2019-09-13 Thread 'Brent Meeker' via Everything List



On 9/13/2019 4:18 AM, Quentin Anciaux wrote:



Le ven. 13 sept. 2019 à 13:16, Bruce Kellett <mailto:bhkellet...@gmail.com>> a écrit :


On Fri, Sep 13, 2019 at 8:49 PM Bruno Marchal mailto:marc...@ulb.ac.be>> wrote:

On 12 Sep 2019, at 01:50, Bruce Kellett mailto:bhkellet...@gmail.com>> wrote:

On Thu, Sep 12, 2019 at 1:55 AM Bruno Marchal
mailto:marc...@ulb.ac.be>> wrote:

On 11 Sep 2019, at 01:30, Bruce Kellett
mailto:bhkell...@optusnet.com.au>> wrote:

From: *Bruno Marchal* mailto:marc...@ulb.ac.be>>

On 8 Sep 2019, at 13:59, Bruce Kellett
mailto:bhkellet...@gmail.com>>
wrote:

If the only relevance you can find for many worlds is
quantum immortality, then many worlds is indeed dead.
Quantum immortality has been shown many times to be a
complete nonsense.


Really. I did not known that. Could you give the
references.


Follow the Wikipedia entry on quantum suicide.


That is not what I mean by a  reference.


I later gave a reference to the paper by Mallah -- whom you
know of, apparently. The paper is available at

https://arxiv.org/abs/0902.0187





Yes, the oldest participant in this list have know Jacques
Mallah, who participated a lot in this list.

Mallah is wrong here:

<<
Max Tegmark publicized the QS idea, but in some ways he is
more of a moderate on the issue than most of its believers
are. If he were to follow in the footsteps of Don Page
and alter his views, recanting belief in QS, it would be a
great help in exposing the belief as a fallacy, and I hold out
hope that it is possible that he will do so.

In his paper [Tegmark 1] QS is explained as follows:

“Since there is exactly one observer having perceptions both
before and after the trigger event, and since it occurred too
fast to notice, the MWI prediction is that” (the experimenter)
“will hear “click” with 100% certainty.”

That is a rather odd statement because he is certainly aware
that in the MWI there is no sense in which it can be
rightfully said that “there is exactly one observer” either before

page13image25488
or after the experiment. The ket notation may be unhelpful
here; indeed, if the tensor product of kets on the left hand
side were expanded instead of factoring out the
observer, there would appear to have been “two observers”
initially.
>>


I don't get Mallah's point here, either. I will have to look more
clearly at his argument against QS. I don't think that case is a
clear-cut as for QI. The fact that I am not the oldest person
around is clear evidence against QI.


It's wrong, that imply you can nerver have been young.


And I am young, therefore quantum immortality is wrong.  But exactly 
where is it wrong. There seem to be two different ideas of quantum 
immortality.  In one verison, the everything-happens version, is that 
whatever your state there is a physically possible way for you to 
survive...like invoking Bruno's magic cosmic rays that just happen to 
trigger the right nerves for the brain damaged student to ace her test.  
In this version, no matter your age or circumstance, there will be a 
'you' that remembers your age and circumstance indefinitely far into the 
future.


The other version says that almost all Everettian 'copies' of your 
future will die but there's a non-zero probability of one still existing 
at any future time.  So then the relative measure of your future self 
depends on the ratio of copies that haven't died to those that have.  
This implicitly assumes that whatever event that causes you to die does 
not also cause a surviving copy to be created (i.e. it's not a Tegmark 
machine gun).


Brent

--
You received this message because you are subscribed to the Google Groups 
"Everything List" group.
To unsubscribe from this group and stop receiving emails from it, send an email 
to everything-list+unsubscr...@googlegroups.com.
To view this discussion on the web visit 
https://groups.google.com/d/msgid/everything-list/b3b00d6b-4757-702b-ca23-2ab2e6e3bfb6%40verizon.net.


Re: Quantum immortality

2019-09-13 Thread Jason Resch
On Wed, Sep 11, 2019 at 6:38 PM Bruce Kellett  wrote:

> On Thu, Sep 12, 2019 at 2:55 AM Jason Resch  wrote:
>
>> On Tuesday, September 10, 2019, Bruce Kellett 
>> wrote:
>>
>>> On Wed, Sep 11, 2019 at 10:18 AM 'Brent Meeker' via Everything List <
>>> everything-list@googlegroups.com> wrote:
>>>
>>>> On 9/10/2019 4:30 PM, Bruce Kellett wrote:
>>>> > Another argument that has been given here before is that if quantum
>>>> > immortality is true, then we should expect to see a number of people
>>>> > who are considerably older than the normal life expectancy -- and we
>>>> > do not see people who are two or three hundred years old. Even if the
>>>> > probabilities are very low, there have been an awful lot of people
>>>> > born within the last 500 or so years -- some must have survived on
>>>> our
>>>> > branch if this scenario is true.
>>>>
>>>> My argument was that each of us should find ourselves to be much older
>>>> than even the oldest people we know.
>>>
>>>
>>> That is probably the best single argument against quantum immortality:
>>> if QI is true, then the measure of our lifetime after one reaches a normal
>>> lifetime is infinitely greater than the measure before age , say, 120 yr.
>>> So if one finds oneself younger than 120 years, QI is false, and if MWI is
>>> still considered to be true, there must be another argument why MWI does
>>> not imply QI.
>>>
>>
>>
>> Why do you think that measure only increases with age? On an objective
>> level it only decreases.
>>
>
> As Bruno would say, "you confuse the 1p with the 1pp." I am talking about
> my personal measure of the number of years I have lived. As I get older,
> the number of years I have lived increases. If I live to 1000, I have lived
> more years between 100 and 1000 than between 1 and 100. This is arithmetic,
> after all.
>

I see.  This reasoning works only under the assumption that finding
yourself in any particular year across your infinite lifespan is
equiprobable (i.e. you can ignore the effects of the number or measure of
the various yous in other branches).  This is what I thought you mean by
measure, in terms of how to calculate probabilities / weights of the
various branches.


>
> But this discussion has gone off the rails. It started as a discussion of
> quantum immortality, and the arguments against this notion, even in MWI.
> The arguments against QI that have been advanced are that life-threatening
> events tend not to be binary or quantum, but rather we enter a period of
> slow decline, due to illness or other factors. Consequently, there is no
> reason for us to expect to be immortal, even in MWI.
>

I don't see how that last sentence follows.  It is true MWI doesn't
guarantee we should expect to always survive in the same condition, but it
does guarantee we should survive in some form.


> The other argument is that if QI is true, then you would expect to be very
> old.
>

We only know we are very old if our memories accumulate without limit, but
MWI does not guarantee persistence of memory.  It also follows from this
that to know one is immortal (has lived an infinite number of years)
requires an infinitely large brain and memory capacity.


> This argument was advanced by Mallah (arXiv: 0905.0187) and has not been
> satisfactorily rebutted.
>

Mallah used to contribute to this list.  You can review some of his past
discussions in the archives which debate this very point:
https://groups.google.com/forum/#!searchin/everything-list/Mallah%7Csort:date
(some
now 20 years old)

Jason

-- 
You received this message because you are subscribed to the Google Groups 
"Everything List" group.
To unsubscribe from this group and stop receiving emails from it, send an email 
to everything-list+unsubscr...@googlegroups.com.
To view this discussion on the web visit 
https://groups.google.com/d/msgid/everything-list/CA%2BBCJUiZ0pLP7%3D2_Oyy3jYHxTVxYfXX4aZwXgri_THNoMCVSYA%40mail.gmail.com.


Re: Quantum immortality

2019-09-13 Thread smitra

On 11-09-2019 19:41, Stathis Papaioannou wrote:

On Thu, 12 Sep 2019 at 00:51, smitra  wrote:


Back to basics. There exists a universal wavefunction that evolves
according to the Schrodinger equation. Observers are internal
structures
in this description. Whether or not one believes that the Born rule
can
be derived or not, what matters in practice is that you'll end up
having
to use it, so you have to assign a measure for observations that is
given by the summation of the squared modulus of the states that
correspond to those observations. The information about personal
identity must then also be extracted from the wavefunction, so one
cannot insert this in an ad hoc way.

Quantum immortality is therefore wrong because the measure of the
states
that correspond to extremely old observers is small.


This means that if you don’t know if you are young or very old and
have to guess, you are more likely to be right if you guess that you
are young. But it does not mean that you won’t inevitably become
very old.--



Indeed, there is always going to be a very small probability of finding 
yourself in a very atypical situation. In the MWI we would need to take 
into account all possible relevant processes here, and forgetting about 
some fact and learning about that again will tend to replace extremely 
atypical information from he memory and replace that by more typical 
information.


In the MWI the so-called stability of records of measurements is not 
valid in an absolute sense. It's only valid in each branch, but you can 
hop from one to another branch such that all your memories are swapped 
with those of your copy. So, if I find myself to be a trillion years old 
and I go top sleep then I'll most likely wake up as a copy that has that 
information about me being a trillion years old replaced by a more 
reasonable lower number.


This also means that even if you win the lottery and become a multi 
millionaire, you'll very likely end up forgetting this and finding 
yourself to not having won the lottery.


Saibal

--
You received this message because you are subscribed to the Google Groups 
"Everything List" group.
To unsubscribe from this group and stop receiving emails from it, send an email 
to everything-list+unsubscr...@googlegroups.com.
To view this discussion on the web visit 
https://groups.google.com/d/msgid/everything-list/cf75a1916068a374012bccbc76e7d237%40zonnet.nl.


Re: Quantum immortality

2019-09-13 Thread John Clark
On Fri, Sep 13, 2019 at 2:14 AM Quentin Anciaux  wrote:

> *Wel if by "dualist soul" you mean something immaterial about our
> consciousness (like I don't know information) can be duplicated then yes it
> is dualist and any computational theory of mind is dualist in this sense
> then.*


Yes I agree completely. I'm a dualist but there is nothing mystical about
that, I just believe that nouns and adjectives refer to different things,
nouns refer to physical objects and adjectives refer to nouns.

> *I suppose you would say that if someone believe "he" can be copied and
> uploaded in a virtual environment then he is a dualist *


I agree again. A standard dictionary will say Information is a noun but I
think that demonstrates an inconsistency in human language because adjectives
describe nouns and so does information. In the case of uploads information
describes the way generic atoms are arranged, and this time I agree with
the dictionary, atoms are nouns. Arrange atoms one way and they're me,
arrange those exact same atoms another way and they're you.

 John K Clark

-- 
You received this message because you are subscribed to the Google Groups 
"Everything List" group.
To unsubscribe from this group and stop receiving emails from it, send an email 
to everything-list+unsubscr...@googlegroups.com.
To view this discussion on the web visit 
https://groups.google.com/d/msgid/everything-list/CAJPayv2DGXZzQfdH4wxwO0YXMaB7SAOP12XoK2C7oimWPskxdQ%40mail.gmail.com.


Re: Quantum immortality

2019-09-13 Thread Quentin Anciaux
Le ven. 13 sept. 2019 à 13:21, Bruce Kellett  a
écrit :

> On Fri, Sep 13, 2019 at 9:18 PM Quentin Anciaux 
> wrote:
>
>> Le ven. 13 sept. 2019 à 13:16, Bruce Kellett  a
>> écrit :
>>
>>>
>>> I don't get Mallah's point here, either. I will have to look more
>>> clearly at his argument against QS. I don't think that case is a clear-cut
>>> as for QI. The fact that I am not the oldest person around is clear
>>> evidence against QI.
>>>
>>
>> It's wrong, that imply you can nerver have been young.
>>
>
> That does not follow.
>
> It does, because if it does the fact you're not the oldest person means
nothing, because *every year* before being old you had the possibilité to
make the same observation and as it's mandatory to lives those before being
old, you can't conclude what you do.


> Bruce
>
-- 
> You received this message because you are subscribed to the Google Groups
> "Everything List" group.
> To unsubscribe from this group and stop receiving emails from it, send an
> email to everything-list+unsubscr...@googlegroups.com.
> To view this discussion on the web visit
> https://groups.google.com/d/msgid/everything-list/CAFxXSLThfTBw75JX3QK5iSaqCeOag0B%2Bk4c1ECWGTCTUxVr2Ow%40mail.gmail.com
> 
> .
>


-- 
All those moments will be lost in time, like tears in rain. (Roy
Batty/Rutger Hauer)

-- 
You received this message because you are subscribed to the Google Groups 
"Everything List" group.
To unsubscribe from this group and stop receiving emails from it, send an email 
to everything-list+unsubscr...@googlegroups.com.
To view this discussion on the web visit 
https://groups.google.com/d/msgid/everything-list/CAMW2kArOgu4X4e2Re8fBaVYQX6iQYY_0Pxv2imLv_UxtQYDz2g%40mail.gmail.com.


Re: Quantum immortality

2019-09-13 Thread Bruce Kellett
On Fri, Sep 13, 2019 at 9:18 PM Quentin Anciaux  wrote:

> Le ven. 13 sept. 2019 à 13:16, Bruce Kellett  a
> écrit :
>
>>
>> I don't get Mallah's point here, either. I will have to look more clearly
>> at his argument against QS. I don't think that case is a clear-cut as for
>> QI. The fact that I am not the oldest person around is clear evidence
>> against QI.
>>
>
> It's wrong, that imply you can nerver have been young.
>

That does not follow.

Bruce

-- 
You received this message because you are subscribed to the Google Groups 
"Everything List" group.
To unsubscribe from this group and stop receiving emails from it, send an email 
to everything-list+unsubscr...@googlegroups.com.
To view this discussion on the web visit 
https://groups.google.com/d/msgid/everything-list/CAFxXSLThfTBw75JX3QK5iSaqCeOag0B%2Bk4c1ECWGTCTUxVr2Ow%40mail.gmail.com.


Re: Quantum immortality

2019-09-13 Thread Quentin Anciaux
Le ven. 13 sept. 2019 à 13:16, Bruce Kellett  a
écrit :

> On Fri, Sep 13, 2019 at 8:49 PM Bruno Marchal  wrote:
>
>> On 12 Sep 2019, at 01:50, Bruce Kellett  wrote:
>>
>> On Thu, Sep 12, 2019 at 1:55 AM Bruno Marchal  wrote:
>>
>>> On 11 Sep 2019, at 01:30, Bruce Kellett 
>>> wrote:
>>>
>>> From: Bruno Marchal 
>>>
>>> On 8 Sep 2019, at 13:59, Bruce Kellett  wrote:
>>>
>>> If the only relevance you can find for many worlds is quantum
>>> immortality, then many worlds is indeed dead. Quantum immortality has been
>>> shown many times to be a complete nonsense.
>>>
>>>
>>> Really. I did not known that. Could you give the references.
>>>
>>> Follow the Wikipedia entry on quantum suicide.
>>>
>>> That is not what I mean by a  reference.
>>>
>>
>> I later gave a reference to the paper by Mallah -- whom you know of,
>> apparently. The paper is available at
>>
>> https://arxiv.org/abs/0902.0187
>>
>>
>>
>>
>> Yes, the oldest participant in this list have know Jacques Mallah, who
>> participated a lot in this list.
>>
>> Mallah is wrong here:
>>
>> <<
>> Max Tegmark publicized the QS idea, but in some ways he is more of a
>> moderate on the issue than most of its believers are. If he were to follow
>> in the footsteps of Don Page and alter his views, recanting belief in QS,
>> it would be a great help in exposing the belief as a fallacy, and I hold
>> out hope that it is possible that he will do so.
>>
>> In his paper [Tegmark 1] QS is explained as follows:
>>
>> “Since there is exactly one observer having perceptions both before and
>> after the trigger event, and since it occurred too fast to notice, the MWI
>> prediction is that” (the experimenter) “will hear “click” with 100%
>> certainty.”
>>
>> That is a rather odd statement because he is certainly aware that in the
>> MWI there is no sense in which it can be rightfully said that “there is
>> exactly one observer” either before
>>
>> [image: page13image25488]
>> or after the experiment. The ket notation may be unhelpful here; indeed,
>> if the tensor product of kets on the left hand side were expanded instead
>> of factoring out the observer, there would appear to have been “two
>> observers” initially.
>> >>
>>
>
> I don't get Mallah's point here, either. I will have to look more clearly
> at his argument against QS. I don't think that case is a clear-cut as for
> QI. The fact that I am not the oldest person around is clear evidence
> against QI.
>

It's wrong, that imply you can nerver have been young.


>
> Bruce
>
> Two different brains doing the same computation gives only one subjective
>> first person.
>>
>> Bruno
>>
> --
> You received this message because you are subscribed to the Google Groups
> "Everything List" group.
> To unsubscribe from this group and stop receiving emails from it, send an
> email to everything-list+unsubscr...@googlegroups.com.
> To view this discussion on the web visit
> https://groups.google.com/d/msgid/everything-list/CAFxXSLQQZ9Ai1GL6-q0wGJjLMU49XG-Oj0JSpqD%3DS-VBU4PjFw%40mail.gmail.com
> <https://groups.google.com/d/msgid/everything-list/CAFxXSLQQZ9Ai1GL6-q0wGJjLMU49XG-Oj0JSpqD%3DS-VBU4PjFw%40mail.gmail.com?utm_medium=email&utm_source=footer>
> .
>


-- 
All those moments will be lost in time, like tears in rain. (Roy
Batty/Rutger Hauer)

-- 
You received this message because you are subscribed to the Google Groups 
"Everything List" group.
To unsubscribe from this group and stop receiving emails from it, send an email 
to everything-list+unsubscr...@googlegroups.com.
To view this discussion on the web visit 
https://groups.google.com/d/msgid/everything-list/CAMW2kAoAsJQbOTV5t7EW4OmOeuiN%2BPo_Ohr5u6ApiwjybNUVrQ%40mail.gmail.com.


Re: Quantum immortality

2019-09-13 Thread Bruce Kellett
On Fri, Sep 13, 2019 at 8:49 PM Bruno Marchal  wrote:

> On 12 Sep 2019, at 01:50, Bruce Kellett  wrote:
>
> On Thu, Sep 12, 2019 at 1:55 AM Bruno Marchal  wrote:
>
>> On 11 Sep 2019, at 01:30, Bruce Kellett 
>> wrote:
>>
>> From: Bruno Marchal 
>>
>> On 8 Sep 2019, at 13:59, Bruce Kellett  wrote:
>>
>> If the only relevance you can find for many worlds is quantum
>> immortality, then many worlds is indeed dead. Quantum immortality has been
>> shown many times to be a complete nonsense.
>>
>>
>> Really. I did not known that. Could you give the references.
>>
>> Follow the Wikipedia entry on quantum suicide.
>>
>> That is not what I mean by a  reference.
>>
>
> I later gave a reference to the paper by Mallah -- whom you know of,
> apparently. The paper is available at
>
> https://arxiv.org/abs/0902.0187
>
>
>
>
> Yes, the oldest participant in this list have know Jacques Mallah, who
> participated a lot in this list.
>
> Mallah is wrong here:
>
> <<
> Max Tegmark publicized the QS idea, but in some ways he is more of a
> moderate on the issue than most of its believers are. If he were to follow
> in the footsteps of Don Page and alter his views, recanting belief in QS,
> it would be a great help in exposing the belief as a fallacy, and I hold
> out hope that it is possible that he will do so.
>
> In his paper [Tegmark 1] QS is explained as follows:
>
> “Since there is exactly one observer having perceptions both before and
> after the trigger event, and since it occurred too fast to notice, the MWI
> prediction is that” (the experimenter) “will hear “click” with 100%
> certainty.”
>
> That is a rather odd statement because he is certainly aware that in the
> MWI there is no sense in which it can be rightfully said that “there is
> exactly one observer” either before
>
> [image: page13image25488]
> or after the experiment. The ket notation may be unhelpful here; indeed,
> if the tensor product of kets on the left hand side were expanded instead
> of factoring out the observer, there would appear to have been “two
> observers” initially.
> >>
>

I don't get Mallah's point here, either. I will have to look more clearly
at his argument against QS. I don't think that case is a clear-cut as for
QI. The fact that I am not the oldest person around is clear evidence
against QI.

Bruce

Two different brains doing the same computation gives only one subjective
> first person.
>
> Bruno
>

-- 
You received this message because you are subscribed to the Google Groups 
"Everything List" group.
To unsubscribe from this group and stop receiving emails from it, send an email 
to everything-list+unsubscr...@googlegroups.com.
To view this discussion on the web visit 
https://groups.google.com/d/msgid/everything-list/CAFxXSLQQZ9Ai1GL6-q0wGJjLMU49XG-Oj0JSpqD%3DS-VBU4PjFw%40mail.gmail.com.


Re: Quantum immortality

2019-09-13 Thread Bruno Marchal

> On 12 Sep 2019, at 01:50, Bruce Kellett  wrote:
> 
> On Thu, Sep 12, 2019 at 1:55 AM Bruno Marchal  <mailto:marc...@ulb.ac.be>> wrote:
> On 11 Sep 2019, at 01:30, Bruce Kellett  <mailto:bhkell...@optusnet.com.au>> wrote:
>> From: Bruno Marchal mailto:marc...@ulb.ac.be>>
>>>> On 8 Sep 2019, at 13:59, Bruce Kellett >>> <mailto:bhkellet...@gmail.com>> wrote:
>>>> 
>>>> If the only relevance you can find for many worlds is quantum immortality, 
>>>> then many worlds is indeed dead. Quantum immortality has been shown many 
>>>> times to be a complete nonsense.
>>> 
>>> Really. I did not known that. Could you give the references.
>> Follow the Wikipedia entry on quantum suicide.
>> 
> That is not what I mean by a  reference.
> 
> I later gave a reference to the paper by Mallah -- whom you know of, 
> apparently. The paper is available at
> 
> https://arxiv.org/abs/0902.0187 <https://arxiv.org/abs/0902.0187>
> 
> 


Yes, the oldest participant in this list have know Jacques Mallah, who 
participated a lot in this list.

Mallah is wrong here:

<<
Max Tegmark publicized the QS idea, but in some ways he is more of a moderate 
on the issue than most of its believers are. If he were to follow in the 
footsteps of Don Page and alter his views, recanting belief in QS, it would be 
a great help in exposing the belief as a fallacy, and I hold out hope that it 
is possible that he will do so.

In his paper [Tegmark 1] QS is explained as follows:

“Since there is exactly one observer having perceptions both before and after 
the trigger event, and since it occurred too fast to notice, the MWI prediction 
is that” (the experimenter) “will hear “click” with 100% certainty.”

That is a rather odd statement because he is certainly aware that in the MWI 
there is no sense in which it can be rightfully said that “there is exactly one 
observer” either before


or after the experiment. The ket notation may be unhelpful here; indeed, if the 
tensor product of kets on the left hand side were expanded instead of factoring 
out the observer, there would appear to have been “two observers” initially.
>>

Two different brains doing the same computation gives only one subjective first 
person.

Bruno




> 
> [.]
> 
>  
>> None of this has anything to do with wave-packet reduction, so you can rest 
>> easy.
>> 
> 
> You lost me here. With the wave reduction, there is just no quantum 
> immortality at all, nor even quantum suicide. I guess I mess something.
> 
> The argument was that QI makes no sense, even in a many-worlds setting.
>  
> The only “reasonable” critics was the one done by Jacques Mallah on this 
> list, which claims that if QI or MI is correct, we should expect to be very 
> old. But Quentin answered this validly: we expect in all situation to be just 
> a bit older than where we remember coming from, and the paradox comes from a 
> confusing between relative and absolute self-sampling on the states or 
> histories.
> 
> The trouble with this is that neither ASSA and RSSA is a law of nature. As I 
> have said, from the 1p perspective, I live more years between 100 and 1000 
> than between 1 and 100. So I expect to be very old. What we remember is 
> actually irrelevant -- we can always check our birth certificate if we forget 
> how old we are. In other words, we can use external sources to refresh 
> memories. What we personally remember at any instant is variable and 
> unreliable. Check against external references.
>  
> Typically, also, old and young are not absolute concept.
> 
> No, they are concepts relative to actual life span -- you are always at your 
> youngest when you are born, and at your oldest just before you die.
> 
> With mechanism or quantum mechanics without collapse, we can say that we are 
> always young.
> 
> Another good reason for abandoning mechanism.
> 
> Bruce 
> 
> -- 
> You received this message because you are subscribed to the Google Groups 
> "Everything List" group.
> To unsubscribe from this group and stop receiving emails from it, send an 
> email to everything-list+unsubscr...@googlegroups.com 
> <mailto:everything-list+unsubscr...@googlegroups.com>.
> To view this discussion on the web visit 
> https://groups.google.com/d/msgid/everything-list/CAFxXSLTM7c97pmzXq5Fb%3D1nq%2BbdxfkqojZn%3DMXsuvhC%2BmLfy8A%40mail.gmail.com
>  
> <https://groups.google.com/d/msgid/everything-list/CAFxXSLTM7c97pmzXq5Fb%3D1nq%2BbdxfkqojZn%3DMXsuvhC%2BmLfy8A%40mail.gmail.com?utm_medium=email&utm_source=footer>.

-- 
You received this message because you are subscribed to the Google Groups 
"Everything List" group.
To unsubscribe from this group and stop receiving emails from it, send an email 
to everything-list+unsubscr...@googlegroups.com.
To view this discussion on the web visit 
https://groups.google.com/d/msgid/everything-list/674E7588-3DA3-4690-9652-63ECCE063476%40ulb.ac.be.


Re: Quantum immortality

2019-09-13 Thread Bruno Marchal


> On 11 Sep 2019, at 16:51, smitra  wrote:
> 
> Back to basics. There exists a universal wavefunction that evolves according 
> to the Schrodinger equation. Observers are internal structures in this 
> description. Whether or not one believes that the Born rule can be derived or 
> not, what matters in practice is that you'll end up having to use it, so you 
> have to assign a measure for observations that is given by the summation of 
> the squared modulus of the states that correspond to those observations. The 
> information about personal identity must then also be extracted from the 
> wavefunction, so one cannot insert this in an ad hoc way.
> 
> Quantum immortality is therefore wrong because the measure of the states that 
> correspond to extremely old observers is small.


The same reasoning would apply to “quantum suicide”, where it is clear that we 
survive all the time; given that we cannot take into account the world where we 
do not.

If in H you are multiplied in W and M, but directly killed in M, you survive in 
W with probability one. That is why we add p or <>t to []p to transform the 
logic of belief ([]p) into a probability logic ([]p & <>t).

Bruno




> 
> 
> Saibal
> 
> -- 
> You received this message because you are subscribed to the Google Groups 
> "Everything List" group.
> To unsubscribe from this group and stop receiving emails from it, send an 
> email to everything-list+unsubscr...@googlegroups.com.
> To view this discussion on the web visit 
> https://groups.google.com/d/msgid/everything-list/ee05eb58fca14c1114ba1fabfc5d26e3%40zonnet.nl.

-- 
You received this message because you are subscribed to the Google Groups 
"Everything List" group.
To unsubscribe from this group and stop receiving emails from it, send an email 
to everything-list+unsubscr...@googlegroups.com.
To view this discussion on the web visit 
https://groups.google.com/d/msgid/everything-list/02446B1E-4C15-4528-AED3-8DD1CA1C6FA7%40ulb.ac.be.


Re: Quantum immortality

2019-09-13 Thread Stathis Papaioannou
On Fri, 13 Sep 2019 at 14:49, Bruce Kellett  wrote:

> On Fri, Sep 13, 2019 at 1:41 PM Stathis Papaioannou 
> wrote:
>
>> On Thu, 12 Sep 2019 at 09:38, Bruce Kellett 
>> wrote:
>>
>>> On Thu, Sep 12, 2019 at 2:55 AM Jason Resch 
>>> wrote:
>>>
>>>> On Tuesday, September 10, 2019, Bruce Kellett 
>>>> wrote:
>>>>
>>>>> On Wed, Sep 11, 2019 at 10:18 AM 'Brent Meeker' via Everything List <
>>>>> everything-list@googlegroups.com> wrote:
>>>>>
>>>>>> On 9/10/2019 4:30 PM, Bruce Kellett wrote:
>>>>>> > Another argument that has been given here before is that if quantum
>>>>>> > immortality is true, then we should expect to see a number of
>>>>>> people
>>>>>> > who are considerably older than the normal life expectancy -- and
>>>>>> we
>>>>>> > do not see people who are two or three hundred years old. Even if
>>>>>> the
>>>>>> > probabilities are very low, there have been an awful lot of people
>>>>>> > born within the last 500 or so years -- some must have survived on
>>>>>> our
>>>>>> > branch if this scenario is true.
>>>>>>
>>>>>> My argument was that each of us should find ourselves to be much
>>>>>> older
>>>>>> than even the oldest people we know.
>>>>>
>>>>>
>>>>> That is probably the best single argument against quantum immortality:
>>>>> if QI is true, then the measure of our lifetime after one reaches a normal
>>>>> lifetime is infinitely greater than the measure before age , say, 120 yr.
>>>>> So if one finds oneself younger than 120 years, QI is false, and if MWI is
>>>>> still considered to be true, there must be another argument why MWI does
>>>>> not imply QI.
>>>>>
>>>>
>>>>
>>>> Why do you think that measure only increases with age? On an objective
>>>> level it only decreases.
>>>>
>>>
>>> As Bruno would say, "you confuse the 1p with the 1pp." I am talking
>>> about my personal measure of the number of years I have lived. As I get
>>> older, the number of years I have lived increases. If I live to 1000, I
>>> have lived more years between 100 and 1000 than between 1 and 100. This is
>>> arithmetic, after all.
>>>
>>> But this discussion has gone off the rails. It started as a discussion
>>> of quantum immortality, and the arguments against this notion, even in MWI.
>>> The arguments against QI that have been advanced are that life-threatening
>>> events tend not to be binary or quantum, but rather we enter a period of
>>> slow decline, due to illness or other factors. Consequently, there is no
>>> reason for us to expect to be immortal, even in MWI. The other argument is
>>> that if QI is true, then you would expect to be very old. This argument was
>>> advanced by Mallah (arXiv: 0905.0187) and has not been satisfactorily
>>> rebutted.
>>>
>>
>> It is not simple arithmetic if you live to be very old that most of your
>> measure is in your older years if you take into account all the copies.
>> Suppose there are 10^100 copies of you under 100 years old and then all but
>> one copy dies, but that one copy goes on to live to 1000. If you did not
>> know how old you were and you had to guess given this information, then you
>> would guess with near certainty that you were under 100 years old. However,
>> you would also know with certainty that you would live to 1000, and you
>> would not notice anything weird happening as you approached your 100th
>> birthday.
>>
>
> The trouble with this argument is that you know that at least one copy of
> you will survive past 100 years (or past any age, for that matter). Given
> that you survive, the probability of survival is one. Taking account of all
> the other copies who die does not alter this fact. If you are all your
> copies, then your probability of survival under the assumption of QI is
> always one.
>
> Your RSSA assumption is effectively a dualist model -- there is only one
> soul that makes you really you, and that soul goes at random into one and
> only one copy at any time. Then the chances that this soul-containing copy
> is the one that survives, does indeed decrease rapidly with age. But that
> is the wrong way to look at it -- there 

Re: Quantum immortality

2019-09-12 Thread Bruce Kellett
On Fri, Sep 13, 2019 at 4:14 PM Quentin Anciaux  wrote:

> Le ven. 13 sept. 2019 à 08:03, Bruce Kellett  a
> écrit :
>
>>
>> Yes. QI is possible only in a many-worlds scenario, but that does not
>> necessarily mean that any many-worlds scenario implies QI. As you say, most
>> of life's significant events are quasi-classical in origin, not decoherence
>> amplified quantum events. One could be "copied" by quantum events that are
>> irrelevant to your survival. For example, by the splitting of worlds
>> occasioned by someone shining a laser on a half-silvered mirror. That just
>> increases the number of copies, all of which probably survive that
>> splitting process.
>>
>> As another aside, it seems to me that Stathis's RSSA, with measure along
>> the lifeline decreasing in accordance with Born's rule (as Russell Standish
>> puts it in his book "Theory of Nothing"), is equivalent to a collapse model
>> (or the existence of a dualist "soul" as I put it before).
>>
>
> Wel if by "dualist soul" you mean something immaterial about our
> consciousness (like I don't know information) can be duplicated then yes it
> is dualist and any computational theory of mind is dualist in this sense
> then. I suppose you would say that if someone believe "he" can be copied
> and uploaded in a virtual environment then he is a dualist ?
>

That is not implied by anything I said. If the measure along your lifeline
decreases according to the Born rule, as Russell suggests, then one is
effectively saying that after quantum duplication, the soul chooses one
copy at random, and that is the one that is "you". This is effectively a
collapse model -- all duplicates except the one randomly chosen cease to be
you. I am certainly not advocating such a position, I was just pointing out
that that is what RSSA amounts to.

Bruce

-- 
You received this message because you are subscribed to the Google Groups 
"Everything List" group.
To unsubscribe from this group and stop receiving emails from it, send an email 
to everything-list+unsubscr...@googlegroups.com.
To view this discussion on the web visit 
https://groups.google.com/d/msgid/everything-list/CAFxXSLR81qLRcVLsJDDpqjap_20SH6Lbdxk1PZWPhvHFjQVSYw%40mail.gmail.com.


Re: Quantum immortality

2019-09-12 Thread Quentin Anciaux
Le ven. 13 sept. 2019 à 08:03, Bruce Kellett  a
écrit :

> On Fri, Sep 13, 2019 at 3:07 PM 'Brent Meeker' via Everything List <
> everything-list@googlegroups.com> wrote:
>
>> On 9/12/2019 9:49 PM, Bruce Kellett wrote:
>>
>> On Fri, Sep 13, 2019 at 1:41 PM Stathis Papaioannou 
>> wrote:
>>
>>> On Thu, 12 Sep 2019 at 09:38, Bruce Kellett 
>>> wrote:
>>>
>>>> On Thu, Sep 12, 2019 at 2:55 AM Jason Resch 
>>>> wrote:
>>>>
>>>>> On Tuesday, September 10, 2019, Bruce Kellett 
>>>>> wrote:
>>>>>
>>>>>> On Wed, Sep 11, 2019 at 10:18 AM 'Brent Meeker' via Everything List <
>>>>>> everything-list@googlegroups.com> wrote:
>>>>>>
>>>>>>> On 9/10/2019 4:30 PM, Bruce Kellett wrote:
>>>>>>> > Another argument that has been given here before is that if
>>>>>>> quantum
>>>>>>> > immortality is true, then we should expect to see a number of
>>>>>>> people
>>>>>>> > who are considerably older than the normal life expectancy -- and
>>>>>>> we
>>>>>>> > do not see people who are two or three hundred years old. Even if
>>>>>>> the
>>>>>>> > probabilities are very low, there have been an awful lot of people
>>>>>>> > born within the last 500 or so years -- some must have survived on
>>>>>>> our
>>>>>>> > branch if this scenario is true.
>>>>>>>
>>>>>>> My argument was that each of us should find ourselves to be much
>>>>>>> older
>>>>>>> than even the oldest people we know.
>>>>>>
>>>>>>
>>>>>> That is probably the best single argument against quantum
>>>>>> immortality: if QI is true, then the measure of our lifetime after one
>>>>>> reaches a normal lifetime is infinitely greater than the measure before 
>>>>>> age
>>>>>> , say, 120 yr. So if one finds oneself younger than 120 years, QI is 
>>>>>> false,
>>>>>> and if MWI is still considered to be true, there must be another argument
>>>>>> why MWI does not imply QI.
>>>>>>
>>>>>
>>>>>
>>>>> Why do you think that measure only increases with age? On an objective
>>>>> level it only decreases.
>>>>>
>>>>
>>>> As Bruno would say, "you confuse the 1p with the 1pp." I am talking
>>>> about my personal measure of the number of years I have lived. As I get
>>>> older, the number of years I have lived increases. If I live to 1000, I
>>>> have lived more years between 100 and 1000 than between 1 and 100. This is
>>>> arithmetic, after all.
>>>>
>>>> But this discussion has gone off the rails. It started as a discussion
>>>> of quantum immortality, and the arguments against this notion, even in MWI.
>>>> The arguments against QI that have been advanced are that life-threatening
>>>> events tend not to be binary or quantum, but rather we enter a period of
>>>> slow decline, due to illness or other factors. Consequently, there is no
>>>> reason for us to expect to be immortal, even in MWI. The other argument is
>>>> that if QI is true, then you would expect to be very old. This argument was
>>>> advanced by Mallah (arXiv: 0905.0187) and has not been satisfactorily
>>>> rebutted.
>>>>
>>>
>>> It is not simple arithmetic if you live to be very old that most of your
>>> measure is in your older years if you take into account all the copies.
>>> Suppose there are 10^100 copies of you under 100 years old and then all but
>>> one copy dies, but that one copy goes on to live to 1000. If you did not
>>> know how old you were and you had to guess given this information, then you
>>> would guess with near certainty that you were under 100 years old. However,
>>> you would also know with certainty that you would live to 1000, and you
>>> would not notice anything weird happening as you approached your 100th
>>> birthday.
>>>
>>
>> The trouble with this argument is that you know that at least one copy of
>> you will survive past 100 years (or past any age, for that matter). Given
>> that you survive,

Re: Quantum immortality

2019-09-12 Thread Bruce Kellett
On Fri, Sep 13, 2019 at 3:07 PM 'Brent Meeker' via Everything List <
everything-list@googlegroups.com> wrote:

> On 9/12/2019 9:49 PM, Bruce Kellett wrote:
>
> On Fri, Sep 13, 2019 at 1:41 PM Stathis Papaioannou 
> wrote:
>
>> On Thu, 12 Sep 2019 at 09:38, Bruce Kellett 
>> wrote:
>>
>>> On Thu, Sep 12, 2019 at 2:55 AM Jason Resch 
>>> wrote:
>>>
>>>> On Tuesday, September 10, 2019, Bruce Kellett 
>>>> wrote:
>>>>
>>>>> On Wed, Sep 11, 2019 at 10:18 AM 'Brent Meeker' via Everything List <
>>>>> everything-list@googlegroups.com> wrote:
>>>>>
>>>>>> On 9/10/2019 4:30 PM, Bruce Kellett wrote:
>>>>>> > Another argument that has been given here before is that if quantum
>>>>>> > immortality is true, then we should expect to see a number of
>>>>>> people
>>>>>> > who are considerably older than the normal life expectancy -- and
>>>>>> we
>>>>>> > do not see people who are two or three hundred years old. Even if
>>>>>> the
>>>>>> > probabilities are very low, there have been an awful lot of people
>>>>>> > born within the last 500 or so years -- some must have survived on
>>>>>> our
>>>>>> > branch if this scenario is true.
>>>>>>
>>>>>> My argument was that each of us should find ourselves to be much
>>>>>> older
>>>>>> than even the oldest people we know.
>>>>>
>>>>>
>>>>> That is probably the best single argument against quantum immortality:
>>>>> if QI is true, then the measure of our lifetime after one reaches a normal
>>>>> lifetime is infinitely greater than the measure before age , say, 120 yr.
>>>>> So if one finds oneself younger than 120 years, QI is false, and if MWI is
>>>>> still considered to be true, there must be another argument why MWI does
>>>>> not imply QI.
>>>>>
>>>>
>>>>
>>>> Why do you think that measure only increases with age? On an objective
>>>> level it only decreases.
>>>>
>>>
>>> As Bruno would say, "you confuse the 1p with the 1pp." I am talking
>>> about my personal measure of the number of years I have lived. As I get
>>> older, the number of years I have lived increases. If I live to 1000, I
>>> have lived more years between 100 and 1000 than between 1 and 100. This is
>>> arithmetic, after all.
>>>
>>> But this discussion has gone off the rails. It started as a discussion
>>> of quantum immortality, and the arguments against this notion, even in MWI.
>>> The arguments against QI that have been advanced are that life-threatening
>>> events tend not to be binary or quantum, but rather we enter a period of
>>> slow decline, due to illness or other factors. Consequently, there is no
>>> reason for us to expect to be immortal, even in MWI. The other argument is
>>> that if QI is true, then you would expect to be very old. This argument was
>>> advanced by Mallah (arXiv: 0905.0187) and has not been satisfactorily
>>> rebutted.
>>>
>>
>> It is not simple arithmetic if you live to be very old that most of your
>> measure is in your older years if you take into account all the copies.
>> Suppose there are 10^100 copies of you under 100 years old and then all but
>> one copy dies, but that one copy goes on to live to 1000. If you did not
>> know how old you were and you had to guess given this information, then you
>> would guess with near certainty that you were under 100 years old. However,
>> you would also know with certainty that you would live to 1000, and you
>> would not notice anything weird happening as you approached your 100th
>> birthday.
>>
>
> The trouble with this argument is that you know that at least one copy of
> you will survive past 100 years (or past any age, for that matter). Given
> that you survive, the probability of survival is one. Taking account of all
> the other copies who die does not alter this fact. If you are all your
> copies, then your probability of survival under the assumption of QI is
> always one.
>
> Your RSSA assumption is effectively a dualist model -- there is only one
> soul that makes you really you, and that soul goes at random into one and
> only one copy at any time. Then the chances that this soul-containing cop

Re: Quantum immortality

2019-09-12 Thread 'Brent Meeker' via Everything List



On 9/12/2019 9:49 PM, Bruce Kellett wrote:
On Fri, Sep 13, 2019 at 1:41 PM Stathis Papaioannou 
mailto:stath...@gmail.com>> wrote:


On Thu, 12 Sep 2019 at 09:38, Bruce Kellett mailto:bhkellet...@gmail.com>> wrote:

On Thu, Sep 12, 2019 at 2:55 AM Jason Resch
mailto:jasonre...@gmail.com>> wrote:

On Tuesday, September 10, 2019, Bruce Kellett
mailto:bhkellet...@gmail.com>> wrote:

On Wed, Sep 11, 2019 at 10:18 AM 'Brent Meeker' via
Everything List mailto:everything-list@googlegroups.com>> wrote:

On 9/10/2019 4:30 PM, Bruce Kellett wrote:
> Another argument that has been given here before
    is that if quantum
> immortality is true, then we should expect to
see a number of people
> who are considerably older than the normal life
expectancy -- and we
> do not see people who are two or three hundred
years old. Even if the
> probabilities are very low, there have been an
awful lot of people
> born within the last 500 or so years -- some
must have survived on our
> branch if this scenario is true.

My argument was that each of us should find
ourselves to be much older
than even the oldest people we know.

That is probably the best single argument against
quantum immortality: if QI is true, then the measure
of our lifetime after one reaches a normal lifetime is
infinitely greater than the measure before age , say,
120 yr. So if one finds oneself younger than 120
years, QI is false, and if MWI is still considered to
be true, there must be another argument why MWI does
not imply QI.



Why do you think that measure only increases with age? On
an objective level it only decreases.


As Bruno would say, "you confuse the 1p with the 1pp." I am
talking about my personal measure of the number of years I
have lived. As I get older, the number of years I have lived
increases. If I live to 1000, I have lived more years between
100 and 1000 than between 1 and 100. This is arithmetic, after
all.

But this discussion has gone off the rails. It started as a
discussion of quantum immortality, and the arguments against
this notion, even in MWI. The arguments against QI that have
been advanced are that life-threatening events tend not to be
binary or quantum, but rather we enter a period of slow
decline, due to illness or other factors. Consequently, there
is no reason for us to expect to be immortal, even in MWI. The
other argument is that if QI is true, then you would expect to
be very old. This argument was advanced by Mallah (arXiv:
0905.0187) and has not been satisfactorily rebutted.


It is not simple arithmetic if you live to be very old that most
of your measure is in your older years if you take into account
all the copies. Suppose there are 10^100 copies of you under 100
years old and then all but one copy dies, but that one copy goes
on to live to 1000. If you did not know how old you were and you
had to guess given this information, then you would guess with
near certainty that you were under 100 years old. However, you
would also know with certainty that you would live to 1000, and
you would not notice anything weird happening as you approached
your 100th birthday.


The trouble with this argument is that you know that at least one copy 
of you will survive past 100 years (or past any age, for that matter). 
Given that you survive, the probability of survival is one. Taking 
account of all the other copies who die does not alter this fact. If 
you are all your copies, then your probability of survival under the 
assumption of QI is always one.


Your RSSA assumption is effectively a dualist model -- there is only 
one soul that makes you really you, and that soul goes at random into 
one and only one copy at any time. Then the chances that this 
soul-containing copy is the one that survives, does indeed decrease 
rapidly with age. But that is the wrong way to look at it -- there is 
no 'soul' that makes a copy you. On the MWI assumptions, every copy is 
'you', so since at least one copy always survives, 'you' will always 
survive. The number of years you survive past age 100 is indefinitely 
large, so you spend more time in those years, and you have probability 
one of getting there.


As I un

Re: Quantum immortality

2019-09-12 Thread Bruce Kellett
On Fri, Sep 13, 2019 at 1:41 PM Stathis Papaioannou 
wrote:

> On Thu, 12 Sep 2019 at 09:38, Bruce Kellett  wrote:
>
>> On Thu, Sep 12, 2019 at 2:55 AM Jason Resch  wrote:
>>
>>> On Tuesday, September 10, 2019, Bruce Kellett 
>>> wrote:
>>>
>>>> On Wed, Sep 11, 2019 at 10:18 AM 'Brent Meeker' via Everything List <
>>>> everything-list@googlegroups.com> wrote:
>>>>
>>>>> On 9/10/2019 4:30 PM, Bruce Kellett wrote:
>>>>> > Another argument that has been given here before is that if quantum
>>>>> > immortality is true, then we should expect to see a number of people
>>>>> > who are considerably older than the normal life expectancy -- and we
>>>>> > do not see people who are two or three hundred years old. Even if
>>>>> the
>>>>> > probabilities are very low, there have been an awful lot of people
>>>>> > born within the last 500 or so years -- some must have survived on
>>>>> our
>>>>> > branch if this scenario is true.
>>>>>
>>>>> My argument was that each of us should find ourselves to be much older
>>>>> than even the oldest people we know.
>>>>
>>>>
>>>> That is probably the best single argument against quantum immortality:
>>>> if QI is true, then the measure of our lifetime after one reaches a normal
>>>> lifetime is infinitely greater than the measure before age , say, 120 yr.
>>>> So if one finds oneself younger than 120 years, QI is false, and if MWI is
>>>> still considered to be true, there must be another argument why MWI does
>>>> not imply QI.
>>>>
>>>
>>>
>>> Why do you think that measure only increases with age? On an objective
>>> level it only decreases.
>>>
>>
>> As Bruno would say, "you confuse the 1p with the 1pp." I am talking about
>> my personal measure of the number of years I have lived. As I get older,
>> the number of years I have lived increases. If I live to 1000, I have lived
>> more years between 100 and 1000 than between 1 and 100. This is arithmetic,
>> after all.
>>
>> But this discussion has gone off the rails. It started as a discussion of
>> quantum immortality, and the arguments against this notion, even in MWI.
>> The arguments against QI that have been advanced are that life-threatening
>> events tend not to be binary or quantum, but rather we enter a period of
>> slow decline, due to illness or other factors. Consequently, there is no
>> reason for us to expect to be immortal, even in MWI. The other argument is
>> that if QI is true, then you would expect to be very old. This argument was
>> advanced by Mallah (arXiv: 0905.0187) and has not been satisfactorily
>> rebutted.
>>
>
> It is not simple arithmetic if you live to be very old that most of your
> measure is in your older years if you take into account all the copies.
> Suppose there are 10^100 copies of you under 100 years old and then all but
> one copy dies, but that one copy goes on to live to 1000. If you did not
> know how old you were and you had to guess given this information, then you
> would guess with near certainty that you were under 100 years old. However,
> you would also know with certainty that you would live to 1000, and you
> would not notice anything weird happening as you approached your 100th
> birthday.
>

The trouble with this argument is that you know that at least one copy of
you will survive past 100 years (or past any age, for that matter). Given
that you survive, the probability of survival is one. Taking account of all
the other copies who die does not alter this fact. If you are all your
copies, then your probability of survival under the assumption of QI is
always one.

Your RSSA assumption is effectively a dualist model -- there is only one
soul that makes you really you, and that soul goes at random into one and
only one copy at any time. Then the chances that this soul-containing copy
is the one that survives, does indeed decrease rapidly with age. But that
is the wrong way to look at it -- there is no 'soul' that makes a copy you.
On the MWI assumptions, every copy is 'you', so since at least one copy
always survives, 'you' will always survive. The number of years you survive
past age 100 is indefinitely large, so you spend more time in those years,
and you have probability one of getting there.

Bruce

-- 
You received this message because you are subscribed to the Google Groups 
"Everything List" group.
To unsubscribe from this group and stop receiving emails from it, send an email 
to everything-list+unsubscr...@googlegroups.com.
To view this discussion on the web visit 
https://groups.google.com/d/msgid/everything-list/CAFxXSLTBi_o%2B6jtterG_19PHd%2BvuJjSvpsSStXuNZ2GDif-hYw%40mail.gmail.com.


Re: Quantum immortality

2019-09-12 Thread Stathis Papaioannou
On Thu, 12 Sep 2019 at 09:38, Bruce Kellett  wrote:

> On Thu, Sep 12, 2019 at 2:55 AM Jason Resch  wrote:
>
>> On Tuesday, September 10, 2019, Bruce Kellett 
>> wrote:
>>
>>> On Wed, Sep 11, 2019 at 10:18 AM 'Brent Meeker' via Everything List <
>>> everything-list@googlegroups.com> wrote:
>>>
>>>> On 9/10/2019 4:30 PM, Bruce Kellett wrote:
>>>> > Another argument that has been given here before is that if quantum
>>>> > immortality is true, then we should expect to see a number of people
>>>> > who are considerably older than the normal life expectancy -- and we
>>>> > do not see people who are two or three hundred years old. Even if the
>>>> > probabilities are very low, there have been an awful lot of people
>>>> > born within the last 500 or so years -- some must have survived on
>>>> our
>>>> > branch if this scenario is true.
>>>>
>>>> My argument was that each of us should find ourselves to be much older
>>>> than even the oldest people we know.
>>>
>>>
>>> That is probably the best single argument against quantum immortality:
>>> if QI is true, then the measure of our lifetime after one reaches a normal
>>> lifetime is infinitely greater than the measure before age , say, 120 yr.
>>> So if one finds oneself younger than 120 years, QI is false, and if MWI is
>>> still considered to be true, there must be another argument why MWI does
>>> not imply QI.
>>>
>>
>>
>> Why do you think that measure only increases with age? On an objective
>> level it only decreases.
>>
>
> As Bruno would say, "you confuse the 1p with the 1pp." I am talking about
> my personal measure of the number of years I have lived. As I get older,
> the number of years I have lived increases. If I live to 1000, I have lived
> more years between 100 and 1000 than between 1 and 100. This is arithmetic,
> after all.
>
> But this discussion has gone off the rails. It started as a discussion of
> quantum immortality, and the arguments against this notion, even in MWI.
> The arguments against QI that have been advanced are that life-threatening
> events tend not to be binary or quantum, but rather we enter a period of
> slow decline, due to illness or other factors. Consequently, there is no
> reason for us to expect to be immortal, even in MWI. The other argument is
> that if QI is true, then you would expect to be very old. This argument was
> advanced by Mallah (arXiv: 0905.0187) and has not been satisfactorily
> rebutted.
>

It is not simple arithmetic if you live to be very old that most of your
measure is in your older years if you take into account all the copies.
Suppose there are 10^100 copies of you under 100 years old and then all but
one copy dies, but that one copy goes on to live to 1000. If you did not
know how old you were and you had to guess given this information, then you
would guess with near certainty that you were under 100 years old. However,
you would also know with certainty that you would live to 1000, and you
would not notice anything weird happening as you approached your 100th
birthday.

-- 
Stathis Papaioannou

<https://www.avast.com/sig-email?utm_medium=email&utm_source=link&utm_campaign=sig-email&utm_content=webmail>
Virus-free.
www.avast.com
<https://www.avast.com/sig-email?utm_medium=email&utm_source=link&utm_campaign=sig-email&utm_content=webmail>
<#DAB4FAD8-2DD7-40BB-A1B8-4E2AA1F9FDF2>

-- 
You received this message because you are subscribed to the Google Groups 
"Everything List" group.
To unsubscribe from this group and stop receiving emails from it, send an email 
to everything-list+unsubscr...@googlegroups.com.
To view this discussion on the web visit 
https://groups.google.com/d/msgid/everything-list/CAH%3D2ypXHCsjA-Ra7Bi6FHmRt6X0hydPMgi5DqUhaQhZOwq%2B5Sw%40mail.gmail.com.


Re: Quantum immortality

2019-09-11 Thread Bruce Kellett
On Thu, Sep 12, 2019 at 1:55 AM Bruno Marchal  wrote:

> On 11 Sep 2019, at 01:30, Bruce Kellett  wrote:
>
> From: Bruno Marchal 
>
> On 8 Sep 2019, at 13:59, Bruce Kellett  wrote:
>
> If the only relevance you can find for many worlds is quantum immortality,
> then many worlds is indeed dead. Quantum immortality has been shown many
> times to be a complete nonsense.
>
>
> Really. I did not known that. Could you give the references.
>
> Follow the Wikipedia entry on quantum suicide.
>
> That is not what I mean by a  reference.
>

I later gave a reference to the paper by Mallah -- whom you know of,
apparently. The paper is available at

https://arxiv.org/abs/0902.0187



[.]



> None of this has anything to do with wave-packet reduction, so you can
> rest easy.
>
>
> You lost me here. With the wave reduction, there is just no quantum
> immortality at all, nor even quantum suicide. I guess I mess something.
>

The argument was that QI makes no sense, even in a many-worlds setting.


> The only “reasonable” critics was the one done by Jacques Mallah on this
> list, which claims that if QI or MI is correct, we should expect to be very
> old. But Quentin answered this validly: we expect in all situation to be
> just a bit older than where we remember coming from, and the paradox comes
> from a confusing between relative and absolute self-sampling on the states
> or histories.
>

The trouble with this is that neither ASSA and RSSA is a law of nature. As
I have said, from the 1p perspective, I live more years between 100 and
1000 than between 1 and 100. So I expect to be very old. What we remember
is actually irrelevant -- we can always check our birth certificate if we
forget how old we are. In other words, we can use external sources to
refresh memories. What we personally remember at any instant is variable
and unreliable. Check against external references.


> Typically, also, old and young are not absolute concept.
>

No, they are concepts relative to actual life span -- you are always at
your youngest when you are born, and at your oldest just before you die.

With mechanism or quantum mechanics without collapse, we can say that we
> are always young.
>

Another good reason for abandoning mechanism.

Bruce

-- 
You received this message because you are subscribed to the Google Groups 
"Everything List" group.
To unsubscribe from this group and stop receiving emails from it, send an email 
to everything-list+unsubscr...@googlegroups.com.
To view this discussion on the web visit 
https://groups.google.com/d/msgid/everything-list/CAFxXSLTM7c97pmzXq5Fb%3D1nq%2BbdxfkqojZn%3DMXsuvhC%2BmLfy8A%40mail.gmail.com.


Re: Quantum immortality

2019-09-11 Thread Bruce Kellett
On Thu, Sep 12, 2019 at 2:55 AM Jason Resch  wrote:

> On Tuesday, September 10, 2019, Bruce Kellett 
> wrote:
>
>> On Wed, Sep 11, 2019 at 10:18 AM 'Brent Meeker' via Everything List <
>> everything-list@googlegroups.com> wrote:
>>
>>> On 9/10/2019 4:30 PM, Bruce Kellett wrote:
>>> > Another argument that has been given here before is that if quantum
>>> > immortality is true, then we should expect to see a number of people
>>> > who are considerably older than the normal life expectancy -- and we
>>> > do not see people who are two or three hundred years old. Even if the
>>> > probabilities are very low, there have been an awful lot of people
>>> > born within the last 500 or so years -- some must have survived on our
>>> > branch if this scenario is true.
>>>
>>> My argument was that each of us should find ourselves to be much older
>>> than even the oldest people we know.
>>
>>
>> That is probably the best single argument against quantum immortality: if
>> QI is true, then the measure of our lifetime after one reaches a normal
>> lifetime is infinitely greater than the measure before age , say, 120 yr.
>> So if one finds oneself younger than 120 years, QI is false, and if MWI is
>> still considered to be true, there must be another argument why MWI does
>> not imply QI.
>>
>
>
> Why do you think that measure only increases with age? On an objective
> level it only decreases.
>

As Bruno would say, "you confuse the 1p with the 1pp." I am talking about
my personal measure of the number of years I have lived. As I get older,
the number of years I have lived increases. If I live to 1000, I have lived
more years between 100 and 1000 than between 1 and 100. This is arithmetic,
after all.

But this discussion has gone off the rails. It started as a discussion of
quantum immortality, and the arguments against this notion, even in MWI.
The arguments against QI that have been advanced are that life-threatening
events tend not to be binary or quantum, but rather we enter a period of
slow decline, due to illness or other factors. Consequently, there is no
reason for us to expect to be immortal, even in MWI. The other argument is
that if QI is true, then you would expect to be very old. This argument was
advanced by Mallah (arXiv: 0905.0187) and has not been satisfactorily
rebutted.

Bruce

-- 
You received this message because you are subscribed to the Google Groups 
"Everything List" group.
To unsubscribe from this group and stop receiving emails from it, send an email 
to everything-list+unsubscr...@googlegroups.com.
To view this discussion on the web visit 
https://groups.google.com/d/msgid/everything-list/CAFxXSLQ92bOxHciguxF6RcVx8Yd1Tmt2D43i2ATF0-ABgge8tw%40mail.gmail.com.


Re: Quantum immortality

2019-09-11 Thread Jason Resch
On Wed, Sep 11, 2019 at 5:17 PM 'Brent Meeker' via Everything List <
everything-list@googlegroups.com> wrote:

>
>
> On 9/11/2019 9:52 AM, Jason Resch wrote:
>
>
>
> On Tuesday, September 10, 2019, 'Brent Meeker' via Everything List <
> everything-list@googlegroups.com> wrote:
>
>>
>>
>> On 9/10/2019 5:35 PM, Jason Resch wrote:
>>
>>
>>
>> On Tue, Sep 10, 2019 at 7:18 PM 'Brent Meeker' via Everything List <
>> everything-list@googlegroups.com> wrote:
>>
>>>
>>>
>>> On 9/10/2019 4:30 PM, Bruce Kellett wrote:
>>> > Another argument that has been given here before is that if quantum
>>> > immortality is true, then we should expect to see a number of people
>>> > who are considerably older than the normal life expectancy -- and we
>>> > do not see people who are two or three hundred years old. Even if the
>>> > probabilities are very low, there have been an awful lot of people
>>> > born within the last 500 or so years -- some must have survived on our
>>> > branch if this scenario is true.
>>>
>>> My argument was that each of us should find ourselves to be much older
>>> than even the oldest people we know.
>>>
>>>
>> You could be very old, but (perhaps temporarily) amnesiac.
>>
>>
>> Then it's strange that so many other people and photographs happen to
>> agree with my memory.  Must be a conspiracy to hide the secret of quantum
>> immortality.  It would certainly be unpopular once people thought about
>> what it means.
>>
>>
>> Brent
>>
>
>
> I mean amnesic regarding the full extent of your billion+ year life.
>
>
> But if I don't remember it, was it me?
>

You might remember more when you wake up.  I think eventually the
probability of your consciousness continuing through an old-age body gets
so low that more likely continuation paths occur, such as waking up as a
technologically advanced being playing sim-human, or having your ever
diminishing conscious awareness eventually intersect with another similarly
diminished consciousness (perhaps that of a brain still developing in a
womb or an egg).

Jason

-- 
You received this message because you are subscribed to the Google Groups 
"Everything List" group.
To unsubscribe from this group and stop receiving emails from it, send an email 
to everything-list+unsubscr...@googlegroups.com.
To view this discussion on the web visit 
https://groups.google.com/d/msgid/everything-list/CA%2BBCJUgH_9wOkx89yFwyMQjTio2x7VCVEoMYqVpqjLy8iqXhPQ%40mail.gmail.com.


Re: Quantum immortality

2019-09-11 Thread 'Brent Meeker' via Everything List




On 9/11/2019 9:55 AM, Jason Resch wrote:
Why do you think that measure only increases with age? On an objective 
level it only decreases.


There's the crux of the question.  The measure of what, or whom?

Brent

--
You received this message because you are subscribed to the Google Groups 
"Everything List" group.
To unsubscribe from this group and stop receiving emails from it, send an email 
to everything-list+unsubscr...@googlegroups.com.
To view this discussion on the web visit 
https://groups.google.com/d/msgid/everything-list/513a427f-32c2-22c0-4d12-332fc395b4c9%40verizon.net.


Re: Quantum immortality

2019-09-11 Thread 'Brent Meeker' via Everything List



On 9/11/2019 9:52 AM, Jason Resch wrote:



On Tuesday, September 10, 2019, 'Brent Meeker' via Everything List 
<mailto:everything-list@googlegroups.com>> wrote:




On 9/10/2019 5:35 PM, Jason Resch wrote:



On Tue, Sep 10, 2019 at 7:18 PM 'Brent Meeker' via Everything
List mailto:everything-list@googlegroups.com>> wrote:



On 9/10/2019 4:30 PM, Bruce Kellett wrote:
> Another argument that has been given here before is that if
quantum
> immortality is true, then we should expect to see a number
of people
> who are considerably older than the normal life expectancy
-- and we
> do not see people who are two or three hundred years old.
Even if the
> probabilities are very low, there have been an awful lot of
people
> born within the last 500 or so years -- some must have
survived on our
> branch if this scenario is true.

My argument was that each of us should find ourselves to be
much older
than even the oldest people we know.


You could be very old, but (perhaps temporarily) amnesiac.


Then it's strange that so many other people and photographs happen
to agree with my memory.  Must be a conspiracy to hide the secret
of quantum immortality.  It would certainly be unpopular once
people thought about what it means.


Brent



I mean amnesic regarding the full extent of your billion+ year life.


But if I don't remember it, was it me?

Brent

--
You received this message because you are subscribed to the Google Groups 
"Everything List" group.
To unsubscribe from this group and stop receiving emails from it, send an email 
to everything-list+unsubscr...@googlegroups.com.
To view this discussion on the web visit 
https://groups.google.com/d/msgid/everything-list/95204b56-50c3-f4ba-5fb9-6a4c2a8a75b0%40verizon.net.


Re: Quantum immortality

2019-09-11 Thread 'Brent Meeker' via Everything List



On 9/11/2019 5:08 AM, Quentin Anciaux wrote:



Le mer. 11 sept. 2019 à 14:01, Bruce Kellett <mailto:bhkellet...@gmail.com>> a écrit :


On Wed, Sep 11, 2019 at 4:57 PM Quentin Anciaux
mailto:allco...@gmail.com>> wrote:

The argument of the measure is based on ASSA and that's why it
is flawed, moments are not random sampled from all possible
moments, with this argument and without QI, you should have
never find yourself young... But somewhere just before your death.


ASSA is not a law of physics. I am not assuming random sampling
from anything. It is just that you spend more time old than young
given quantum immortality. That is not to say that you are never
young -- of course you have to pass through all the years since
your birth, one year at a time. It is just that there are more
years after any given age than before that age.


And so by this reasoning I must be old near death, and it's not the 
case, so something is wrong with your theory.


Or with the theory of quantum immortality and MWI.

Brent

--
You received this message because you are subscribed to the Google Groups 
"Everything List" group.
To unsubscribe from this group and stop receiving emails from it, send an email 
to everything-list+unsubscr...@googlegroups.com.
To view this discussion on the web visit 
https://groups.google.com/d/msgid/everything-list/4e2d2632-36d8-ea84-7f24-1e056dfd5738%40verizon.net.


Re: Quantum immortality

2019-09-11 Thread 'Brent Meeker' via Everything List



On 9/10/2019 11:25 PM, Stathis Papaioannou wrote:



On Wed, 11 Sep 2019 at 12:00, Bruce Kellett <mailto:bhkellet...@gmail.com>> wrote:


On Wed, Sep 11, 2019 at 10:18 AM 'Brent Meeker' via Everything
List mailto:everything-list@googlegroups.com>> wrote:

On 9/10/2019 4:30 PM, Bruce Kellett wrote:
> Another argument that has been given here before is that if
quantum
> immortality is true, then we should expect to see a number
of people
> who are considerably older than the normal life expectancy
-- and we
> do not see people who are two or three hundred years old.
Even if the
> probabilities are very low, there have been an awful lot of
people
> born within the last 500 or so years -- some must have
survived on our
> branch if this scenario is true.

My argument was that each of us should find ourselves to be
much older
than even the oldest people we know.

That is probably the best single argument against quantum
immortality: if QI is true, then the measure of our lifetime after
one reaches a normal lifetime is infinitely greater than the
measure before age , say, 120 yr. So if one finds oneself younger
than 120 years, QI is false, and if MWI is still considered to be
true, there must be another argument why MWI does not imply QI.


The measure of our lifetime when young might be larger than the 
measure when very old if surviving as a very old person becomes 
exponentially less likely. In any case, this is not relevant if it is 
given that there will be a very old version of you in some corner of 
the world, whether distant in time, space or in a parallel universe. 
You cannot avoid surviving to become this version if it actually exists.


Right.  Your observation of your age is conditional on your having 
survived.  So that your survival to a great age is improbable is irrelevant.


Brent


--
Stathis Papaioannou
--
You received this message because you are subscribed to the Google 
Groups "Everything List" group.
To unsubscribe from this group and stop receiving emails from it, send 
an email to everything-list+unsubscr...@googlegroups.com 
<mailto:everything-list+unsubscr...@googlegroups.com>.
To view this discussion on the web visit 
https://groups.google.com/d/msgid/everything-list/CAH%3D2ypW3Nz0gZBF%2BqO3OYq%3DQyPJh_COrJmGGEQM0ssmWn2cf9g%40mail.gmail.com 
<https://groups.google.com/d/msgid/everything-list/CAH%3D2ypW3Nz0gZBF%2BqO3OYq%3DQyPJh_COrJmGGEQM0ssmWn2cf9g%40mail.gmail.com?utm_medium=email&utm_source=footer>.


--
You received this message because you are subscribed to the Google Groups 
"Everything List" group.
To unsubscribe from this group and stop receiving emails from it, send an email 
to everything-list+unsubscr...@googlegroups.com.
To view this discussion on the web visit 
https://groups.google.com/d/msgid/everything-list/7ab1fb4d-8982-aa73-56f1-ba8f202fd150%40verizon.net.


Re: Quantum immortality

2019-09-11 Thread Stathis Papaioannou
On Thu, 12 Sep 2019 at 00:51, smitra  wrote:

> Back to basics. There exists a universal wavefunction that evolves
> according to the Schrodinger equation. Observers are internal structures
> in this description. Whether or not one believes that the Born rule can
> be derived or not, what matters in practice is that you'll end up having
> to use it, so you have to assign a measure for observations that is
> given by the summation of the squared modulus of the states that
> correspond to those observations. The information about personal
> identity must then also be extracted from the wavefunction, so one
> cannot insert this in an ad hoc way.
>
> Quantum immortality is therefore wrong because the measure of the states
> that correspond to extremely old observers is small.


This means that if you don’t know if you are young or very old and have to
guess, you are more likely to be right if you guess that you are young. But
it does not mean that you won’t inevitably become very old.
-- 
Stathis Papaioannou

-- 
You received this message because you are subscribed to the Google Groups 
"Everything List" group.
To unsubscribe from this group and stop receiving emails from it, send an email 
to everything-list+unsubscr...@googlegroups.com.
To view this discussion on the web visit 
https://groups.google.com/d/msgid/everything-list/CAH%3D2ypU%2Bi1zyWR1Bfs97PDB3RZ5GCw%2BW7ApsS1T-Sxijh7tNdQ%40mail.gmail.com.


Re: Quantum immortality

2019-09-11 Thread Jason Resch
On Wednesday, September 11, 2019, Bruce Kellett 
wrote:

> On Wed, Sep 11, 2019 at 4:26 PM Stathis Papaioannou 
> wrote:
>
>> On Wed, 11 Sep 2019 at 12:00, Bruce Kellett 
>> wrote:
>>
>>> On Wed, Sep 11, 2019 at 10:18 AM 'Brent Meeker' via Everything List <
>>> everything-list@googlegroups.com> wrote:
>>>
>>>> On 9/10/2019 4:30 PM, Bruce Kellett wrote:
>>>> > Another argument that has been given here before is that if quantum
>>>> > immortality is true, then we should expect to see a number of people
>>>> > who are considerably older than the normal life expectancy -- and we
>>>> > do not see people who are two or three hundred years old. Even if the
>>>> > probabilities are very low, there have been an awful lot of people
>>>> > born within the last 500 or so years -- some must have survived on
>>>> our
>>>> > branch if this scenario is true.
>>>>
>>>> My argument was that each of us should find ourselves to be much older
>>>> than even the oldest people we know.
>>>
>>>
>>> That is probably the best single argument against quantum immortality:
>>> if QI is true, then the measure of our lifetime after one reaches a normal
>>> lifetime is infinitely greater than the measure before age , say, 120 yr.
>>> So if one finds oneself younger than 120 years, QI is false, and if MWI is
>>> still considered to be true, there must be another argument why MWI does
>>> not imply QI.
>>>
>>
>> The measure of our lifetime when young might be larger than the measure
>> when very old if surviving as a very old person becomes exponentially less
>> likely. In any case, this is not relevant if it is given that there will be
>> a very old version of you in some corner of the world, whether distant in
>> time, space or in a parallel universe. You cannot avoid surviving to become
>> this version if it actually exists.
>>
>
> I think the point of quantum immortality is that everyone is immortal --
> it is not that this is very unlikely because it happens to everyone. So I
> am not sure what measure you think is exponentially decreasing. My personal
> measure of life-years is clearly greater for periods after age 120 yr than
> for the period before. Since this happens for everyone, the collective
> measure over all people is likewise exponentially greater. Even if one
> considers an infinite universe, with an infinite number of copies of me,
> all of these are immortal on the basis of the QI argument. So, again, the
> measure of old age is not decreasing with age.
>
> The situation is different for quantum suicide in the absence of quantum
> immortality. Then one is deliberately courting death on ever run of the
> scenario, and the number of survivors inevitably decreases, even if one
> copy survives indefinitely.
>

Playing Schrodinger's cat over and over with you as the cat is about the
same as living a year. It's just that each iteration of Schrodinger's cat
has a 50% chance if killing you while living another year as a healthy
adult let's say, has a 1 in 900 chance if killing you.  But the
consequences and conclusions are the same in either scenario.

Jason



>
> Bruce
>
> --
> You received this message because you are subscribed to the Google Groups
> "Everything List" group.
> To unsubscribe from this group and stop receiving emails from it, send an
> email to everything-list+unsubscr...@googlegroups.com.
> To view this discussion on the web visit https://groups.google.com/d/
> msgid/everything-list/CAFxXSLQmLkp3c9XgVs3AjciQ8MemX
> 6Qr%2B1ND-t0eyGr1Jh5kKg%40mail.gmail.com
> <https://groups.google.com/d/msgid/everything-list/CAFxXSLQmLkp3c9XgVs3AjciQ8MemX6Qr%2B1ND-t0eyGr1Jh5kKg%40mail.gmail.com?utm_medium=email&utm_source=footer>
> .
>

-- 
You received this message because you are subscribed to the Google Groups 
"Everything List" group.
To unsubscribe from this group and stop receiving emails from it, send an email 
to everything-list+unsubscr...@googlegroups.com.
To view this discussion on the web visit 
https://groups.google.com/d/msgid/everything-list/CA%2BBCJUjH793sEHDjDM-CVrsc8aH6LpdEkmXOCYWBSZczM9wnYA%40mail.gmail.com.


Re: Quantum immortality

2019-09-11 Thread Jason Resch
On Tuesday, September 10, 2019, Bruce Kellett  wrote:

> On Wed, Sep 11, 2019 at 10:18 AM 'Brent Meeker' via Everything List <
> everything-list@googlegroups.com> wrote:
>
>> On 9/10/2019 4:30 PM, Bruce Kellett wrote:
>> > Another argument that has been given here before is that if quantum
>> > immortality is true, then we should expect to see a number of people
>> > who are considerably older than the normal life expectancy -- and we
>> > do not see people who are two or three hundred years old. Even if the
>> > probabilities are very low, there have been an awful lot of people
>> > born within the last 500 or so years -- some must have survived on our
>> > branch if this scenario is true.
>>
>> My argument was that each of us should find ourselves to be much older
>> than even the oldest people we know.
>
>
> That is probably the best single argument against quantum immortality: if
> QI is true, then the measure of our lifetime after one reaches a normal
> lifetime is infinitely greater than the measure before age , say, 120 yr.
> So if one finds oneself younger than 120 years, QI is false, and if MWI is
> still considered to be true, there must be another argument why MWI does
> not imply QI.
>


Why do you think that measure only increases with age? On an objective
level it only decreases.

Jason


>
> Bruce
>
> --
> You received this message because you are subscribed to the Google Groups
> "Everything List" group.
> To unsubscribe from this group and stop receiving emails from it, send an
> email to everything-list+unsubscr...@googlegroups.com.
> To view this discussion on the web visit https://groups.google.com/d/
> msgid/everything-list/CAFxXSLRd0dyOn_JreAHuEfBKpWuSP5f2VkBpO9rZ4QbG
> 79sUQQ%40mail.gmail.com
> <https://groups.google.com/d/msgid/everything-list/CAFxXSLRd0dyOn_JreAHuEfBKpWuSP5f2VkBpO9rZ4QbG79sUQQ%40mail.gmail.com?utm_medium=email&utm_source=footer>
> .
>

-- 
You received this message because you are subscribed to the Google Groups 
"Everything List" group.
To unsubscribe from this group and stop receiving emails from it, send an email 
to everything-list+unsubscr...@googlegroups.com.
To view this discussion on the web visit 
https://groups.google.com/d/msgid/everything-list/CA%2BBCJUjKYK4Ndve2ddO%2B9qCcC2MedwLcccjZYwd0J0_3mdY9Qg%40mail.gmail.com.


Re: Quantum immortality

2019-09-11 Thread Jason Resch
On Tuesday, September 10, 2019, 'Brent Meeker' via Everything List <
everything-list@googlegroups.com> wrote:

>
>
> On 9/10/2019 5:35 PM, Jason Resch wrote:
>
>
>
> On Tue, Sep 10, 2019 at 7:18 PM 'Brent Meeker' via Everything List <
> everything-list@googlegroups.com> wrote:
>
>>
>>
>> On 9/10/2019 4:30 PM, Bruce Kellett wrote:
>> > Another argument that has been given here before is that if quantum
>> > immortality is true, then we should expect to see a number of people
>> > who are considerably older than the normal life expectancy -- and we
>> > do not see people who are two or three hundred years old. Even if the
>> > probabilities are very low, there have been an awful lot of people
>> > born within the last 500 or so years -- some must have survived on our
>> > branch if this scenario is true.
>>
>> My argument was that each of us should find ourselves to be much older
>> than even the oldest people we know.
>>
>>
> You could be very old, but (perhaps temporarily) amnesiac.
>
>
> Then it's strange that so many other people and photographs happen to
> agree with my memory.  Must be a conspiracy to hide the secret of quantum
> immortality.  It would certainly be unpopular once people thought about
> what it means.
>
>
> Brent
>


I mean amnesic regarding the full extent of your billion+ year life.

Jason



> --
> You received this message because you are subscribed to the Google Groups
> "Everything List" group.
> To unsubscribe from this group and stop receiving emails from it, send an
> email to everything-list+unsubscr...@googlegroups.com.
> To view this discussion on the web visit https://groups.google.com/d/
> msgid/everything-list/0c9b0ed2-f183-c862-8503-428da91d0964%40verizon.net
> <https://groups.google.com/d/msgid/everything-list/0c9b0ed2-f183-c862-8503-428da91d0964%40verizon.net?utm_medium=email&utm_source=footer>
> .
>

-- 
You received this message because you are subscribed to the Google Groups 
"Everything List" group.
To unsubscribe from this group and stop receiving emails from it, send an email 
to everything-list+unsubscr...@googlegroups.com.
To view this discussion on the web visit 
https://groups.google.com/d/msgid/everything-list/CA%2BBCJUhfKpJkZjNDvbd4BrnPwadh-9dzy%2BjG1-yuJ25H378h%2Bg%40mail.gmail.com.


Re: Quantum immortality

2019-09-11 Thread Bruno Marchal

> On 11 Sep 2019, at 01:30, Bruce Kellett  wrote:
> 
> From: Bruno Marchal mailto:marc...@ulb.ac.be>>
>>> On 8 Sep 2019, at 13:59, Bruce Kellett >> <mailto:bhkellet...@gmail.com>> wrote:
>>> 
>>> On Sun, Sep 8, 2019 at 8:45 PM Bruno Marchal >> <mailto:marc...@ulb.ac.be>> wrote:
>>> On 7 Sep 2019, at 08:04, Bruce Kellett >> <mailto:bhkellet...@gmail.com>> wrote:
>>>> On Sat, Sep 7, 2019 at 3:54 PM 'Brent Meeker' via Everything List 
>>>> >>> <mailto:everything-list@googlegroups.com>> wrote:
>>>> On 9/6/2019 10:21 PM, Philip Thrift wrote:
>>>>> Sean Carroll is on a nationwide speaking tour now evangelizing Many 
>>>>> Worlds.
>>>>> 
>>>>> What is the predictive power of Many Worlds?
>>>> 
>>>> None, unless someone can figure out how to derive Born's rule from 
>>>> it...which I think is impossible.  But it does go a way toward making the 
>>>> story of measurement more consistent.
>>>> 
>>>> Amplify the above statement.
>>>> 
>>>> Even Zurek, who starts from a many worlds perspective, thinks that 
>>>> ultimately one can abandon the non-seen worlds as irrelevant.
>>> 
>>> But irrelevant does not mean false. So it is irrelevant in physics, but it 
>>> is not irrelevant in theology. It might plays a role concerning the 
>>> interpretation of death, like with quantum immortality.
>>> 
>>> If the only relevance you can find for many worlds is quantum immortality, 
>>> then many worlds is indeed dead. Quantum immortality has been shown many 
>>> times to be a complete nonsense.
>> 
>> Really. I did not known that. Could you give the references.
> Follow the Wikipedia entry on quantum suicide.
> 
That is not what I mean by a  reference.


> The main problem with the idea of quantum immortality is that not all 
> life-threatening events that one can encounter are in the form of alternative 
> outcomes to quantum processes.
> 
The point is that at each instant we have an infinity (plausibly aleph_1, at 
least aleph_0) alternate accessible histories, and it is up to you to prove 
that when we die in some history, we die in all. That is dubious, because there 
are always consistent extensions (but of course I use Mechanism here).





> Quantum suicide is an attempt to overcome this problem by linking death or 
> survival directly to the outcome of a particular quantum process. David 
> Deutsch was sceptical that this worked:
>  'Physicist David Deutsch <https://en.wikipedia.org/wiki/David_Deutsch>, 
> though a proponent of the many-worlds interpretation, states regarding 
> quantum suicide that "that way of applying probabilities does not follow 
> directly from quantum theory, as the usual one does. It requires an 
> additional assumption, namely that when making decisions one should ignore 
> the histories in which the decision-maker is absent[M]y guess is that the 
> assumption is false."
> Tegmark was also doubtful about the chances for quantum immortality -- 
> pointing out that dying is rarely a binary event; it is more often the result 
> of a slow cumulative process.
> 
> Another argument that has been given here before is that if quantum 
> immortality is true, then we should expect to see a number of people who are 
> considerably older than the normal life expectancy
> 
That makes no sense. The argument rest typically on the first person, not the 
first person plural.



> -- and we do not see people who are two or three hundred years old.
> 
Just compute the probability. That would be akin to a white rabbit. The 
argument concerns only the first person experience, and it can involved amnesia.



> Even if the probabilities are very low, there have been an awful lot of 
> people born within the last 500 or so years -- some must have survived on our 
> branch if this scenario is true.
> 
The probability is the same as the one with a beam splitter (some “half 
mirror”) and all photons going on the same path. Nobody as seen this. But if 
you kill yourself if the photon go in the non rare path, you will see the 
photon going on the rare path with probability one, by the cul-de-sac principle 
(again, I use, like Everett; the mechanist hypothesis).




> 
>> That would be an indice that Mechanism is false, given that quantum 
>> immortality is deduce here from the already much more obvious arithmetical 
>> immortality, which is disturbing, but hard to avoid.
> 
> Well, as you know, I consider mechanism to be false in any case, so the 
> failure of quantum im

Re: Quantum immortality

2019-09-11 Thread smitra
Back to basics. There exists a universal wavefunction that evolves 
according to the Schrodinger equation. Observers are internal structures 
in this description. Whether or not one believes that the Born rule can 
be derived or not, what matters in practice is that you'll end up having 
to use it, so you have to assign a measure for observations that is 
given by the summation of the squared modulus of the states that 
correspond to those observations. The information about personal 
identity must then also be extracted from the wavefunction, so one 
cannot insert this in an ad hoc way.


Quantum immortality is therefore wrong because the measure of the states 
that correspond to extremely old observers is small.



Saibal

--
You received this message because you are subscribed to the Google Groups 
"Everything List" group.
To unsubscribe from this group and stop receiving emails from it, send an email 
to everything-list+unsubscr...@googlegroups.com.
To view this discussion on the web visit 
https://groups.google.com/d/msgid/everything-list/ee05eb58fca14c1114ba1fabfc5d26e3%40zonnet.nl.


Re: Quantum immortality

2019-09-11 Thread John Clark
On Tue, Sep 10, 2019 at 7:30 PM Bruce Kellett 
wrote:

*> Another argument that has been given here before is that if quantum
> immortality is true, then we should expect to see a number of people who
> are considerably older than the normal life expectancy -- and we do not see
> people who are two or three hundred years old. Even if the probabilities
> are very low, there have been an awful lot of people born within the last
> 500 or so years -- some must have survived on our branch if this scenario
> is true.*


That doesn't follow. Yes there have been an awful lot of people born within
the last 500 years but that number is utterally dwarfed by the number of
quantum interactions that occurred in the last 500 years, and if Many
Worlds is correct then each one created a universe. So there is indeed a
universe that has a 300 year old man in it and even a 30,000 year old man,
but it is very unlikely you're living in one of them.

By the way, Hugh Everett the inventor of the Many Worlds idea was working
for the pentagon in 1962 and everybody knew the 1962 Cuban Missile Crisis
was extremely dangerous, but recently as new information gets unclassified
it's now apparent it was vastly more dangerous than anybody thought at the
time. I think a logical person analyzing all the data would conclude the
most likely outcome was global thermonuclear war. But as unlikely as it
seems we somehow survived, at least in this universe.

 John K Clark

-- 
You received this message because you are subscribed to the Google Groups 
"Everything List" group.
To unsubscribe from this group and stop receiving emails from it, send an email 
to everything-list+unsubscr...@googlegroups.com.
To view this discussion on the web visit 
https://groups.google.com/d/msgid/everything-list/CAJPayv1ACY10%2BRV8wHSCzP34zqR_MG9mKHOh4ZZ7c%3Df4FW7YAg%40mail.gmail.com.


Re: Quantum immortality

2019-09-11 Thread Philip Thrift


My only point is that if you go to the actual Google Group

 Everything List 
<https://groups.google.com/forum/?hl=en#!forum/everything-list>

https://groups.google.com/forum/?hl=en#!forum/everything-list

there is the Topic 
 
"The Delusion of Scientific Omniscience" (John Horgan) 
<https://groups.google.com/forum/?hl=en#!topic/everything-list/NJWeGLXI2yw>


https://groups.google.com/forum/?hl=en#!topic/everything-list/NJWeGLXI2yw

All the posts *Re*:* Quantum immortality *are under that Topic,

There is  ontologically no such thing as as a fork or a thread in Google 
Groups. You can only create a new Topic.

@philipthrift

On Wednesday, September 11, 2019 at 4:25:30 AM UTC-5, telmo wrote:
>
>
>
> On Wed, Sep 11, 2019, at 07:01, Philip Thrift wrote:
>
>
>
> >   *Re: Quantum immortality*
>
>
> Noting that changing a "Subject" in an emailer does not change the Topic
>
>
> It's not a change of topic, it's a fork :) You can continue the original 
> thread.
>
> Telmo.
>
>
> "The Delusion of Scientific Omniscience" (John Horgan)
> 66 posts by 12 authors
>  
>
>
>
> a post is under in Google Groups.
>
> @philipthrift
>
>
>

-- 
You received this message because you are subscribed to the Google Groups 
"Everything List" group.
To unsubscribe from this group and stop receiving emails from it, send an email 
to everything-list+unsubscr...@googlegroups.com.
To view this discussion on the web visit 
https://groups.google.com/d/msgid/everything-list/575f8b74-8776-4f52-86f4-a67bf44c6c82%40googlegroups.com.


Re: Quantum immortality

2019-09-11 Thread Bruce Kellett
On Wed, Sep 11, 2019 at 10:08 PM Quentin Anciaux  wrote:

> Le mer. 11 sept. 2019 à 14:01, Bruce Kellett  a
> écrit :
>
>> On Wed, Sep 11, 2019 at 4:57 PM Quentin Anciaux 
>> wrote:
>>
>>> The argument of the measure is based on ASSA and that's why it is
>>> flawed, moments are not random sampled from all possible moments, with this
>>> argument and without QI, you should have never find yourself young... But
>>> somewhere just before your death.
>>>
>>
>> ASSA is not a law of physics. I am not assuming random sampling from
>> anything. It is just that you spend more time old than young given quantum
>> immortality. That is not to say that you are never young -- of course you
>> have to pass through all the years since your birth, one year at a time. It
>> is just that there are more years after any given age than before that age.
>>
>
> And so by this reasoning I must be old near death, and it's not the case,
> so something is wrong with your theory.
>

That does not follow. "years" is only "un facon de parle".

Bruce

-- 
You received this message because you are subscribed to the Google Groups 
"Everything List" group.
To unsubscribe from this group and stop receiving emails from it, send an email 
to everything-list+unsubscr...@googlegroups.com.
To view this discussion on the web visit 
https://groups.google.com/d/msgid/everything-list/CAFxXSLT9-k-rOnKw4Ze-Ty6aXt5p9_kGLW%3D%2B7yy-DYoMbuLAtw%40mail.gmail.com.


Re: Quantum immortality

2019-09-11 Thread Bruce Kellett
On Wed, Sep 11, 2019 at 5:50 PM Stathis Papaioannou 
wrote:

> On Wed, 11 Sep 2019 at 16:43, Bruce Kellett  wrote:
>
>>
>> I think the point of quantum immortality is that everyone is immortal --
>> it is not that this is very unlikely because it happens to everyone. So I
>> am not sure what measure you think is exponentially decreasing. My personal
>> measure of life-years is clearly greater for periods after age 120 yr than
>> for the period before. Since this happens for everyone, the collective
>> measure over all people is likewise exponentially greater. Even if one
>> considers an infinite universe, with an infinite number of copies of me,
>> all of these are immortal on the basis of the QI argument. So, again, the
>> measure of old age is not decreasing with age.
>>
>> The situation is different for quantum suicide in the absence of quantum
>> immortality. Then one is deliberately courting death on ever run of the
>> scenario, and the number of survivors inevitably decreases, even if one
>> copy survives indefinitely.
>>
>
> Do you accept the idea that if there is continual duplication of the world
> through whatever means and each individual is mortal (the probability that
> he will survive a period t approaches zero as t approaches infinity), you
> will survive indefinitely?
>

A copy of you will. When I say that everyone is immortal under QI, I mean
that a copy of every person survives to any arbitrarily large age. This is
not because each near-death experience is quantum in origin, but because in
MWI there is continual duplication due to random quantum events. Every copy
inevitably dies at some point, but with new copies continually generated,
some will survive indefinitely. Not that this is anywhere as much as many
copies as people fear, because quantum events only give rise to world
splitting if the effect is magnified to be of macroscopic, decohered,
significance. That does not happen for every random neutrino that passes
through your body, although there a a very large number of scattering
events that could happen for any such neutrino. But none, even if there is
an interaction in your body, ever becomes macroscopic. But this is
irrelevant to the argument from relative measures.

Bruce

-- 
You received this message because you are subscribed to the Google Groups 
"Everything List" group.
To unsubscribe from this group and stop receiving emails from it, send an email 
to everything-list+unsubscr...@googlegroups.com.
To view this discussion on the web visit 
https://groups.google.com/d/msgid/everything-list/CAFxXSLRLqS8jU3fnBqWakFGnu6B9K%3D1GbfpRSA-KEn_a9f_a0w%40mail.gmail.com.


Re: Quantum immortality

2019-09-11 Thread Quentin Anciaux
Le mer. 11 sept. 2019 à 14:08, Quentin Anciaux  a
écrit :

>
>
> Le mer. 11 sept. 2019 à 14:01, Bruce Kellett  a
> écrit :
>
>> On Wed, Sep 11, 2019 at 4:57 PM Quentin Anciaux 
>> wrote:
>>
>>> The argument of the measure is based on ASSA and that's why it is
>>> flawed, moments are not random sampled from all possible moments, with this
>>> argument and without QI, you should have never find yourself young... But
>>> somewhere just before your death.
>>>
>>
>> ASSA is not a law of physics. I am not assuming random sampling from
>> anything. It is just that you spend more time old than young given quantum
>> immortality. That is not to say that you are never young -- of course you
>> have to pass through all the years since your birth, one year at a time. It
>> is just that there are more years after any given age than before that age.
>>
>
> And so by this reasoning I must be old near death, and it's not the case,
> so something is wrong with your theory.
>

I mean even *without* QI...

>
>> Bruce
>>
>> --
>> You received this message because you are subscribed to the Google Groups
>> "Everything List" group.
>> To unsubscribe from this group and stop receiving emails from it, send an
>> email to everything-list+unsubscr...@googlegroups.com.
>> To view this discussion on the web visit
>> https://groups.google.com/d/msgid/everything-list/CAFxXSLQ_Qq0vB89yE080%2B9jshUY_6bBtUrw-ibb8rsohQPtaCg%40mail.gmail.com
>> <https://groups.google.com/d/msgid/everything-list/CAFxXSLQ_Qq0vB89yE080%2B9jshUY_6bBtUrw-ibb8rsohQPtaCg%40mail.gmail.com?utm_medium=email&utm_source=footer>
>> .
>>
>
>
> --
> All those moments will be lost in time, like tears in rain. (Roy
> Batty/Rutger Hauer)
>


-- 
All those moments will be lost in time, like tears in rain. (Roy
Batty/Rutger Hauer)

-- 
You received this message because you are subscribed to the Google Groups 
"Everything List" group.
To unsubscribe from this group and stop receiving emails from it, send an email 
to everything-list+unsubscr...@googlegroups.com.
To view this discussion on the web visit 
https://groups.google.com/d/msgid/everything-list/CAMW2kArsUP2cYCs43Acjj5M7s4%3DVY-qrHcRUjBtMTS_WE4Y1sA%40mail.gmail.com.


Re: Quantum immortality

2019-09-11 Thread Quentin Anciaux
Le mer. 11 sept. 2019 à 14:01, Bruce Kellett  a
écrit :

> On Wed, Sep 11, 2019 at 4:57 PM Quentin Anciaux 
> wrote:
>
>> The argument of the measure is based on ASSA and that's why it is flawed,
>> moments are not random sampled from all possible moments, with this
>> argument and without QI, you should have never find yourself young... But
>> somewhere just before your death.
>>
>
> ASSA is not a law of physics. I am not assuming random sampling from
> anything. It is just that you spend more time old than young given quantum
> immortality. That is not to say that you are never young -- of course you
> have to pass through all the years since your birth, one year at a time. It
> is just that there are more years after any given age than before that age.
>

And so by this reasoning I must be old near death, and it's not the case,
so something is wrong with your theory.

>
> Bruce
>
> --
> You received this message because you are subscribed to the Google Groups
> "Everything List" group.
> To unsubscribe from this group and stop receiving emails from it, send an
> email to everything-list+unsubscr...@googlegroups.com.
> To view this discussion on the web visit
> https://groups.google.com/d/msgid/everything-list/CAFxXSLQ_Qq0vB89yE080%2B9jshUY_6bBtUrw-ibb8rsohQPtaCg%40mail.gmail.com
> <https://groups.google.com/d/msgid/everything-list/CAFxXSLQ_Qq0vB89yE080%2B9jshUY_6bBtUrw-ibb8rsohQPtaCg%40mail.gmail.com?utm_medium=email&utm_source=footer>
> .
>


-- 
All those moments will be lost in time, like tears in rain. (Roy
Batty/Rutger Hauer)

-- 
You received this message because you are subscribed to the Google Groups 
"Everything List" group.
To unsubscribe from this group and stop receiving emails from it, send an email 
to everything-list+unsubscr...@googlegroups.com.
To view this discussion on the web visit 
https://groups.google.com/d/msgid/everything-list/CAMW2kAofOsTyd5JHND%2BS2XwhSc7Zj20QmmBCzBe-8JEY81X3%3Dw%40mail.gmail.com.


Re: Quantum immortality

2019-09-11 Thread Bruce Kellett
On Wed, Sep 11, 2019 at 4:57 PM Quentin Anciaux  wrote:

> The argument of the measure is based on ASSA and that's why it is flawed,
> moments are not random sampled from all possible moments, with this
> argument and without QI, you should have never find yourself young... But
> somewhere just before your death.
>

ASSA is not a law of physics. I am not assuming random sampling from
anything. It is just that you spend more time old than young given quantum
immortality. That is not to say that you are never young -- of course you
have to pass through all the years since your birth, one year at a time. It
is just that there are more years after any given age than before that age.

Bruce

-- 
You received this message because you are subscribed to the Google Groups 
"Everything List" group.
To unsubscribe from this group and stop receiving emails from it, send an email 
to everything-list+unsubscr...@googlegroups.com.
To view this discussion on the web visit 
https://groups.google.com/d/msgid/everything-list/CAFxXSLQ_Qq0vB89yE080%2B9jshUY_6bBtUrw-ibb8rsohQPtaCg%40mail.gmail.com.


Re: Quantum immortality

2019-09-11 Thread Telmo Menezes


On Wed, Sep 11, 2019, at 07:01, Philip Thrift wrote:
> 
> 
> > *Re: Quantum immortality*
> *
*
> **
> Noting that changing a "Subject" in an emailer does not change the Topic

It's not a change of topic, it's a fork :) You can continue the original thread.

Telmo.

> 
> "The Delusion of Scientific Omniscience" (John Horgan)
> 66 posts by 12 authors
> 
> 
> 
> 
> a post is under in Google Groups.
> 
> @philipthrift
> 
> 
> 
> 
>> 
> 

> --
>  You received this message because you are subscribed to the Google Groups 
> "Everything List" group.
>  To unsubscribe from this group and stop receiving emails from it, send an 
> email to everything-list+unsubscr...@googlegroups.com.
>  To view this discussion on the web visit 
> https://groups.google.com/d/msgid/everything-list/5d19acc8-1041-4b38-b259-b6494c20c4db%40googlegroups.com
>  
> <https://groups.google.com/d/msgid/everything-list/5d19acc8-1041-4b38-b259-b6494c20c4db%40googlegroups.com?utm_medium=email&utm_source=footer>.
> 
> 
> 
> *Attachments:*
>  * Auto Generated Inline Image 1

-- 
You received this message because you are subscribed to the Google Groups 
"Everything List" group.
To unsubscribe from this group and stop receiving emails from it, send an email 
to everything-list+unsubscr...@googlegroups.com.
To view this discussion on the web visit 
https://groups.google.com/d/msgid/everything-list/ae3120f9-bf67-4eec-93d5-7e37e26b19ed%40www.fastmail.com.


Re: Quantum immortality

2019-09-11 Thread Stathis Papaioannou
On Wed, 11 Sep 2019 at 16:43, Bruce Kellett  wrote:

> On Wed, Sep 11, 2019 at 4:26 PM Stathis Papaioannou 
> wrote:
>
>> On Wed, 11 Sep 2019 at 12:00, Bruce Kellett 
>> wrote:
>>
>>> On Wed, Sep 11, 2019 at 10:18 AM 'Brent Meeker' via Everything List <
>>> everything-list@googlegroups.com> wrote:
>>>
>>>> On 9/10/2019 4:30 PM, Bruce Kellett wrote:
>>>> > Another argument that has been given here before is that if quantum
>>>> > immortality is true, then we should expect to see a number of people
>>>> > who are considerably older than the normal life expectancy -- and we
>>>> > do not see people who are two or three hundred years old. Even if the
>>>> > probabilities are very low, there have been an awful lot of people
>>>> > born within the last 500 or so years -- some must have survived on
>>>> our
>>>> > branch if this scenario is true.
>>>>
>>>> My argument was that each of us should find ourselves to be much older
>>>> than even the oldest people we know.
>>>
>>>
>>> That is probably the best single argument against quantum immortality:
>>> if QI is true, then the measure of our lifetime after one reaches a normal
>>> lifetime is infinitely greater than the measure before age , say, 120 yr.
>>> So if one finds oneself younger than 120 years, QI is false, and if MWI is
>>> still considered to be true, there must be another argument why MWI does
>>> not imply QI.
>>>
>>
>> The measure of our lifetime when young might be larger than the measure
>> when very old if surviving as a very old person becomes exponentially less
>> likely. In any case, this is not relevant if it is given that there will be
>> a very old version of you in some corner of the world, whether distant in
>> time, space or in a parallel universe. You cannot avoid surviving to become
>> this version if it actually exists.
>>
>
> I think the point of quantum immortality is that everyone is immortal --
> it is not that this is very unlikely because it happens to everyone. So I
> am not sure what measure you think is exponentially decreasing. My personal
> measure of life-years is clearly greater for periods after age 120 yr than
> for the period before. Since this happens for everyone, the collective
> measure over all people is likewise exponentially greater. Even if one
> considers an infinite universe, with an infinite number of copies of me,
> all of these are immortal on the basis of the QI argument. So, again, the
> measure of old age is not decreasing with age.
>
> The situation is different for quantum suicide in the absence of quantum
> immortality. Then one is deliberately courting death on ever run of the
> scenario, and the number of survivors inevitably decreases, even if one
> copy survives indefinitely.
>

Do you accept the idea that if there is continual duplication of the world
through whatever means and each individual is mortal (the probability that
he will survive a period t approaches zero as t approaches infinity), you
will survive indefinitely?

> --
Stathis Papaioannou

-- 
You received this message because you are subscribed to the Google Groups 
"Everything List" group.
To unsubscribe from this group and stop receiving emails from it, send an email 
to everything-list+unsubscr...@googlegroups.com.
To view this discussion on the web visit 
https://groups.google.com/d/msgid/everything-list/CAH%3D2ypXZa9VNUmjKvKq24U_pL4kEU%3DCG--DFT%2B%3DLJEp%3Dfx-oWw%40mail.gmail.com.


Re: Quantum immortality

2019-09-11 Thread Philip Thrift


>   Re: Quantum immortality


Noting that changing a "Subject" in an emailer does not change the Topic

"The Delusion of Scientific Omniscience" (John Horgan)
66 posts by 12 authors
 


a post is under in Google Groups.

@philipthrift




-- 
You received this message because you are subscribed to the Google Groups 
"Everything List" group.
To unsubscribe from this group and stop receiving emails from it, send an email 
to everything-list+unsubscr...@googlegroups.com.
To view this discussion on the web visit 
https://groups.google.com/d/msgid/everything-list/5d19acc8-1041-4b38-b259-b6494c20c4db%40googlegroups.com.


Re: Quantum immortality

2019-09-11 Thread Telmo Menezes



On Wed, Sep 11, 2019, at 00:18, 'Brent Meeker' via Everything List wrote:
> 
> 
> On 9/10/2019 4:30 PM, Bruce Kellett wrote:
> > Another argument that has been given here before is that if quantum 
> > immortality is true, then we should expect to see a number of people 
> > who are considerably older than the normal life expectancy -- and we 
> > do not see people who are two or three hundred years old. Even if the 
> > probabilities are very low, there have been an awful lot of people 
> > born within the last 500 or so years -- some must have survived on our 
> > branch if this scenario is true.
> 
> My argument was that each of us should find ourselves to be much older 
> than even the oldest people we know.

That is my argument against the existence of heaven and hell. If they existed, 
we would already be there -- oh wait :)
However, here it only applies if the tree is infinitely deep but not infinitely 
wide.

Telmo.

> Brent
> 
> -- 
> You received this message because you are subscribed to the Google 
> Groups "Everything List" group.
> To unsubscribe from this group and stop receiving emails from it, send 
> an email to everything-list+unsubscr...@googlegroups.com.
> To view this discussion on the web visit 
> https://groups.google.com/d/msgid/everything-list/7ec88255-a555-7f57-eb80-a31c2a559a3d%40verizon.net.
>

-- 
You received this message because you are subscribed to the Google Groups 
"Everything List" group.
To unsubscribe from this group and stop receiving emails from it, send an email 
to everything-list+unsubscr...@googlegroups.com.
To view this discussion on the web visit 
https://groups.google.com/d/msgid/everything-list/9e0adf5c-ad03-42c2-aaf0-91d18d37169b%40www.fastmail.com.


Re: Quantum immortality

2019-09-10 Thread Quentin Anciaux
The argument of the measure is based on ASSA and that's why it is flawed,
moments are not random sampled from all possible moments, with this
argument and without QI, you should have never find yourself young... But
somewhere just before your death.

Quentin

Le mer. 11 sept. 2019 à 08:43, Bruce Kellett  a
écrit :

> On Wed, Sep 11, 2019 at 4:26 PM Stathis Papaioannou 
> wrote:
>
>> On Wed, 11 Sep 2019 at 12:00, Bruce Kellett 
>> wrote:
>>
>>> On Wed, Sep 11, 2019 at 10:18 AM 'Brent Meeker' via Everything List <
>>> everything-list@googlegroups.com> wrote:
>>>
>>>> On 9/10/2019 4:30 PM, Bruce Kellett wrote:
>>>> > Another argument that has been given here before is that if quantum
>>>> > immortality is true, then we should expect to see a number of people
>>>> > who are considerably older than the normal life expectancy -- and we
>>>> > do not see people who are two or three hundred years old. Even if the
>>>> > probabilities are very low, there have been an awful lot of people
>>>> > born within the last 500 or so years -- some must have survived on
>>>> our
>>>> > branch if this scenario is true.
>>>>
>>>> My argument was that each of us should find ourselves to be much older
>>>> than even the oldest people we know.
>>>
>>>
>>> That is probably the best single argument against quantum immortality:
>>> if QI is true, then the measure of our lifetime after one reaches a normal
>>> lifetime is infinitely greater than the measure before age , say, 120 yr.
>>> So if one finds oneself younger than 120 years, QI is false, and if MWI is
>>> still considered to be true, there must be another argument why MWI does
>>> not imply QI.
>>>
>>
>> The measure of our lifetime when young might be larger than the measure
>> when very old if surviving as a very old person becomes exponentially less
>> likely. In any case, this is not relevant if it is given that there will be
>> a very old version of you in some corner of the world, whether distant in
>> time, space or in a parallel universe. You cannot avoid surviving to become
>> this version if it actually exists.
>>
>
> I think the point of quantum immortality is that everyone is immortal --
> it is not that this is very unlikely because it happens to everyone. So I
> am not sure what measure you think is exponentially decreasing. My personal
> measure of life-years is clearly greater for periods after age 120 yr than
> for the period before. Since this happens for everyone, the collective
> measure over all people is likewise exponentially greater. Even if one
> considers an infinite universe, with an infinite number of copies of me,
> all of these are immortal on the basis of the QI argument. So, again, the
> measure of old age is not decreasing with age.
>
> The situation is different for quantum suicide in the absence of quantum
> immortality. Then one is deliberately courting death on ever run of the
> scenario, and the number of survivors inevitably decreases, even if one
> copy survives indefinitely.
>
> Bruce
>
> --
> You received this message because you are subscribed to the Google Groups
> "Everything List" group.
> To unsubscribe from this group and stop receiving emails from it, send an
> email to everything-list+unsubscr...@googlegroups.com.
> To view this discussion on the web visit
> https://groups.google.com/d/msgid/everything-list/CAFxXSLQmLkp3c9XgVs3AjciQ8MemX6Qr%2B1ND-t0eyGr1Jh5kKg%40mail.gmail.com
> <https://groups.google.com/d/msgid/everything-list/CAFxXSLQmLkp3c9XgVs3AjciQ8MemX6Qr%2B1ND-t0eyGr1Jh5kKg%40mail.gmail.com?utm_medium=email&utm_source=footer>
> .
>

-- 
You received this message because you are subscribed to the Google Groups 
"Everything List" group.
To unsubscribe from this group and stop receiving emails from it, send an email 
to everything-list+unsubscr...@googlegroups.com.
To view this discussion on the web visit 
https://groups.google.com/d/msgid/everything-list/CAMW2kApb7f-x%2BRUuwTKk%3DTKEeBFBbGs21wOWy5Z83uAxZQ%2BOxQ%40mail.gmail.com.


Re: Quantum immortality

2019-09-10 Thread Bruce Kellett
On Wed, Sep 11, 2019 at 4:26 PM Stathis Papaioannou 
wrote:

> On Wed, 11 Sep 2019 at 12:00, Bruce Kellett  wrote:
>
>> On Wed, Sep 11, 2019 at 10:18 AM 'Brent Meeker' via Everything List <
>> everything-list@googlegroups.com> wrote:
>>
>>> On 9/10/2019 4:30 PM, Bruce Kellett wrote:
>>> > Another argument that has been given here before is that if quantum
>>> > immortality is true, then we should expect to see a number of people
>>> > who are considerably older than the normal life expectancy -- and we
>>> > do not see people who are two or three hundred years old. Even if the
>>> > probabilities are very low, there have been an awful lot of people
>>> > born within the last 500 or so years -- some must have survived on our
>>> > branch if this scenario is true.
>>>
>>> My argument was that each of us should find ourselves to be much older
>>> than even the oldest people we know.
>>
>>
>> That is probably the best single argument against quantum immortality: if
>> QI is true, then the measure of our lifetime after one reaches a normal
>> lifetime is infinitely greater than the measure before age , say, 120 yr.
>> So if one finds oneself younger than 120 years, QI is false, and if MWI is
>> still considered to be true, there must be another argument why MWI does
>> not imply QI.
>>
>
> The measure of our lifetime when young might be larger than the measure
> when very old if surviving as a very old person becomes exponentially less
> likely. In any case, this is not relevant if it is given that there will be
> a very old version of you in some corner of the world, whether distant in
> time, space or in a parallel universe. You cannot avoid surviving to become
> this version if it actually exists.
>

I think the point of quantum immortality is that everyone is immortal -- it
is not that this is very unlikely because it happens to everyone. So I am
not sure what measure you think is exponentially decreasing. My personal
measure of life-years is clearly greater for periods after age 120 yr than
for the period before. Since this happens for everyone, the collective
measure over all people is likewise exponentially greater. Even if one
considers an infinite universe, with an infinite number of copies of me,
all of these are immortal on the basis of the QI argument. So, again, the
measure of old age is not decreasing with age.

The situation is different for quantum suicide in the absence of quantum
immortality. Then one is deliberately courting death on ever run of the
scenario, and the number of survivors inevitably decreases, even if one
copy survives indefinitely.

Bruce

-- 
You received this message because you are subscribed to the Google Groups 
"Everything List" group.
To unsubscribe from this group and stop receiving emails from it, send an email 
to everything-list+unsubscr...@googlegroups.com.
To view this discussion on the web visit 
https://groups.google.com/d/msgid/everything-list/CAFxXSLQmLkp3c9XgVs3AjciQ8MemX6Qr%2B1ND-t0eyGr1Jh5kKg%40mail.gmail.com.


Re: Quantum immortality

2019-09-10 Thread Stathis Papaioannou
On Wed, 11 Sep 2019 at 12:00, Bruce Kellett  wrote:

> On Wed, Sep 11, 2019 at 10:18 AM 'Brent Meeker' via Everything List <
> everything-list@googlegroups.com> wrote:
>
>> On 9/10/2019 4:30 PM, Bruce Kellett wrote:
>> > Another argument that has been given here before is that if quantum
>> > immortality is true, then we should expect to see a number of people
>> > who are considerably older than the normal life expectancy -- and we
>> > do not see people who are two or three hundred years old. Even if the
>> > probabilities are very low, there have been an awful lot of people
>> > born within the last 500 or so years -- some must have survived on our
>> > branch if this scenario is true.
>>
>> My argument was that each of us should find ourselves to be much older
>> than even the oldest people we know.
>
>
> That is probably the best single argument against quantum immortality: if
> QI is true, then the measure of our lifetime after one reaches a normal
> lifetime is infinitely greater than the measure before age , say, 120 yr.
> So if one finds oneself younger than 120 years, QI is false, and if MWI is
> still considered to be true, there must be another argument why MWI does
> not imply QI.
>

The measure of our lifetime when young might be larger than the measure
when very old if surviving as a very old person becomes exponentially less
likely. In any case, this is not relevant if it is given that there will be
a very old version of you in some corner of the world, whether distant in
time, space or in a parallel universe. You cannot avoid surviving to become
this version if it actually exists.

> --
Stathis Papaioannou

-- 
You received this message because you are subscribed to the Google Groups 
"Everything List" group.
To unsubscribe from this group and stop receiving emails from it, send an email 
to everything-list+unsubscr...@googlegroups.com.
To view this discussion on the web visit 
https://groups.google.com/d/msgid/everything-list/CAH%3D2ypW3Nz0gZBF%2BqO3OYq%3DQyPJh_COrJmGGEQM0ssmWn2cf9g%40mail.gmail.com.


Re: Quantum immortality

2019-09-10 Thread Bruce Kellett
On Wed, Sep 11, 2019 at 10:18 AM 'Brent Meeker' via Everything List <
everything-list@googlegroups.com> wrote:

> On 9/10/2019 4:30 PM, Bruce Kellett wrote:
> > Another argument that has been given here before is that if quantum
> > immortality is true, then we should expect to see a number of people
> > who are considerably older than the normal life expectancy -- and we
> > do not see people who are two or three hundred years old. Even if the
> > probabilities are very low, there have been an awful lot of people
> > born within the last 500 or so years -- some must have survived on our
> > branch if this scenario is true.
>
> My argument was that each of us should find ourselves to be much older
> than even the oldest people we know.


That is probably the best single argument against quantum immortality: if
QI is true, then the measure of our lifetime after one reaches a normal
lifetime is infinitely greater than the measure before age , say, 120 yr.
So if one finds oneself younger than 120 years, QI is false, and if MWI is
still considered to be true, there must be another argument why MWI does
not imply QI.

Bruce

-- 
You received this message because you are subscribed to the Google Groups 
"Everything List" group.
To unsubscribe from this group and stop receiving emails from it, send an email 
to everything-list+unsubscr...@googlegroups.com.
To view this discussion on the web visit 
https://groups.google.com/d/msgid/everything-list/CAFxXSLRd0dyOn_JreAHuEfBKpWuSP5f2VkBpO9rZ4QbG79sUQQ%40mail.gmail.com.


Re: Quantum immortality

2019-09-10 Thread Bruce Kellett
On Wed, Sep 11, 2019 at 10:51 AM 'Brent Meeker' via Everything List <
everything-list@googlegroups.com> wrote:

> On 9/10/2019 5:35 PM, Jason Resch wrote:
>
> On Tue, Sep 10, 2019 at 7:18 PM 'Brent Meeker' via Everything List <
> everything-list@googlegroups.com> wrote:
>
>>
>>
>> On 9/10/2019 4:30 PM, Bruce Kellett wrote:
>> > Another argument that has been given here before is that if quantum
>> > immortality is true, then we should expect to see a number of people
>> > who are considerably older than the normal life expectancy -- and we
>> > do not see people who are two or three hundred years old. Even if the
>> > probabilities are very low, there have been an awful lot of people
>> > born within the last 500 or so years -- some must have survived on our
>> > branch if this scenario is true.
>>
>> My argument was that each of us should find ourselves to be much older
>> than even the oldest people we know.
>>
>>
> You could be very old, but (perhaps temporarily) amnesiac.
>
>
> Then it's strange that so many other people and photographs happen to
> agree with my memory.  Must be a conspiracy to hide the secret of quantum
> immortality.  It would certainly be unpopular once people thought about
> what it means.
>

Exploring further after my initial post on this, I found the following
paper by Mallah:

https://arxiv.org/abs/0902.0187

which explores the issues in some detail. He consider the arguments against
both quantum suicide and quantum immortality to be decisive.

Bruce

-- 
You received this message because you are subscribed to the Google Groups 
"Everything List" group.
To unsubscribe from this group and stop receiving emails from it, send an email 
to everything-list+unsubscr...@googlegroups.com.
To view this discussion on the web visit 
https://groups.google.com/d/msgid/everything-list/CAFxXSLTsFDps%2BzhudGxrdUuf%2BdxTtRZnBYW0zdxhTutdhq7NAQ%40mail.gmail.com.


Re: Quantum immortality

2019-09-10 Thread 'Brent Meeker' via Everything List



On 9/10/2019 5:35 PM, Jason Resch wrote:



On Tue, Sep 10, 2019 at 7:18 PM 'Brent Meeker' via Everything List 
<mailto:everything-list@googlegroups.com>> wrote:




On 9/10/2019 4:30 PM, Bruce Kellett wrote:
> Another argument that has been given here before is that if quantum
> immortality is true, then we should expect to see a number of
people
> who are considerably older than the normal life expectancy --
and we
> do not see people who are two or three hundred years old. Even
if the
> probabilities are very low, there have been an awful lot of people
> born within the last 500 or so years -- some must have survived
on our
> branch if this scenario is true.

My argument was that each of us should find ourselves to be much
older
than even the oldest people we know.


You could be very old, but (perhaps temporarily) amnesiac.


Then it's strange that so many other people and photographs happen to 
agree with my memory.  Must be a conspiracy to hide the secret of 
quantum immortality.  It would certainly be unpopular once people 
thought about what it means.


Brent

--
You received this message because you are subscribed to the Google Groups 
"Everything List" group.
To unsubscribe from this group and stop receiving emails from it, send an email 
to everything-list+unsubscr...@googlegroups.com.
To view this discussion on the web visit 
https://groups.google.com/d/msgid/everything-list/0c9b0ed2-f183-c862-8503-428da91d0964%40verizon.net.


Re: Quantum immortality

2019-09-10 Thread Jason Resch
On Tue, Sep 10, 2019 at 7:18 PM 'Brent Meeker' via Everything List <
everything-list@googlegroups.com> wrote:

>
>
> On 9/10/2019 4:30 PM, Bruce Kellett wrote:
> > Another argument that has been given here before is that if quantum
> > immortality is true, then we should expect to see a number of people
> > who are considerably older than the normal life expectancy -- and we
> > do not see people who are two or three hundred years old. Even if the
> > probabilities are very low, there have been an awful lot of people
> > born within the last 500 or so years -- some must have survived on our
> > branch if this scenario is true.
>
> My argument was that each of us should find ourselves to be much older
> than even the oldest people we know.
>
>
You could be very old, but (perhaps temporarily) amnesiac.

Jason

-- 
You received this message because you are subscribed to the Google Groups 
"Everything List" group.
To unsubscribe from this group and stop receiving emails from it, send an email 
to everything-list+unsubscr...@googlegroups.com.
To view this discussion on the web visit 
https://groups.google.com/d/msgid/everything-list/CA%2BBCJUgzf_ArKHcfBuXU5EZp5RRtX7555x7EeC3v7wkohiOxLQ%40mail.gmail.com.


Re: Quantum immortality

2019-09-10 Thread 'Brent Meeker' via Everything List




On 9/10/2019 4:30 PM, Bruce Kellett wrote:
Another argument that has been given here before is that if quantum 
immortality is true, then we should expect to see a number of people 
who are considerably older than the normal life expectancy -- and we 
do not see people who are two or three hundred years old. Even if the 
probabilities are very low, there have been an awful lot of people 
born within the last 500 or so years -- some must have survived on our 
branch if this scenario is true.


My argument was that each of us should find ourselves to be much older 
than even the oldest people we know.


Brent

--
You received this message because you are subscribed to the Google Groups 
"Everything List" group.
To unsubscribe from this group and stop receiving emails from it, send an email 
to everything-list+unsubscr...@googlegroups.com.
To view this discussion on the web visit 
https://groups.google.com/d/msgid/everything-list/7ec88255-a555-7f57-eb80-a31c2a559a3d%40verizon.net.


Quantum immortality

2019-09-10 Thread Bruce Kellett

From: *Bruno Marchal* mailto:marc...@ulb.ac.be>>
On 8 Sep 2019, at 13:59, Bruce Kellett <mailto:bhkellet...@gmail.com>> wrote:


On Sun, Sep 8, 2019 at 8:45 PM Bruno Marchal <mailto:marc...@ulb.ac.be>> wrote:


On 7 Sep 2019, at 08:04, Bruce Kellett mailto:bhkellet...@gmail.com>> wrote:

On Sat, Sep 7, 2019 at 3:54 PM 'Brent Meeker' via Everything
List mailto:everything-list@googlegroups.com>> wrote:

On 9/6/2019 10:21 PM, Philip Thrift wrote:

Sean Carroll is on a nationwide speaking tour now
evangelizing Many Worlds.

What is the predictive power of Many Worlds?


None, unless someone can figure out how to derive Born's
rule from it...which I think is impossible.  But it does go
a way toward making the story of measurement more consistent.


Amplify the above statement.

Even Zurek, who starts from a many worlds perspective, thinks
that ultimately one can abandon the non-seen worlds as irrelevant.


But irrelevant does not mean false. So it is irrelevant in
physics, but it is not irrelevant in theology. It might plays a
role concerning the interpretation of death, like with quantum
immortality.


If the only relevance you can find for many worlds is quantum 
immortality, then many worlds is indeed dead. Quantum immortality has 
been shown many times to be a complete nonsense.


Really. I did not known that. Could you give the references.


Follow the Wikipedia entry on quantum suicide. The main problem with the 
idea of quantum immortality is that not all life-threatening events that 
one can encounter are in the form of alternative outcomes to quantum 
processes. Quantum suicide is an attempt to overcome this problem by 
linking death or survival directly to the outcome of a particular 
quantum process. David Deutsch was sceptical that this worked:


 'PhysicistDavid Deutsch <https://en.wikipedia.org/wiki/David_Deutsch>, 
though a proponent of the many-worlds interpretation, states regarding 
quantum suicide that "that way of applying probabilities does not follow 
directly from quantum theory, as the usual one does. It requires an 
additional assumption, namely that when making decisions one should 
ignore the histories in which the decision-maker is absent[M]y guess 
is that the assumption is false."


Tegmark was also doubtful about the chances for quantum immortality -- 
pointing out that dying is rarely a binary event; it is more often the 
result of a slow cumulative process.


Another argument that has been given here before is that if quantum 
immortality is true, then we should expect to see a number of people who 
are considerably older than the normal life expectancy -- and we do not 
see people who are two or three hundred years old. Even if the 
probabilities are very low, there have been an awful lot of people born 
within the last 500 or so years -- some must have survived on our branch 
if this scenario is true.



That would be an indice that Mechanism is false, given that quantum 
immortality is deduce here from the already much more obvious 
arithmetical immortality, which is disturbing, but hard to avoid.



Well, as you know, I consider mechanism to be false in any case, so the 
failure of quantum immortality is no news to me.



Are you saying that quantum suicide is also a non-sense 
(metaphysically, it is a practical non-sense).



It relates to the standard problem for Many worlds theory -- if a 
quantum experiment with binary outcomes is performed many times, there 
will always be observers who see major deviations from the expected 
quantum probabilities. In which case, we cannot rely on repeated 
experiments to be a reliable indicator of the underlying probabilities. 
And if you cannot use long-run relative frequencies to estimate 
probabilities, what do you use? David Wallace attempts to get around 
this by simply dismissing the outliers as "irrelevant" (You, I recall, 
have made a similar argument.) Wallace even suggests that these outlying 
sets of results are "lost in the quantum noise", but he does not 
elaborate on this totally stupid claim. (Wallace, in "The Emergent 
Multiverse" (2012))



If the reference assume a wave packet reduction, or a way “matter” can 
interfere with the computations in arithmetic, no need to give the 
references. It is just working in different theories.


None of this has anything to do with wave-packet reduction, so you can 
rest easy.


Bruce

--
You received this message because you are subscribed to the Google Groups 
"Everything List" group.
To unsubscribe from this group and stop receiving emails from it, send an email 
to everything-list+unsubscr...@googlegroups.com.
To view this discussion on the web visit 
https://groups.google.com/d/msgid/everything-list/f239f2e2-d9ec-96ba-aba6-75d99c9856ac%40optusnet.com.au.


internet immortality

2012-12-12 Thread Roger Clough
I'm not looking for a job.

My apologies for sending out my resume
but I sent it to some yahoolists I belong to
as an attempt  at internet imortality.

Google keeps up on all of our postings.
If I search on Leibniz my name comes up a 
lot from this group.

I might post my resume to google docs.


[Roger Clough], [rclo...@verizon.net]
12/12/2012 
"Forever is a long time, especially near the end." -Woody Allen

-- 
You received this message because you are subscribed to the Google Groups 
"Everything List" group.
To post to this group, send email to everything-list@googlegroups.com.
To unsubscribe from this group, send email to 
everything-list+unsubscr...@googlegroups.com.
For more options, visit this group at 
http://groups.google.com/group/everything-list?hl=en.



Re: Quantum Immortality considering "Passing Out"

2010-05-27 Thread Brent Meeker
When one's mind boggles on hearing a term, like "infinite" that means 
you have no clear understanding of it and if you use the term you 
literally don't know what you're talking about.  I think the infinity of 
the integers is clear enough, and the infinity of the reals, and even 
the infinity of square integrable functions (a Hilbert space).  But when 
someone talks about infinitely many infinite universes coming into being 
at an infinite number of moments within a finite duration - as is 
implied by in some relative state interpretations of QM - then I wonder 
if they know what they're talking about.


Brent

On 5/27/2010 12:43 PM, John Mikes wrote:

/*Stathis wrote:*/
/*You may as well claim that an infinite single universe _should not 
exist_ because it boggles the human mind.*/


**
/*Stathis Papaioannou*/
*/---/*
We are talking *"think of"* rather than* 'exist'* - unless you 
consider it as 'existing in someones boggled mind" as an idea (boggled 
thought, nightmare).

John M

On 5/25/10, *Michael Gough* <mailto:innovative.engin...@gmail.com>> wrote:


The branching is occurring at every moment, so if even one set of
said parents got it on, there would be "umpteen trillons(TM)" of
copies of said individual. It has nothing to do really with the
parents at all. Once you exist, there's umpteen trillions of
copies that stem from the state of the individual at each moment
in time.

On Fri, May 21, 2010 at 5:59 AM, m.a. mailto:marty...@bellsouth.net>> wrote:

- Original Message -
*From:* Stathis Papaioannou <mailto:stath...@gmail.com>
*To:* everything-list@googlegroups.com
<mailto:everything-list@googlegroups.com>
*Sent:* Thursday, May 20, 2010 4:35 PM
*Subject:* Re: Quantum Immortality considering "Passing Out"




On 20/05/2010, at 4:12 PM, "m.a." mailto:marty...@bellsouth.net>> wrote:


I may have this all wrong, but it seems to me that for
there to be umpteen trillion copies of a person there had
to be umpteen trillion (UT) copies of his parents. And
only a relatively small sub-group of those met and
cohabited at the exact moment of his/her conception. But
the same must have been true for their parents and their
parents' parents and so forth back to the primoridal
slime. And this staggering foliation of universes only
covers one specific zygote of two specific gametes. What
of all the other UT^UT combinations leading to the
creation of other individuals just on this family tree?
And what of all the other combinations and histories of
every human, animal, insect and bacterium on this planet?
Does it really make sense to assume numbers of universes
so far beyond our ability to conceive of?marty a.


You may as well claim that an infinite single universe
should not exist because it boggles the human mind.

Stathis Papaioannou
I don't know, Stathis. Somehow it seems easier for me to
conceive of ONE infinite universe than to conceive of
umpteen trillion trillion trillion^umpteen trillion
trillion trillion^umpteen...universes. My "mind" is
obviously more limited than yours. m.a.



--
You received this message because you are subscribed to the Google Groups 
"Everything List" group.
To post to this group, send email to everything-l...@googlegroups.com.
To unsubscribe from this group, send email to 
everything-list+unsubscr...@googlegroups.com.
For more options, visit this group at 
http://groups.google.com/group/everything-list?hl=en.



Re: Quantum Immortality considering "Passing Out"

2010-05-27 Thread John Mikes
*Stathis wrote:*
 *You may as well claim that an infinite single universe should not
existbecause it boggles the human mind.
*

**
*Stathis Papaioannou*
*---*
We are talking *"think of"* rather than* 'exist'* - unless you consider it
as 'existing in someones boggled mind" as an idea (boggled thought,
nightmare).
John M


On 5/25/10, Michael Gough  wrote:
>
> The branching is occurring at every moment, so if even one set of said
> parents got it on, there would be "umpteen trillons(TM)" of copies of said
> individual. It has nothing to do really with the parents at all. Once you
> exist, there's umpteen trillions of copies that stem from the state of the
> individual at each moment in time.
>
> On Fri, May 21, 2010 at 5:59 AM, m.a.  wrote:
>
>>
>>
>>  - Original Message -
>> *From:* Stathis Papaioannou 
>> *To:* everything-list@googlegroups.com
>>  *Sent:* Thursday, May 20, 2010 4:35 PM
>> *Subject:* Re: Quantum Immortality considering "Passing Out"
>>
>>
>>
>>
>>
>>
>> On 20/05/2010, at 4:12 PM, "m.a."  wrote:
>>
>>
>>
>>  I may have this all wrong, but it seems to me that for there to be
>> umpteen trillion copies of a person there had to be umpteen trillion (UT)
>> copies of his parents. And only a relatively small sub-group of those met
>> and cohabited at the exact moment of his/her conception. But the same must
>> have been true for their parents and their parents' parents and so forth
>> back to the primoridal slime. And this staggering foliation of universes
>> only covers one specific zygote of two specific gametes. What of all the
>> other UT^UT combinations leading to the creation of other individuals just
>> on this family tree? And what of all the other combinations and histories of
>> every human, animal, insect and bacterium on this planet? Does it really
>> make sense to assume numbers of universes so far beyond our ability to
>> conceive of?marty a.
>>
>>
>> You may as well claim that an infinite single universe should not exist
>> because it boggles the human mind.
>>
>>
>> Stathis Papaioannou
>>
>> I don't know, Stathis. Somehow it seems easier for me to conceive of ONE
>> infinite universe than to conceive of umpteen trillion trillion
>> trillion^umpteen trillion trillion trillion^umpteen...universes. My "mind"
>> is obviously more limited than yours. m.a.
>>
>>
>>
>>
>>
>> --
>> You received this message because you are subscribed to the Google Groups
>> "Everything List" group.
>> To post to this group, send email to everything-l...@googlegroups.com.
>> To unsubscribe from this group, send email to
>> everything-list+unsubscr...@googlegroups.com
>> .
>> For more options, visit this group at
>> http://groups.google.com/group/everything-list?hl=en.
>>
>>   --
>> You received this message because you are subscribed to the Google Groups
>> "Everything List" group.
>> To post to this group, send email to everything-l...@googlegroups.com.
>> To unsubscribe from this group, send email to
>> everything-list+unsubscr...@googlegroups.com
>> .
>> For more options, visit this group at
>> http://groups.google.com/group/everything-list?hl=en.
>>
>
> --
> You received this message because you are subscribed to the Google Groups
> "Everything List" group.
> To post to this group, send email to everything-l...@googlegroups.com.
> To unsubscribe from this group, send email to
> everything-list+unsubscr...@googlegroups.com
> .
> For more options, visit this group at
> http://groups.google.com/group/everything-list?hl=en.
>

-- 
You received this message because you are subscribed to the Google Groups 
"Everything List" group.
To post to this group, send email to everything-l...@googlegroups.com.
To unsubscribe from this group, send email to 
everything-list+unsubscr...@googlegroups.com.
For more options, visit this group at 
http://groups.google.com/group/everything-list?hl=en.



Re: Quantum Immortality considering "Passing Out"

2010-05-25 Thread Michael Gough
The branching is occurring at every moment, so if even one set of said
parents got it on, there would be "umpteen trillons(TM)" of copies of said
individual. It has nothing to do really with the parents at all. Once you
exist, there's umpteen trillions of copies that stem from the state of the
individual at each moment in time.

On Fri, May 21, 2010 at 5:59 AM, m.a.  wrote:

>
>
> - Original Message -
> *From:* Stathis Papaioannou 
> *To:* everything-list@googlegroups.com
> *Sent:* Thursday, May 20, 2010 4:35 PM
> *Subject:* Re: Quantum Immortality considering "Passing Out"
>
>
>
>
> On 20/05/2010, at 4:12 PM, "m.a."  wrote:
>
>   I may have this all wrong, but it seems to me that for there to be
> umpteen trillion copies of a person there had to be umpteen trillion (UT)
> copies of his parents. And only a relatively small sub-group of those met
> and cohabited at the exact moment of his/her conception. But the same must
> have been true for their parents and their parents' parents and so forth
> back to the primoridal slime. And this staggering foliation of universes
> only covers one specific zygote of two specific gametes. What of all the
> other UT^UT combinations leading to the creation of other individuals just
> on this family tree? And what of all the other combinations and histories of
> every human, animal, insect and bacterium on this planet? Does it really
> make sense to assume numbers of universes so far beyond our ability to
> conceive of?marty a.
>
>
> You may as well claim that an infinite single universe should not exist
> because it boggles the human mind.
>
> Stathis Papaioannou
>
> I don't know, Stathis. Somehow it seems easier for me to conceive of ONE
> infinite universe than to conceive of umpteen trillion trillion
> trillion^umpteen trillion trillion trillion^umpteen...universes. My "mind"
> is obviously more limited than yours. m.a.
>
>
>
>
>
> --
> You received this message because you are subscribed to the Google Groups
> "Everything List" group.
> To post to this group, send email to everything-l...@googlegroups.com.
> To unsubscribe from this group, send email to
> everything-list+unsubscr...@googlegroups.com
> .
> For more options, visit this group at
> http://groups.google.com/group/everything-list?hl=en.
>
>  --
> You received this message because you are subscribed to the Google Groups
> "Everything List" group.
> To post to this group, send email to everything-l...@googlegroups.com.
> To unsubscribe from this group, send email to
> everything-list+unsubscr...@googlegroups.com
> .
> For more options, visit this group at
> http://groups.google.com/group/everything-list?hl=en.
>

-- 
You received this message because you are subscribed to the Google Groups 
"Everything List" group.
To post to this group, send email to everything-l...@googlegroups.com.
To unsubscribe from this group, send email to 
everything-list+unsubscr...@googlegroups.com.
For more options, visit this group at 
http://groups.google.com/group/everything-list?hl=en.



Re: Quantum Immortality considering "Passing Out"

2010-05-25 Thread John Mikes
I am afraid you start from the 2nd step: first you accept whatever 'we'
(humans) think as an evidence in the system we can absorb and evaluate
(explain) and then - *in the framework of that *we imagine our science.
Indeed not much more than a belief system of today.
Not too different from the so called religion, which accepts hearsay as
truth and evidence, the bible as proof and builds on such belief system. The
workings of the world are not shrinkable into such 'truth' we use as much as
we can.
The 'new evidence' - you say - that *overturns* tomorrow today's theory is
just a similar belief.
"Tentative" with a bucketful of pretension - called either scientific or
religious. Flat Earth...?

The whole idea behind my 1st post in this topic was *questioning 'evidence'
via our human restrictedness - vs the unlimit(able)ed workings of the
world*- by far not coverable by us.
We *observe* *what we can and how we can* and *explain *by *what we know*.
It was different in B.C. times, in ~1500AD, yesterday and will be different
500/5000 years from now.
And: I don't buy the nanosec as small, nature can use it as very big, and
vice versa, Brent's timespan can be a 'blinking'. Magnitude-scales are
insecure: we like our body-size median.

John Mikes




On 5/24/10, Stathis Papaioannou  wrote:
>
> On 24 May 2010 23:08, John Mikes  wrote:
> > Stathis,
> >
> > you seemed bored: you jumped into assigning a bit more to my text than it
> > really contained:
> > "...saying that we can know nothing about it at all..."
> > what I did not say. I spoke about a 'hypothetical' functioning of the
> world
> > (read the 'imagining"it")
> > and it refers to how we explain 'it'  (i.e. whatever we 'got' -
> explaining
> > rightly  or wrongly).
> > Bruno assumes that we are digitalizable machines - eo ipso numbers are
> 'in'
> > for him. A religious devotee assumes that we are God's creations - with
> all
> > pertinent explanations and combinations.
> >  I assume "we don't know".
> > The 'system' what conventional sciences developed over the past millennia
> is
> > not so perfect, in spite of all the technology we developed. There are
> > faults (due to imperfections). paradoxes and - "mind boggling". We
> reached
> > such a complicated (complex?) level that nobody dares to start from anew
> in
> > looking into all the facets believed to be "true". Theories are
> sacrosanct,
> > the network is all encompassing and we still do not know a lot of the
> > basics. We assume them. And build on that.
>
> It sounds like you are talking about religion rather than science.
> Science is always tentative: a theory can be overturned tomorrow by
> new evidence, and finding such evidence is one of the most impressive
> things a scientist can do.
>
>
> --
> Stathis Papaioannou
>
> --
> You received this message because you are subscribed to the Google Groups
> "Everything List" group.
> To post to this group, send email to everything-l...@googlegroups.com.
> To unsubscribe from this group, send email to
> everything-list+unsubscr...@googlegroups.com
> .
> For more options, visit this group at
> http://groups.google.com/group/everything-list?hl=en.
>
>

-- 
You received this message because you are subscribed to the Google Groups 
"Everything List" group.
To post to this group, send email to everything-l...@googlegroups.com.
To unsubscribe from this group, send email to 
everything-list+unsubscr...@googlegroups.com.
For more options, visit this group at 
http://groups.google.com/group/everything-list?hl=en.



Re: Quantum Immortality considering "Passing Out"

2010-05-25 Thread Bruno Marchal


On 23 May 2010, at 23:01, Brent Meeker wrote:


On 5/23/2010 9:32 AM, Bruno Marchal wrote:


Hi Alex, hi Quentin,

On 20 May 2010, at 15:19, Quentin Anciaux wrote:


Hi,

2010/5/20 awak 

1. Hello everyone! I'm Alex. I'm a civil engineer with an avid  
passion for
Popular Science books. I'm not a scientist, nor a native English  
speaker, so
please excuse my possible inconsistencies in both Scientific logic  
or
English grammar. Again, sorry if this question has already been  
posed.


2. I've just finished reading Russel Standish's "Theory of  
Nothing" so the
following question, concerning Quantum Immortality, has its base  
in the

information found in this book.

3. From what i understand, Functionalism and Computationalism  
implies that
my consciousness will follow all the world-lines where i live at a  
maximum
age - this considering that there might be a limit to "Quantum  
Immortality",
even though this is in contradiction with the definition of this  
concept;
for the purpose my question let's just say there might be some  
worlds where

i live until 200 yrs.

4. From Wikipedia : "Syncope (pronounced /ˈsɪŋkəpi/) is the  
medical term for
fainting, a sudden, usually temporary, loss of consciousness  
generally
caused by insufficient oxygen in the brain either through cerebral  
hypoxia
or through hypotension, but possibly for other reasons. Typical  
symptoms
progress through dizziness, clamminess of the skin, a dimming of  
vision or
greyout, possibly tinnitus, complete loss of vision, weakness of  
limbs to
physical collapse. These symptoms falling short of complete  
collapse, or a

fall down, may be referred to as a syncoptic episode."

So i take this as evidence that consciousness is not continuous.

5. MY QUESTION: "Why is this possible, for me to pass out, losing my
consciousness because of cerebral hypoxia, hypotension, or because  
i am hit

by someone, considering that Quantum Immortality implies continuous
consciousness"? More to that, shouldn't we find ourselves in  
worlds where we
don't sleep (where we are semi-conscious just like dolphins are  
because they
sleep only with half of their brains) so we don't lose  
consciousness?



Quantum immortality doesn't implies continuous consciousness... it  
just implies that there will always be a next moment. So you can  
passed out but you will eventually wake up.



It is an eternally recurring question/objection to many-worlders. I  
think Quentin is basically right, as far as we agree that QM is  
correct and decoherence does its work. With DM (Digital mechanism,  
actually used by QM) the math is awfully complex. All we can say is  
that the measure one obeys a non boolean sort of quantum logic.
IF DM and/or QM is correct the notion of normality for relatively  
computable histories (the arithmetical world-lines) makes higher  
your survive a cerebral hypoxia in the normal third person sharable  
common reality. For irreversible damages, like with alzheimer, or  
with death, the question of the first person indeterminacy is more  
complex. By a 'galois connection', you normally augment the  
possibilities, but there may be jumps, amnesia, and it may depend  
eventually on what you identify yourself with.


But the "jumps" can be arbitrarily long.  So is a jump of 10^10yrs =  
death?



By jump I was alluding to first person jumps. I do that frequently  
when doing a nap. I am conscious of being somewhere, but to sleepy to  
remember where and when. I am awake, and I know that the opening of my  
eyes will instantaneously get me back in the flux, at some precise  
point. I am always 100% sure to be at morning, in the dark, but once I  
open the eyes I remember quasi instantaneously I was doing a nap, in  
the day light.


Some jump can be amnesic jump, or memory recovery jump. It is a sudden  
change in the subjective experience. It happens also when awakening  
from a REM dream.


Concerning the UD jumps, measured in years, (with an UD time step = 1  
nanosecond, say) we have that, by first person delay invariance, those  
delays don't introduce any change in the subjective experience. Be it  
10^10yrs, or 10^(10^10)yrs, or, if you remember omega[omega]omega  
years. But keep in mind that the uncertainty measure is not on the  
states, but on the histories, and from inside they have a complex  
topology. It is not a boolean measure (accepting Theaetetus's theory  
of knowledge and its arithmetization).



Bruno



Brent



Those 'modern theological' questions are awfully difficult, but  
computer science can translate them into questions (or set of  
questions) of arithmetic (in the DM theory, that is assuming we are  
digitalizable machine).


Bruno

http://iridia.ulb.ac.be/~marchal/



--
You received this message because you are subscribed to the Google  
Groups "Everything List" group.
To 

  1   2   3   4   5   6   >